You are on page 1of 204

Слободан Филиповски

200
ТЕОРИЈА НА БРОЕВИ
подготвителни задачи

Скопје
2013

1
Рецензенти: Слаѓан Станковиќ, Раде Филиповски

Компјутерска обработка: Слободан Филиповски

Печати: АБЦ принт, Скопје

Корица: Трајче Павлов

2
Благодарност до
Советот на општините Радовиш и Конче
за финансиската помош за печатење на ова збирка

Посветено на
Стефанија Ангина

3
С О Д Р Ж И Н А

1. Деливост……………………………………7

2. НЗД И НЗС…………………………………..10

3. Прости броеви……………………………...15

4. Рационални и ирационални броеви………19

5. Модуларна аритметика……………………22

6. Биномни коефициенти…………………….30

7. Квадратни остатоци……………………….36

8. Равенка на Пел……………………………..42

9. Низа на Фибоначи…………………………46

10. Низа на Фареј……………………………...49

Задачи.................................................................51

Задачи и решенија.............................................71

Литература…………………………………….202

Додаток………………………………………..203

4
Предговор

Пишувањето збирка од областа на Теорија на броеви е последица на моето


четиригодишно активно учество во организација на математички натпревари и
олимпијади. Збирката е наменета за талентирани ученици за математика кои сакаат да ги
продлабочат своите знаења од ова област. Дел од задачите се од припремниот материјал
кој го обработивме со олимпијците од приватната гимназија ,,Јахја Ќемал’’.
Содржи 200 задачи селектирани од најразлични математички натпреварувања. Меѓу нив
има и мои авторски. Пред текстот на задачите е дадено објаснување за нивното потекло, за
земјата, годината или рангот на натпреварување. Задачите без такво објаснување не се
најдени во оригинална форма.
Првиот дел од збирката содржи повеќе тематски единици опфатени со најважните
дефиниции, теореми и својства. Со нивно совладување ученикот е спремен за решавање
на најразлични задачи од ова област. На крајот од секоја лекција се дадени неколку
примери поврзани со лекцијата.
Вториот дел го содржат задачите и решенијата. Задачите се подредени по тежина,
најчесто со едно решение, со што му се остава простор на ученикот да најде ново и
поелегантно решение.
Збирката може да им послужи на сите ученици кои се подготвуваат за математички
натпревари и олимпијади, на менторите кои учествуваат во нивните подготовки, на
студентите за предметите Теорија на броеви и Криптографија, како и на секој вљубеник во
математиката.
На крај, Голема Благодарност на сите кои на некаков начин помогнаа во
реализација на ова збирка, а посебно на мојот пријател и колега Здравко Цветковски кој
несебично учествуваше во нивна реализација со неговите сугестии и коментари.

Од авторот,
Скопје, 2013

5
Да прашаш некого која е убавината на броевите е исто
како да прашаш која е убавината на деветата симфонија на Бетовен.
Ако сам не ја откриеш, верувај дека никој нема да ти ја покаже..
Јас знам зошто броевите се убави. Ако тие не беа убави,
тогаш ништо немаше да биде...

Пол Ердош

6
1. ДЕЛИВОСТ

Дефиниција 1.1. Целиот број a го дели целиот број b, означуваме a b, ако постои цел
број c така што b  ac.

Теорема 1.2. Алгоритам за делење. За секој природен број a и цел број b, постојат
единствени цели броеви q и r така што b  qa  r и 0  r  a.

Карактеристични критериуми за деливост:

Нека a  a n a n 1 ...a1 a0 е даден природен број.

1) 8 a ако и само ако 8 4a2  2а1  а0 .

2) 9 a ако и само ако 9 an  an1  ....  a0 .


3) 11 a ако и само ако a0  a1  a2  a3  ...   1 an  0.
n

4) Бројот а се дели со 7,11 или 13 ако и само ако бројот аn an1....a3  a2 a1a0 го има
истото својство.
5) Бројот а се дели со 27 или 37 ако и само ако бројот аn an1....a3  a2 a1a0 го има
истото својство.
6) Бројот a се дели со 2k или 5k  k  n ако и само ако бројот аk 1ak 2. ....a0 го има
истото својство.

Нека n  p11  p22  pkk , каде pi се прости делители на бројот n.

Со d n  го означуваме вкупниот број на делители на n, со   n  збирот на сите делители


на n.

Важат формулите:

p11 1  1 p22 1  1 pkk 1  1


d n  1  1 1   2     1   k  и   n     .
p1  1 p2  1 pk  1

7
200 ТЕОРИЈА НА БРОЕВИ - подготвителни задачи

Примери
Пример 1.1. Нека x и y се цели броеви. Докажи дека 2 x  3 y се дели со 17 ако и само
ако 9 x  5 y се дели со 17.

Решение. Важи 17  2 x  3 y  17 13  2 x  3 y   17  26 x  39 y  

17 17 x  34 y    9 x  5 y  17  9 x  5 y  .

Пример 1.2. Романија 2002. Нека p, q се различни прости броеви. Докажи дека постојат
природни броеви a, b така што аритметичката средина на сите природни делители на
бројот n  p a q b е природен број.

Решение. Збирот на сите делители на бројот n е даден со формулата


1  p  p 2
 
 ....  p a 1  q  q 2  ...  q b . Бројот n има a  1b  1 природни делители па

затоа нивната аритметичка средина е s 


1  p  p 2
 
 ....  p a 1  q  q 2  ...  q b
.
a  1b  1
Ако p и q се непарни броеви избираме p  a, q  b и лесно се гледа дека s е природен
број. Ако p  2 и q е непарен број, тогаш избираме b  q, a  1  1  q 2  ...  q q 1 . Во овој
случај s  1  2  2 2  ...  2 a . Аналогно се избира ако p е непарен и q  2 .

Пример 1.3. Чешка 2011. Најди ги сите парови од природни броеви m, n  за кои m  n 
2

го дели бројот 4mn  1.

Решение. Бидејќи m  n2  0 следува дека m 2  2mn  n 2  4mn, односно


4mn  m  n .
2

Од условот на задачата следува m  n  4mn  4. Значи 4mn  m  n  4mn  4.


2 2

Сега јасно е дека m  n  4mn или m  n  4mn  4.


2 2

Во првиот случај добиваме m  n т.е. 4m 2 4m 2  4 , што повлекува дека m  n  1.

Во вториот случај добиваме m  n  4 т.е. m  n  2.


2

8
200 ТЕОРИЈА НА БРОЕВИ - подготвителни задачи

Пример 1.4. Докажи дека бројот 34  45 е производ на два цели броеви поголеми од
5 6

102002.
4
56 1
1  5 1  b4
6

44 4
Решение. Нека a  3 44
и b4 4
. Тогаш важи 3  3 45
 a и 4   4 4  44   .
4 56

4  
 4
Добиваме
2 2
b4  b2   b2   b2  b2 
       a 2     ab    a 2   ab  a 2   ab  .
2 2
34  45  a 4   a2
5 6

4  2  2  2  2 
56 1
25 1
2 6
b2  b  b2 b2 4 2
 
2002
Важи a   ab   a       2  25 1  210008  24  102002.
6
2

2  2  4 4 4 2

Пример 1.5. Нека k е парен број. Дали е можно да го запишиме бројот 1 како збир од
реципрочности на k непарни цели броеви?
Решение. Нека претпоставиме дека постојат непарни цели броеви n1 , n2 ,..., nk така што
1 1 1
1   ...  . Добиваме дека n1n2  nk  m1  m2  ...  mk , каде mi  n1n2 ni 1  ni 1 nk ,
n1 n2 nk
за i 1, 2,..., k . Бројот n1n2  nk е непарен бидејќи е производ на непарни броеви, додека
бројот m1  m2  ...  mk е парен, бидејќи е збир на k -(парен број) на непарни броеви.
Оттука такво запишување на бројот 1 не е можно.
Ако k е непарен број тогаш такво запишување е можно.
Пример,

1 1 1 1 1 1 1 1 1
k  9; 1          .
3 5 7 9 11 15 35 45 231

Пример 1.6. Определи ги сите петцифрени броеви од облик 37abc такви што секој од
броевите 37abc ,37bca и 37cab е делив со 37.
Решение. Бројот 37abc се дели со 37 ако и само ако бројот abc се дели со 37.
Нека x  abc  100a  10b  c, y  bca  100b  10c  a, z  cab  100c  10a  b.
Со одземање добиваме 10 x  y  999a, 10 y  z  999b, 10 z  x  999c. Бидејќи 999  37  27
добиваме дека ако еден од броевите x, y и z е делив со 37 тогаш такви се и другите два.
Значи, сите барани броеви се броеви чии последни три цифри формираат број кој се дели
со 37, односно броевите 37000,37037,37074,....,37999.

9
2. НЗД И НЗС

Дефиниција 2.1. Најголем заеднички делител на два природни броеви a и b е


најголемиот природен број кој ги дели a и b, означуваме со НЗД a, b.
Најмал заеднички содржател на два природни броеви a и b е најмалиот природен број
кој се дели со a и b, означуваме со НЗС a, b.

Дефиниција 2.2. За природните броеви a и b велиме дека се заемно прости ако


НЗД a, b  1.

Својство 2.3.

1) НЗД  a, b   НЗС  a, b   ab,


2) НЗД  ma, mb   m  НЗД  a, b  , НЗС  ma, mb   m  НЗС  a, b  ,
3) НЗД  n, n  1  1,

4) НЗД  a, b  НЗС  a, b  ,
5) НЗД a, b  НЗД a  bc, b .

Теорема 2.4. (Идентитет на Безу) Ако НЗД  a, b   d тогаш постојат цели броеви x 0 и
y 0 така што d  ax0  by0 .

Доказ. Нека c е најмалиот природен број од облик ax  by, т.е. нека постојат цели броеви
x0 и y0 така што c  ax0  by0 . Ако a не се дели со c тогаш постојат природни броеви q и
r  c така што a  cq  r. Добиваме дека r  a  cq  a   ax0  by0  q  a 1  qx0   bqy0  0

и r е број од облик ax  by , помал од c, што не е можно. Значи c a.

На ист начин се докажува дека c b. Бидејќи НЗД  a, b   d следува дека постојат цели
броеви a1 и b1 така што a  a1d и b  b1d . Оттука добиваме c  ax0  by0  d  a1 x0  b1 y0  ,

односно d c. Значи d  c. Од друга страна од c a и c b следува дека c е заеднички


делител на броевите a и b, но бидејќи d е нивниот најголем заеднички делител имаме
дека c  d . Од d  c и c  d следува c  d .

10
200 ТЕОРИЈА НА БРОЕВИ - подготвителни задачи

Теорема 2.5. Ако а  bq  r тогаш НЗД  a, b   НЗД  b, r  .

Доказ. Нека НЗД  а, b   d . Постојат цели броеви а1 и b1 така што а  a1d , b  b1d .

Добиваме a1d  b1dq  r , од каде следува дека d r. Бидејќи d b имаме d НЗД  b, r  .

Ако d  НЗД  b, r  постои природен број d1 таков што d1  НЗД  b, r  .

Од а  bq  r следува дека d1 a односно d1 е заеднички делител на броевите а и b.

Последното не е можно бидејќи d е најголемиот заеднички делител на броевите а и b.

Останува дека важи d  НЗД  b, r   НЗД  a, b  .

Нека a и b се природни броеви и нека q1 е количникот а r1 е остатокот при делње на a со


b. Следните равенства се познати како Евклидов алгоритам:

a  bq1  r1 , 0  r1  b
b  r1q2  r2 , 0  r2  r1
r1  r2 q3  r3 , 0  r3  r2
........... .......
rk  2  rk 1qk  rk , 0  rk  rk 1
rk 1  rk qk 1  rk 1 , 0  rk 1  rk
........... .......

Низата ri  е монотоно опаѓачка т.е. важи r1  r2  ...  rk  .. , па ваквата постапка ќе


заврши по конечен број на чекори, односно постои природен број n така што rn1  qn1rn
односно rn1  0. Оттука јасно е дека rn rn1.

Теорема 2.6. Последниот остаток rn кој е различен од нула во Евклидовиот алгоритам е


еднаков на најголемиот заеднички делител на броевите a и b.

Доказ. Од Теорема 2.5. важи НЗД  a, b   НЗД  b, r1   НЗД  r1 , r2   ...  НЗД  rn1 , rn   rn .

Својство 2.7. Нека m и n се два заемно прости броеви. Ако k е цел број таков што
важи m k и n k тогаш следува дека mn k .

11
200 ТЕОРИЈА НА БРОЕВИ - подготвителни задачи

Доказ. Од m k следува дека постои цел број x така што важи k  mx. Значи n mx .
Бидејќи НЗД m, n  1 следува дека n x т.е. постои цел број y таков што x  ny. Оттука
добиваме дека k  mx  mny, односно mn k .

Својство 2.8. Нека a  p11  p22  pkk и b  p11  p22  pkk . Тогаш важи

max 2 , 2 
НЗС  a, b   p1max1 , 1  p2
max k , k 
 pk .

Примери
Пример 2.1. С.Ф. Пресметај НЗД a, b, c  ако a, b и c се природни броеви за кои важи
ab  bc  ca  2012 и притоа НЗД a, b, c   1 .

Решение. Нека НЗД a, b, c   d  1 . Постојат природни броеви a1 ,b1 и c1 за кои


a  a1d , b  b1d и c  c1d . Од условот на задачата ab  bc  ca  2012 добиваме дека важи
2012  d 2 a1b1  b1c1  a1c1   2 2  503 . Бидејќи 503 е прост број следува дека d  2 .

21n  4
Пример 2.2. ИМО 1959. Докажи дека за било кој природен број n дропката
14n  3

не може да се скрати.

Решение. Нека d  1 е најголемиот заеднички делител за 21n  4 и 14n  3.

Ако a и b се природни броеви и a  b тогаш НЗД  a, b   НЗД  a, b  a  .

НЗД 21n  4,14n  3  НЗД 14n  3, 21n  4  14n  3  НЗД 14n  3, 7n  1 


 НЗД 7n  1,14n  3  7n  1  НЗД 7n  1, 7n  2  НЗД 7n  1, 7n  2  7n  1 
 НЗД 7n  1,1  1.

21n  4
Според тоа дропката е нескратлива.
14n  3

Пример 2.3. Нека НЗД  a, b   1. Докажи дека НЗД  a  b, а 2  b2   1 или 2.

Решение. Нека НЗД  a  b, a 2  b2   d . Од d a  b и d  a  b   2ab следува дека


2

12
200 ТЕОРИЈА НА БРОЕВИ - подготвителни задачи

d 2ab. Ако d a или d b тогаш од d a  b следува дека d a и d b, што не е можно


бидејќи a и b се заемно прости броеви. Останува d 2 т.е. d  1 или d  2.

Пример 2.4. Нека Tn  22  1, за секој ненегативен цел број n. Докажи дека ако m  n
n

тогаш Tm и Tn се заемно прости броеви.

Решение. Имаме:

 1  Tn1  1  1  Tn21  2Tn1  Tn1 Tn1  2.


n 1
2
Tn  2  2 2  1  2 2
n 2

Според тоа за секој природен број n важи

Tn  2  Tn1 Tn1  2  Tn1  Tn2 Tn2  2  ...  Tn1  Tn2  ..  T0  T0  2  Tn1  Tn2  ..  T0 .

Значи Tn  Tn1Tn2  T0  2 и Tm  Tm1Tm2  T0  2. Без губење на општоста нека m  n.

Тогаш Tn Tm1Tm2  T0 т.е. Tn Tm  2. Ако НЗД Tn , Tm   d тогаш важи d Tn Tm  2 ,

и бидејќи d Tm следува дека d 2. На крај добиваме d  1, бидејќи d е непарен број.

Значи броевите Tm и Tn се заемно прости.

Пример 2.5. Бугарија 2001. Најди ги сите тројки од природни броеви  a, b, c  така што
бројот a  b  c се дели со броевите a b, b c и c a.
3 3 3 2 2 2

Решение. Нека d  НЗД  a, b  . Јасно е дека d 3 a 2b , па од условот на задачата следува


дека d 3 a3  b3  c3 . Бидејќи d 3 a 3 и d 3 b3 следува дека d 3 c3 т.е. d c. Оттука најголемиот
заеднички делител на два од броевите a, b и c е најголемиот заеднички делител на a, b и
a b c
c. Нека  r , s, t    , ,  . Тогаш тројката  r , s, t  ги задоволува условите на задачата, и
d d d 
r , s и t се попарно заемно прости броеви. Јасно е дека r 2 r 3  s3  t 3 , s 2 r 3  s3  t 3 и

t 2 r 3  s3  t 3 , и бидејќи r , s и t се попарно заемно прости следува дека r 2 s 2t 2 r 3  s3  t 3 .


Ќе докажеме дека  r , s, t   1,1,1 и пермутациите на тројката 1, 2,3 се сите решенија за
 r , s, t  . Без губење на општоста земаме дека r  s  t.

13
200 ТЕОРИЈА НА БРОЕВИ - подготвителни задачи

s 2t 2
Важи 3r 3  r 3  s3  t 3  r 2 s 2t 2 од каде следува дека r  . Јасно е дека r 2 s3  t 3 од
3
s 4t 4
каде следува 2s3  s 3  t 3  r 2  . ..(1).
9
18 18
Ако t  2 тогаш s    2  t , што не е можно бидејќи s  t. Останува t  1.
t 4 16
Ако s  1 тогаш треба да биде исполнето r 2 r 3  2 , кое важи само за r  1.
Значи  r , s, t   1,1,1 . Ако s  2 тогаш r  s бидејќи r и s се заемно прости броеви.
s2
Добиваме 2r 3  r 3  s3  1  r 3  s3  t 3  r 2 s 2t 2  r 2 s 2 , т.е. r  .
2
s4
односно s  4. Со проверка добиваме дека  3, 2,1 е
Од (1) имаме s3  1  s 3  t 3  r 2 
4
тројка која ги задоволува условите на задачата.
Значи  d , d , d  ,  3d , 2d , d  , 3d , d , 2d  ,  2d ,3d , d  ,  2d , d ,3d  ,  d , 2d ,3d  и  d ,3d , 2d  се сите
тројки природни броеви кои го исполнуваат условот на задачата.

14
3. ПРОСТИ БРОЕВИ

Теорема 3.1. ( Евклид). Постојат бесконечно многу прости броеви.

Доказ. Нека претпоставиме спротивно, т.е. нека постојат конечен број n на прости броеви
p1 , p2 , ....., pn . Го разгледуваме природниот број p  p1 p2    pn  1 . Бидејќи секој
природен број има единствена канонична факторизација следува дека постои i  1, 2.,..., n
така што pi p . Оттука добиваме pi p1 p2    pn  1 . Бидејќи pi p1 p2    pn следува pi 1 ,
што не е можно. Од добиената контрадикција следува дека постојат бесконечно многу
прости броеви.

Својство 3.2. 1) p ab  p a или p b,

2) p a1a2    ak  p ai за некој i  1, 2, ..., k.

Доказ. 1) Нека претпоставиме дека p не го дели а. Тогаш НЗД  a, p   1 , и од идентитет


на Безу следува дека постојат цели броеви m, n така што am  pn  1, од каде следува дека

abm  pbn  b. Бидејќи p abm и p pbn следува дека p b. Слично, ако претпоставиме
дека p не го дели b тогаш ќе добиеме дека p го дели a.

Теорема 3.3. Фундаментална Теорема на Аритметиката. Секој цел број поголем од 1

може на единствен начин да биде запишан во облик p11  p22  pkk , каде p1 , p 2 ,..., p k се
различни прости броеви и  1 ,  2 ,....,  k се природни броеви.

Ваквото претставување се нарекува канонична факторизација на даден број.


n 1
Теорема 3.4. Ако p n е n -тиот прост број, тогаш pn  2 2 .

Доказ. Теоремата ќе ја докажеме со помош на математичка индукција по n.


n 1
За n  1 имаме p1  2  2  2 2 . Нека претпоставиме дека за 2  n  k важи pn  2 2 .
0

Од индуктивната претпоставка добиваме:


k 1
... 2k 1
pk 1  p1 p2    pk  1  2  2 2  2 2    2 2  1  2122  1  22 1
 1  22 .
2 2 k k

15
200 ТЕОРИЈА НА БРОЕВИ - подготвителни задачи

Теорема 3.5. Ако n  5 , тогаш помеѓу n и 2n постојат барем два прости броеви.

Својство 3.6. (Постулат на Бертранд). Ако n  3 , тогаш помеѓу n и 2n  2 постои


барем еден прост број.

Доказ. Ако n  4 тогаш меѓу 4 и 6 постои прост број, бројот 5.

Ако n  5 тогаш меѓу 5 и 8 постои прост број, бројот 7.

Ако n  5 тогаш од Теорема 3.5. следува дека помеѓу n и 2n постојат барем два прости
броеви. Ако поголемиот од нив е бројот 2n  1 , бидејќи 2n  2  2n  1 е сложен број
следува дека помалиот прост број е помал од 2n  2 .

Теорема 3.7. Постојат бесконечен број на прости броеви од облик 4k  3.

Доказ. Нека претпоставуваме спротивно, т.е. нека постојат конечен број n на прости
броеви од облик 4k  3 , p1 , p2 ,...., pn . Го разгледуваме бројот p  4 p1 p2    pn  1.

Јасно p  4 p1 p2    pn  1  3 е непарен број од облик 4k  3.

Сите прости делители на p се броеви од облик 4k  1 и 4k  3. Ако сите се од облик


4k  1 тогаш и бројот p е од таков облик. Значи постои најмалку еден прост делител на p
од облик 4k  3 т.е. постои i  1, 2, ..., n така што pi p.

Бидејќи pi 4 p1 p2    pn следува дека pi p  1 . Последното не е можно бидејќи pi p.

Теорема 3.8. Секој прост број поголем од 3 има остаток 1 или 5 при делење со 6.

Доказ. Нека p  3 е прост број. Ако p  0 mod 6 тогаш p  6k е делив со 6.

Ако p  2 mod 6 тогаш p  6k  2  23k  1 е делив со 2.

Ако p  3 mod 6 тогаш p  6k  3  32k  1 е делив со 3.

Ако p  4 mod 6 тогаш p  6k  4  23k  2 е делив со 2.

Останува дека секој прост број поголем од 3 има остаток 1 или 5 при делење со 6.

Теорема 3.9. (Дирихле) Ако a и b се заемно прости броеви тогаш постојат бесконечно
прости броеви од облик a  kb, k  0,1, ......

Теорема 3.10. За секој природен број k , постојат k последователни сложени броеви.

16
200 ТЕОРИЈА НА БРОЕВИ - подготвителни задачи

Доказ. Ја разгледуваме низата од k последователни природни броеви:

k  1!2, k  1!3,......, k  1!k  1.


Лесно се гледа дека i k  1!i, i  2, 3, ...., k  1 бидејќи од i  k  1 следува i k  1!.

Дефиниција 3.11. За простите броеви p и q велиме дека се близнаци прости броеви ако
p  q  2.

Пример: 3, 5; 11,13; 1997,1999; ..

Примери

Пример 3.1. Ако p  5 е прост број, докажи дека p 2  2 е сложен број.

Доказ. Бидејќи p  1mod 6 добиваме дека p 2  1mod 6. Оттука p 2  2  3 mod 6 , т.е.

p 2  2  6k  3  32k  1 е делив со 3.

Пример 3.2. Ако n  1 е сложен број тогаш n има прост делител p  n .

Решение. Бидејќи n е сложен број следува дека постојат природни броеви 1  a, b  n така
што n  ab. Барем еден од броевите a и b е помал од n бидејќи во спротивно би важело
n  ab  n  n  n, што не е можно. Без губење на општоста земаме a  n .

Ако a е прост број тогаш доказот е завршен. Во спротивно a има прост делител за кој
важи p  a  n .

Пример 3.3. (Предлог ЈБМО 2000). Најди ги сите природни броеви а и b за кои
a 4  4b 4 е прост број.

Решение. Бројот a 4  4b 4 го разложуваме на следниот начин:

a 4  4b 4  a 2   2b 2   a 2  2b 2   2ab 
2 2 2 2

 a 2  2ab  2b 2 a 2  2ab  2b 2 .

За a  2 или b  2 добиваме a 2  2b 2  3ab  2ab  1 и a 2  2ab  2b 2  a 2  2ab  2b 2 ,

17
200 ТЕОРИЈА НА БРОЕВИ - подготвителни задачи

па затоа добиваме дека a 4  4b 4 може да се претстави како производ на два различни


природни броеви поголеми од еден. Во овој случај a 4  4b 4 е сложен број. Ако a  b  1
добиваме a 4  4b 4  5 , кој е единствен прост број од ваков облик.

Пример 3.4. Нека n и 8n 2  1 се два прости броеви. Докажи дека и бројот 8n 2  1 е


прост број.

Решение. Ако n  2 тогаш 8n 2  1  33 не е прост број.

Ако n  3 тогаш 8n 2  1  73 е прост број. Исто така и 8n 2  1  71 е прост број.

Сега нека n  3 е прост број. Тогаш важи n 2  1mod 6.

Оттука 8n 2  1  9  3 mod 6, што не е можно бидејќи секој прост број поголем од три има
остаток еден или пет при делење со шест.

Останува n  3 е единствен прост број кој ги исполнува условите на задачата.

Пример 3.5. а) Докажи дека за n  2 постои прост број p за кој важи n  p  n!.

б) За n  1 докажи дека секој прост делител на бројот n !1 е непарен број поголем од n.

Доказ. а) Го разгледуваме бројот n !1 и нека p е негов прост делител. Ако n !1 е прост
број тогаш p  n!1. Ако n !1 е сложен број тогаш секој негов делител е поголем од n.

Ако 2  d  n тогаш d n !, па оттука е јасно дека d не е делител на n !1 .

Според тоа за p избираме прост делител на n !1 , за кој ќе важи n  p  n!.

б) Бидејќи n  1 следува дека n ! е парен број, па оттука n !1 е непарен број.

Значи сите прости делители на n !1 се непарни. Нека p n !1 , p е прост број.

Ако 2  p  n тогаш p n !. Од p n !1 и p n ! добиваме дека p НЗД n!, n!1  1 ,

што не е можно.

18
4. РАЦИОНАЛНИ И ИРАЦИОНАЛНИ БРОЕВИ
p
Дефиниција 4.1. Секој број кој може да се претстави како дропка од два цели броеви
q
p и q , каде q  0, се нарекува рационален број. Во спротивно бројот е ирационален.

Множеството од рационални броеви го означуваме со Q, додека од ирационалните


броеви со I .

Својство 4.2. Децималниот запис на секој рационален број е формиран од конечен број на
цифри или од низа од цифри која периодично се повторува.

Својство 4.3. Помеѓу секои два рационални броеви постои ирационален број.

Својство 4.4. Помеѓу секои два ирационални броеви постои рационален број.

Теорема 4.5. Ако p е прост број тогаш p е ирационален број.

Доказ. Нека претпоставиме дека p е рационален број. Тогаш постојат заемно прости
a
броеви a, b така што p  . Оттука важи a 2  pb 2 , па според тоа p a.
b

Постои цел број c за кој a  cp. Имаме c 2 p 2  pb 2 т.е. c 2 p  b 2 .

Следува дека p b. Добиваме дека простиот број p ги дели целите броеви a и b ,

што е противречност со НЗД a, b  1.

Теорема 4.6. Ако n не е полн квадрат тогаш n е ирационален број.

Доказ. Слично како доказот на Теорема 4.5.

Последица 4.7. Од Теорема 4.5. следува дека 2 и 3 се ирационални броеви, бидејќи 2


и 3 се прости бреови.

Примери
Пример 4.1. Естонија 2008. Нека x, y, z се агли на даден триаголник, (во степени).

x y z
а) Докажи дека ако , , се рационални броеви, тогаш x, y, z се исто така рационални
y z x
броеви.

19
200 ТЕОРИЈА НА БРОЕВИ - подготвителни задачи

x y z
б) Докажи дека ако точно еден од броевите , , е рационален, тогаш x, y, z се
y z x
ирационални броеви.

180 x  y  z y z
Решение. Забележуваме дека важи   1  .
x x x x

x y 1
а) Бидејќи е рационален број тогаш и бројот  е рационален број. Според тоа
y x x
y
180
добиваме дека е рационален број бидејќи е збир од три рационални броеви.
x

Оттука x е рационален број. На ист начин се докажува дека и y и z се рационални


броеви.

x y z
б) Без губење на општоста претпоставуваме дека е рационален број и , се
y z x
y z x
ирационални броеви. Следува дека е рационален број, додека и се ирационални
x y z
180
броеви. Значи е збир од два рационални и еден ирационален број, па според тоа е
x
x
ирационален број. Оттука x е ирационален број. Бидејќи е рационален број следува
y
дека y е ирационален број. Сега нека претпоставиме дека z е рационален број. Тогаш

x y
x  y  180  z е рационален број. Добиваме дека е ирационален број бидејќи е
y
x y x
количник од рационален и ирационален број. Од друга страна  1  е рационален
y y
број бидејќи е збир од два рационални броеви. Се добива контрадикција. Според тоа z е
ирационален број.

Пример 4.2. Индонезија 2005. Нека k и m се природни броеви такви што


1
 k  4 m  k  е цел број. Докажи дека k е рационален број.
2 

1
Решение. Нека  k  4 m  k   c , каде c е цел број. Имаме k  4 m  2c  k
2 

и по квадрирање добиваме дека m  c 2  c k т.е. m  c 4  c 2 k  2c 3 k .

20
200 ТЕОРИЈА НА БРОЕВИ - подготвителни задачи

m  c4  c2k
Значи, k  Q .
2c 3

Пример 4.3. Докажи дека бројот 2  3 3 е ирационален број.

Решение. Нека претпоставиме спротивно, т.е. бројот 2  3 3 е рационален. Постојат


p
цели броеви p и q , каде q  0, и притоа важи 233 .
q

 
2
3
p  p3 3 p 2 2 3 p 2
  p3 3 p 2 2 6 p
3
Имаме 3    2   3    2    2 2 
q  q q2 q q3 q2 q

p3 6 p  3 p2 
 3
  2  2  2 .
q q  q 

p3 6 p
 3
q3 q p3  6 pq 2  3q 3
Сега добиваме 2  т.е. 2 е рационален број, што е
3 p2 3 p 2 q  2q 3
 2 q 3

q2
противречност. Значи 2  3 3 е ирационален број.

Пример 4.4. Нека S (n) е збирот на цифрите на бројот n . После запирката ги пишуваме
еден по друг броевите S (1), S (2),... . Докажи дека бројот што се добива е ирационален.

Решение. Бројот 0, S (1)S (2)... е ирационален ако е непериодичен. Да претпоставиме дека


бројот е периодичен и нека периодот има должина d . Јасно е дека периодот мора да
содржи цифри различни од 0 бидејќи во спротивно бројот би бил од облик
0, S (1)S (2)...S (k ) и секогаш постои број поголем од k чиј збир на цифри е различен од 0,
(на пример 10 m за доволно голем m е поголем од k а збирот на цифри му е 1). Меѓутоа,
постои доволно голем природен број чиј што збир на цифри завршува на 2d нули, (11...11
со 10 2 d единици е таков што е поголем од збирот на цифри му е 10 2 d ). Тогаш периодот со
должина d мора да се јави целосно во бројот S (10 2d ) , од каде мора да се состои само од
нули.

21
5. МОДУЛАРНА АРИТМЕТИКА

Конгруенции
Дефиниција 5.1. Нека m е природен број и a и b се цели броеви.

Велиме дека a е конгруентно со b по модул m ако m a  b, и означуваме a  b mod m.

Теорема 5.2. Ако a  b mod m и c  d mod m тогаш a  c  b  d mod m и

ac  bd mod m.

Својство 5.3. За секој природен број n важи

0, n  4k
0, n  2k 0, n  3k
n 
2
 mod 4  , n  1, n  2k  1 mod 8 ,
2
n2   mod 3 .
1, n  2 k  1 4, n  4k  2 1, n  3k  1,3k  2

Својство 5.4. Нека x е природен број и p е прост број така што НЗД  x, p   1.

Ако за природните броеви m и n важи x m  1 mod p  и x n  1 mod p  тогаш важи

x НЗД  m, n   1 mod p  .

Доказ. Нека НЗД  m, n   d . Од теорема на Безу постојат цели броеви m0 и n0 така што
важи d  mm0  nn0 .

Ако m0 , n0  0 добиваме x НЗД  m, n  x mm0  nn0   x m    x n   11  1 mod p  .


m0 n0

Сега, без губење на општоста нека m0  0 и n0  0.

x mm0
т.е. x НЗД  m, n   x НЗД  m, n    x n    x m   1 mod p  .
 n0
Добиваме дека важи x НЗД  m, n  
m0
 nn0
x

Својство 5.5. За сите цели броеви x, m и n такви што m, n  0 важи

НЗД x m  1, x n  1  x НЗД m, n   1.


Доказ. Ќе докажеме дека НЗД x m  1, x n  1 x  НЗД  m, n 
 1 и x НЗД  m, n  1 НЗД  x m  1, x n  1 .

22
200 ТЕОРИЈА НА БРОЕВИ - подготвителни задачи

 
Нека d  НЗД x m  1, x n  1 . Тогаш x m  1 mod d  и x n  1 mod d  . Нека k  НЗД  m, n  .

Од теорема на Безу постојат цели броеви m0 и n0 така што k  mm0  nn0 .

Добиваме x НЗД  m, n  1  x mm0 nn0  1   x m    x n   1  1  1  0  mod d  .


m0 n0

Со ова докажавме дека НЗД x m  1, x n  1 x   НЗД  m, n 


 1 . ...(1).

Сега нека x НЗД  m, n   1  d . Јасно е дека важи x НЗД  m, n   1 mod d  , па оттука добиваме

   
m n

x m  1  x НЗД  m, n  НЗД  m , n 
 1  0  mod d  и x n  1  x НЗД  m, n НЗД  m, n
 1  0  mod d  .

Бидејќи d x m  1 и d x n  1 следува дека x


НЗД  m , n 

 1 НЗД x m  1, x n  1 . ...(2). 
Од (1) и (2) следува дека идентитетот важи.

Ојлерова функција, Теорема на Ојлер, Теорема на Ферма,

Теорема на Вилсон, Кинеска теорема за остатоци

Дефиниција 5.6. Нека n  p11 p2 2   pk k е канонична факторизација на природниот број n.

Ојлеровата функција за бројот n го пресметува бројот на сите броеви помали од n кои


се заемно прости со n , означуваме со  n .

Теорема 5.7. Ако p е прост број и k  1 тогаш  p k  p k  p k 1 .  


Доказ. Нека n  p k , каде p е прост број. Единствени броеви меѓу 1 и p k кои не се заемно
прости со p k се броевите p, 2 p,3 p,... p k 1 p. Значи нивниот број е p k 1. Оттука добиваме
дека   n     p k   p k  p k 1.

Теорема 5.8. (Ојлерова формула). Ако n  p11 p2 2   pk k е даден природен број тогаш

 1  1   1 
 n   n1  1      1  .
 p1  p2   p k 

23
200 ТЕОРИЈА НА БРОЕВИ - подготвителни задачи

Доказ. Ако n  p k , тогаш од Теорема 5.7. добиваме дека  p k  p k  p k 1 .  


Нека n  p11 p22  pkk . Бидејќи Ојлеровата функција е мултипликативна т.е. важи
  ab     a    b  , (докажи), добиваме:

 1  2  1   1 
  p1 p2  pk     p1    p2     pk   p1 1 
1 2 k 1 2 k 1 k
 p2 1    pk 1   
 p1   p2   pk 

 1  1   1   1  1   1 
 p11 p22  pkk 1  1    1    n  1  1    1   .
 p1  p2   pk   p1  p2   pk 

Својство 5.9. Ако p е прост број тогаш   p   p  1.

Теорема 5.10. (Ојлер) Ако a е цел број заемно прост со n , тогаш a  n   1mod n.

Доказ. Нека a1 , a2 ,..., a n  се сите природни броеви помали и заемно прости со n.

Ги разгледуваме броевите aa1 , aa2 ,...., aa n  . Бидејќи НЗД a, n  1 и НЗД аi , n  1
следува дека НЗД ааi , n  1. Бидејќи ai  n следува дека постои природен број
k  1, 2, ...,  n така што за секој i  1, 2, ...,  nважи aai  ak mod n. Ако постојат i, j
такви што aai  aa j  ak mod n тогаш бидејќи НЗД a, n  1 следува дека ai  a j mod n.
Последната конгруенција не е можна бидејќи ai , a j  n, ai  a j .

Останува дека броевите aa1 , aa2 ,...., aa n  се конгруентни со некоја пермутација на
броевите a1 , a2 ,..., a n  по модул n.

Добиваме a1a2    a n   aa1 aa2     aa n    a  n  a1a2    a n  mod n.

Од последната конгруенција добиваме a  n   1mod n.

Последица 5.11. (Теорема на Ферма) Нека p е прост број кој е заемно прост со целиот
број a. Тогаш a p 1  1mod p .

Доказ. Ако n  p тогаш  n    p   p  1 и од Ојлеровата теорема веднаш се докажува


бараното тврдење.

24
200 ТЕОРИЈА НА БРОЕВИ - подготвителни задачи

Својство 5.12. Секој прост делител p  3 на број од облик x 2  1, x  1 е од облик 4k  1.

Доказ. Нека p x 2  1 , p  3 е прост број. Јасно x 2  1mod p  т.е. x 4  1mod p .

Од p x 2  1 очигледно е дека x не се дели со p , односно НЗД x, p   1.

Од теорема на Ферма имаме x p 1  1mod p . Бидејќи x 4k  1mod p  следува дека


p  1  4k , од каде p  4k  1.

Лема 5.13. Ако a, n се природни броеви и НЗД  a, n   1 тогаш конгруентната равенка


ax  b  mod n  има решение.

Доказ. Од n  1 следува дека   n   1. Бидејќи НЗД  a, n   1, од теорема на Ојлер имаме

a  n  1 mod n   a  nb  b  mod n   ab a  n1  b  mod n  . Значи x  a


  n  1
b е решение на
конгруентната равенка ax  b  mod n  .

Теорема 5.14. (Вилсон) Ако p е прост број тогаш  p  1! 1mod p .

Доказ. За p  2 и p  3 лесно се проверува дека теоремата важи.

Сега, нека претпоставиме дека p  5 . Јасно е дека 1  1(mod p) и p  1  1(mod p) . За


секој j , 2  j  p  2 важи НЗД ( j, p)  1, па постои еден и само еден i  N таков што
ji  1(mod p) и 0  i  p  1 . Очигледно i {0,1, p  1} , па затоа за секој j , 2  j  p  2
постои еден и само еден i таков што ji  1(mod p) и притоа 2  i  p  2 . Имаме i  j ,
бидејќи за j , 2  j  p  2 добиваме НЗД ( j 1, p)  НЗД ( j  1, p)  1 и оттука
НЗД  j 2  1, p   НЗД   j  1 j  1 , p   1.

Од друга страна имаме j 2  1  0(mod p), што не е можно. Значи, броевите 2,3,..., p  2 ги
p 3
поделивме на дисјунктни двоелементни множества {i, j} за кои важи ij  1(mod p) .
2
Ако ги помножиме овие конгуренции добиваме 2  3  4  ...  ( p  2)  1(mod p) .

Последната конгуренција ја множиме со конгуренцијата 1 ( p 1)  1(mod p) и добиваме


( p  1)!  1(mod p) , што требаше да се докаже.

25
200 ТЕОРИЈА НА БРОЕВИ - подготвителни задачи

Лема 5.15. (Туе) Нека m  1 и x се цели броеви, НЗД m, x   1. Докажете дека постојат
такви природни броеви a и b , каде a, b   m  и еден од броевите ax  b или ax  b се
дели со m.

Решение. Ги разгледуваме броевите од облик u ,    x   , каде 0   ,    m .


Нивниот број е  
2
m  1  m , од принцип на Дирихле следува дека постојат два различни
пара  1 , 1  и  2 ,  2  т.ш. u1 , 1   u 2 ,  2 mod m т.е. 1   2 x   2  1 mod m.

Бидејќи НЗД m, x   1 следува дека  1   2 и 1   2 . Нека a   1   2 и b  1   2 .


Добиваме дека ax  b mod m. Според тоа m ax  b или m ax  b.

Теорема 5.16. (Кинеска теорема за остатоци) Ако n1 , n2 , ...., nk се природни броеви,


попарно заемно прости, и a1 , a2 ,...., ak се цели броеви тогаш системот од конгруентни
равенки

 x  a1  mod n1 

 x  a2  mod n2 

 ..............
 x  a  mod n 
 k k

има единствено решение модул n1n2    nk .

n
Доказ. Нека n  n1n2  nk . Бројот е заемно прост со бројот n1 , од Лема 5.13 постои
n1
n n
цел број k1 така што k1   1 mod n1  . Ако ставиме k1   s1 добиваме дека
n1 n1
s1  1 mod n1  и s1  0  mod n j  , j  1. Јасно за секој i 1, 2,..., k  постои цел број si така
што si  1 mod ni  и si  0  mod n j  , j  i. Важи x  a1s1  a2 s2  ...  ak sk  ai  mod ni  т.е
x  a1s1  a2 s2  ...  ak sk е решение на дадениот систем. Ако претпоставиме дека системот
има второ решение x1 тогаш од x  x1  0  mod ni  , за секој i , следува дека
x1  x  mod n1n2  nk  .

26
200 ТЕОРИЈА НА БРОЕВИ - подготвителни задачи

Примери

Пример 5.1. Докажи дека ако a, b и c се непарни природни броеви тогаш a 2  b 2  c 2

не е квадрат на природен број.

Решение. Секој полн квадрат има остаток 0 или 1 при делење со 4 . Бидејќи a, b и c се
непарни природни броеви следува дека остатокот на нивните квадрати при делење со 4 е
1 . Значи, остатокот при делење на a 2  b 2  c 2 со 4 е 1  1  1  3 . Оттука следува дека
a 2  b 2  c 2 не е квадрат на природен број.

Пример 5.2. Ако x  a mod n докажи дека x  a mod 2n или x  a  n mod 2n.

Решение. Од x  a mod n следува дека n x  a т.е. постои цел број k таков што
x  a  kn.

1) Ако k  2s имаме x  a  2ns т.е. 2n x  a. Значи x  a mod 2n.


2) Ако k  2s  1 имаме x  a  2ns  n т.е. 2n x  a  n. Значи x  a  n mod 2n.

Пример 5.3. Ако целите броеви x1 , x2 , ...., x9 не се делат со 3, докажи дека

x12  x22  ....  x92  0 mod 3.

Решение. Бидејќи x i не се дели со 3 следува дека xi  1mod 3 или xi  2 mod 3 .

Според тоа xi2  1mod 3. Оттука x12  x22  ....  x92  9  0 mod 3.

Пример 5.4. Докажи дека ако n е природен број поголем од еден, тогаш n не го дели
бројот 2 n  1.

Решение. Нека важи спротивно n 2 n  1 , и нека p е најмалиот прост делител на n.


Бидејќи 2  1 е непарен број јасно е дека p  3. Важи p 2 n  1 т.е. 2 n  1mod p .
n

Бидејќи НЗД 2, p   1 и од теорема на Ферма имаме дека 2 p 1  1mod p .

Од Својство 5.4. важи 2 НЗД  p 1, n   1mod p . Бидејќи p n следува дека НЗД  p  1, n  1.

Значи 2  1mod p , што не е можно. Оттука следува дека 2n  1 не се дели со n.

27
200 ТЕОРИЈА НА БРОЕВИ - подготвителни задачи

Пример 5.5. Реши ја конгруентната равенка x12  2 x7  x3  1  0  mod 5 .

Решение. Од теорема на Ферма важи x5  x  mod 5 па оттука имаме:

   x 2  x 2  x 2  x 4  mod 5 и x7  x5  x 2  x  x2  x3  mod 5 .
2
x12  x5

Добиваме x12  2 x7  x3  1  x4  x3  1  0  mod5 . Сега со проверка за x  0,1, 2,3, 4  mod 5

во конгруентната равенка x 4  x3  1 0  mod 5 добиваме дека x  3  mod 5 .

Пример 5.6. Докажи дека ако a и b се заемно прости природни броеви, тогаш постојат
цели броеви m и n така што a m  b n  1mod ab.

Решение. Нека m   b и n   a  , каде  е Ојлерова функција.

Од Ојлерова теорема имаме a m  a  b   1mod b. Значи a m  b n  1mod b т.е.

b a m  b n  1. На ист начин добиваме a m  b n  1mod a  односно a a m  b n  1.

Бидејќи a и b се заемно прости природни броеви следува дека ab a  b  1


m n

т.е. a m  b n  1mod ab.

Пример 5.7. Нека p и q се различни непарни прости броеви така што p  1 q  1.

Ако НЗД a, pq   1 докажи дека pq a  1.


q 1

Решение. Од малата теорема на Ферма имаме: a p 1  1mod p  и a q 1  1mod q  .

Бидејќи p  1 q  1 постои природен број k така што q  1  k  p  1.

Имаме a q 1  a   1k  1 mod p  односно p a  1. Јасно p и q се заемно прости,


p 1 k q 1

q 1 q 1 q 1
па од p a  1 и q a  1 следува дека pq a  1.

28
200 ТЕОРИЈА НА БРОЕВИ - подготвителни задачи

Пример 5.8. Реши го системот конгруентни равенки

 x  2  mod 3

 x  3  mod 5 

 x  2  mod 7  .

Решение. Бидејќи НЗД  3,5  НЗД  5,7   НЗД  3,7   1 , од кинеската теорема за
остатоци следува дека дадениот конгруентен систем има единствено решение модул
3  5  7  105. Сега, според доказот на Теорема 5.16 избираме природни броеви k1 , k2 , k3
105 105 105
така што k1  1 mod 3 , k2  1 mod 5 , k3  1 mod 7  , односно треба да важат
3 5 7
конгруенциите 35k1  1 mod 3 , 21k2  1 mod 5 и 15k3  1 mod 7 .

Избираме k1  2, k2  1 и k3  1. Оттука s1  35k1  70, s2  21k2  21 и s3  15k3  15.

На крај добиваме x  a1s1  a2 s2  a3s3  2  70  3  21  2 15  233  23  mod105 .

Значи x  105t  23, каде t е произволен цел број.

Пример 5.9. Докажи дека бројот 225  1 се дели со бројот 5n1.


n

     
Решение. Од теорема на Ојлер важи 2
 5n1
2
  5 5n
2
4 5n
 
 1 mod 5n1 .

  
Значи 5n1 245  1  5n1 225  1  225  1 . Бидејќи НЗД 225  1, 225  1  1 и
n n n

 n n


225  1  1  1  3  mod 5 следува дека 225  1 се дели со бројот 5n1.
n n

29
6. БИНОМНИ КОЕФИЦИЕНТИ

Дефиниција 6.1. За ненегативните цели броеви n и k , k  n , дефинираме биномен


n n n! n
коефициент   со формулата    . Ако k  n тогаш    0.
k   k  k !n  k ! k 

Својство 6.2. Важат следните идентитети:

n n n  n   n   n  1   n  1


1)       1, 2)     , 3)      .
1   n  k   nk   k   k  1  k 

n n!  n  1! n  n. Исто така важи n n!


Доказ. 1) Имаме        1.
 
1  n  1 ! 1!  n  1!  
n n ! 0!

n n! n!  n 
2)      .
 k   n  k ! k !  n   n  k  !  n  k !  n  k 

 n  1   n  1  n  1!   n  1!   n  1!  k  n  k   n!   n  .


3)   
 k  1  k   k  1!  n  k !  n  1  k ! k !  n  k ! k !  n  k ! k !  k 

6.3. Биномна формула (Формула на Њутн). За секој природен број n и реални броеви
n
n
x, y важи формулата  x  y      x n i y i .
n

i 0  i 

Доказ. Формулата ќе ја докажеме со помош на математичка индукција.


1
 1  1   1
Ако n  1 тогаш важи i  x 1i
yi    x   y  x y   x  y  .
1

i 0      
0 1

n 1 n  1
  n1k k
Нека претопставиме дека важи  x  y   
n 1
x y .
k 0  k 

Добиваме:

П . М . И n 1 n  1 n1k k n 1 n  1
  nk k n 1  n  1  n1i i 1
 x  y   x  y  x  y   y   x  y    x y  
n n 1
x x y 
k 0  k  k 0  k  i 0  i 

30
200 ТЕОРИЈА НА БРОЕВИ - подготвителни задачи

n 1 n  1
  n k k n 1  n  1  n i 1 i 1 n 1  n  1 n k k n  n  1 n k k
   x y    i  1  1 x y    x y   x y 
k 0  k  i 0     k 0  k  k 1  k  1

 n  n  1 n  k k  n  1 0 n   n  n  1  n  k k  n  1 n 0 
   x y   x y     x y  x y 
 k 0  k   n    k  0  k  1  1  

n  n 1
   n  1  n  k k n
 n  n k
   x y    x y   x  y  .
n

k  0   k   k  1  k 0  k 

Својство 6.4. Важат следните формули:


2
k n n n 2n 
n n n
1)   1    0, 2)  k   2 , n
3)  k    .
k 0 k  k 0   k 0   n 

Доказ. Ја применуваме Биномната формула за изразот 1  x  .


n

n n 2 n n  n


n
k n
1) За x  1 имаме 0  1  1      1     1    .....   1      1   .
n

0 1  2  n  k 0 k

n n  n
2) За x  1 имаме 2n  1  1        ....    .
n

 0  1   n

3) За секој реален број x важи 1  x   1  x   1  x  . Биномниот коефициент


2n n n

 2n 
1  x 
2n
пред x n во развојот на е еднаков на   . Од друга страна, биномниот
n 
n
коефициенет пред x i во развојот на биномот 1  x  е еднаков на   , додека биномниот
n

i 
 n  n
коефициент пред x n i во развојот на биномот 1  x  е      . Оттука следува дека
n

 n  i i 
биномниот коефициент пред x n во развојот на 1  x   1  x  е еднаков на
n n

2 2 2
 n  n   n  n   n  n   n   n   n
        ....            ...    .
    
0 n 1 n  1        
n 0 0 1  n

31
200 ТЕОРИЈА НА БРОЕВИ - подготвителни задачи

2
n
n 2n 
Значи следува дека важи  k 
k 0  
  .
n 

 n  m   n  n  k 
Теорема 6.5. За 0  k  m  n важи       .
 m  k   k  m  k 

 n  m  n! m! n!
Доказ. Имаме       
 m  k  m! n  m ! k ! m  k ! k ! n  m ! m  k !


n!

 n  k !   n  n  k  .
 n  k !k !  n  m ! m  k !  k  
 m  k 

Теорема 6.5. Нека n  2 е сложен број. Тогаш барем еден член на низата
n n  n 
  ,   ,....,   не се дели со n.
1   2   n  1

 p
Теорема 6.6. Ако p е прост број тогаш    0 mod p  , за 1  i  p  1.
i

Последица 6.7. Ако p е прост број тогаш 1  x   1  x p mod p .


p

 p  p   p  p 1
Доказ. Од Биномната формула имаме 1  x   1    x    x 2  ....    x  x p .
p

   
1 2  p  1

 p
Според Теорема 6.6 добиваме дека    0 mod p  за 1  i  p  1 , па оттука следува дека
i
1  x p  1  x p mod p.

Теорема 6.8. (Лукас) Нека m и n се ненегативни цели броеви, и нека p е прост број.

Нека m  mk mk 1 .....m0 p и n  nk nk 1 .....n0 p . Тогаш важи

 m   mk  mk 1   m1  m0 
            mod p  .
 n   nk  nk 1   n1  n0 

Доказ. Имаме m  mk p k  mk 1 p k 1  ..  m1 p  m0 и n  nk p k  nk 1 p k 1  ...  n1 p  n0 ,

32
200 ТЕОРИЈА НА БРОЕВИ - подготвителни задачи

како и 1  x   1  x   1  x  1  xm    1  x  1  xm


m mk p k  mk 1 p k 1 ...m1 p  m0 mk p k k 1 p
k 1
m1 p 0
.

Од Последица 6.7. важи 1  x   1  x p  mod p  , i  0,1, ..., k.


pi i

Според тоа имаме 1  x   1  x p


m
 k
 1  x 
mk
p k 1
mk 1
   1  x p  1  x 
m1 m0
mod p .
m
Биномниот коефициент пред x n во 1  x m е   . За десната страна добиваме
 n

k 1  mk  mk 1   m1  m0 
   x n1 p x n0 , па коефициентот пред x n е еднаков на       .
k
x n  x nk p x nk 1 p
 nk  nk 1   n1  n0 

 m   m  m   m  m 
Оттука добиваме     k  k 1      1  0  mod p .
 n   nk  nk 1   n1  n0 

Примери

2p
Пример 6.1. Нека p е прост број. Докажи дека    2 mod p .
 p 
2 2 2
2 p  p  p  p
Решение. Ќе го примениме идентитетот          ...    . Од Теорема 6 6
     
p 0 1  p
2
 p
следува дека    0  mod p  , за 1  i  p  1.
i

Добиваме
2 2
2 p  p  p
      0  ...  0     2 mod p .
 p   0  p

Пример 6.2. Нека a и b се ненегативни цели броеви, и p е прост број. Докажи дека

 pa   a 
 pb    b  mod p .
   

33
200 ТЕОРИЈА НА БРОЕВИ - подготвителни задачи

Решение. Од Последица 6.7. имаме 1  x 


pa
 1 x p  mod p.
a
Биномниот коефициент
 pa 
пред x pb во развојот на 1  x  pa е еднаков на   додека биномниот коефициент пред
 pb 
a  pa   a 
a
 
x pb во развојот на 1  x p е   . Оттука добиваме дека важи      mod p .
 
b  pb   b 

Пример 6.3. Предлог ИМО 1988. Природниот број го нарекуваме двоен број ако неговата
декадна репрезентација се состои од блок цифри. На пример, 360360 е двоен број додека
36036 не е. Докажи дека постојат бесконечно многу двојни броеви кои се полни квадрати.

Решение. Од Биномната формула добиваме:

7 7
   1001  1   7 143  1   7 143     7 143     7 143  ...
7 7 7 7 6 5
1021  103
1   2
7
    7 143  1  1 mod 49  .
6

Според тоа бројот 1021n  1 се дели со 49 за секој непарен број n.

    
2
1021n  1  3 1021n  1
9
   
2
Оттука добиваме A  1021n  1  9   102 n  1    N.
49 49  7 
 

9
 
2
Лесно се проверува дека A  1021n  1 е двоен број.
49

 pn 
Пример 6.4. Ако p е прост број тогаш    p n 1 mod p n .  
p 

 pn   p n  1
Решение. Најпрво ќе го докажеме идентитетот    p n 1  .
p   p 1 

n 1  p n  1 n 1 
pn 1 ! 
n 1 p
n pn 1 ! 
p n  1 ! p n  pn    
p   p  n p  n n  n   .
 p 1   
p  p ! p  1! p p  p ! p  1!  
p  p ! p  1! p  p   

 p n  1  
p n  1 p n  2  p n   p  1   
Сега добиваме    k  N.
 p 1  1 2   p  1

34
200 ТЕОРИЈА НА БРОЕВИ - подготвителни задачи

Значи p n
 1 p n  2    p n   p  1   k 1 2   p  1  k   p  1!.

Бидејќи  p 1!  1 mod p  добиваме  1 p1  p 1!  k   p 1!1  k  mod p  .
 pn   p n  1
Според тоа добиваме    p n1 

n 1
  p k  p
n 1
mod p n .  
 
p  p 1 

35
7. КВАДРАТНИ ОСТАТОЦИ
Дефиниција 7.1. Нека m е цел број поголем од 1 , и a е цел број заемно прост со m. Ако
конгруенцијата x 2  a mod m има решение, тогаш велиме дека a е квадратен остаток
на m. Спротивно, велиме дека a е квадратен неостаток на m.

a
Сега, нека p е непарен прост број. Симбол на Лежандер означуваме со   притоа
 p
важи:

a
ако a е квадратен остаток на p тогаш    1,
 p

a
ако a е квадратен неостаток на p тогаш    1.
 p

Теорема 7.2. Нека p е непарен прост број, и a и b се цели броеви заемно прости со p.

Тогаш
p 1
a
а)    a 2 mod p  (Ојлеров криетериум).
 p

 a  b   ab 
б)      .
 p  p   p 

p 1 p 1

Доказ. Ако конгруенцијата x 2  a mod p  има решение тогаш x 2   2 a 2


mod p  .
p 1
Значи x p 1  a 2
mod p . Од теорема на Ферма следува дека x p 1  1mod p  т.е.

p 1

a 2
 1 mod p . Од друга страна бидејќи конгруенцијата x 2  a mod p  има решение
p 1
a a
следува    1 . Значи    a 2  mod p  .
 p  p

Сега нека x 2  a mod p  нема решение. Тогаш за секој i,1  i  p  1 постои единствен
j  i,1  j  p  1 , така што ij  a mod p . Од теорема на Вилсон добиваме

36
200 ТЕОРИЈА НА БРОЕВИ - подготвителни задачи

p 1 p 1
a
a 2
 a  a  ...  a  1  2  ...   p  1   p  1!  1mod p  . Значи    1  a 2  mod p  .
 p
p 1 p 1
 ab  p 1
 a  b 
б) Од Ојлеровиот критериум следува дека     ab  2  a 2 b 2      mod p  .
 p  p  p 

 ab   a  b   ab   a   b 
Оттука важи p       ...(1). Бидејќи   ,   ,   добиваат вредности 1 или
 p   p  p   p   p  p
 ab   a  b 
1 следува дека        2, 0, 2. Бидејќи p е непарен прост број следува дека
 p   p  p 
 ab   a  b   ab   a  b 
(1) важи само ако        0. Значи       .
 p   p  p   p   p  p 

 1  p 1
Лема 7.3. Нека p е непарен прост број. Тогаш важи     1 2 .
 p

 1  p 1
 1  p 1
Доказ. Од Ојлеровиот критериум имаме     1 2  mod p  . Значи p     1 2 .
 p  p

 1  p 1
 1  p 1
 1  p 1
Бидејќи     1 2  2, 0, 2 јасно е дека     1 2  0 т.е.     1 2 .
 p  p  p

Теорема 7.4. Ако p е непарен прост број, тогаш важи

  1  1, p  1 mod 4,


   
 p   1, p  3 mod 4.

Доказ. Секој непарен прост број може да се запише како p  4n  1 или 4n  3.

 1  p 1
Ако p  4n  1 тогаш од Лема 7.3. следува дека     1 2   1  1.
2n

 p

 1  p 1
Ако p  4n  3 тогаш     1 2   1
2 n 1
 1.
 p

2 p 1 2

Теорема 7.5. Ако p е непарен прост број, тогаш     1 8 , т.е.


 p

37
200 ТЕОРИЈА НА БРОЕВИ - подготвителни задачи

 2   1, p  1, 7 mod 8
   
 p    1, p  3, 5 mod 8.

Доказ. Ако p  1 или 5  mod8 , и искористиме дека  p  i   i  mod p  добиваме

p 1 p 1
 p 1   p 1   p 1 
   1 2      2 1   2  2    2  3   2    2  4  6   p  1 
2 2
2 ! 2
 2   2   2 

 p 1   p  3  p 1
 p 1 
 2  4  6        5   3   1   1 4   ! mod p  .
 2   2   2 
p 1 p 1
Следува дека 2 2
  1 4  mod p  . Од Ојлеровиот критериум имаме дека
p 1 p 1
2 2 p 1

   2  mod p  , па оттука добиваме дека    2   1 4  mod p  .


2 2

 p  p

2 8k
2
Ако p  1 mod8 тогаш p  8k  1 , од каде следува     1 4  1 mod p  , т.е.    1.
 
p  p

2 8k  4
Ако p  5  mod8 тогаш p  8k  5 , од каде следува     1 4  1 mod p  ,
 p

2
т.е.    1.
 p

Слично, ако p  3 или 7  mod8 , и искористиме дека  p  i   i  mod p  добиваме

p 1 p 1
 p 1   p 1   p 1 
  !  2 1 2      2 1   2  2    2  3   2    2  4  6   p  1 
2 2
2
 2   2   2 

 p  3   p 1  p 1
 p 1 
 2  4  6        5   3   1   1 4   ! mod p  .
 2   2   2 
p 1 p 1
Следува дека 2 2
  1 4  mod p  . Од Ојлеровиот критериум имаме дека
p 1 p 1
2 2 p 1

   2  mod p  , па оттука добиваме дека    2   1 4  mod p  .


2 2

 p  p

38
200 ТЕОРИЈА НА БРОЕВИ - подготвителни задачи

2 8k  4
Ако p  3  mod8 тогаш p  8k  3 , од каде следува     1 4  1 mod p  ,
 p

2 2 8 k 8
т.е.    1. Ако p  7  mod8 тогаш p  8k  7 , од каде     1 4  1 mod p  ,
 p  p

2
т.е.    1.
 p

Теорема 7.6. (Лема на Гаус) Нека p е непарен прост број и нека a е заемно прост со p.

 p  1
Ги разгледуваме најмалите ненегативни остатоци на броевите a, 2a,3a,...  a при
 2 
a
, тогаш     1 .
p n
делење со p. Ако n е бројот на сите остатоци поголеми од
2  
p

Теорема 7.7. (Правило на квадратни реципрочности) Ако p и q се различни непарни


 p  q  p 1 q 1
прости броеви тогаш      1 2 2 .

 q  p 

Примери
Пример 7.1. Нека p  4k  1 е прост број, k  N . Докажи дека ако a е цел број таков што
конгруенцијата x 2  a  mod p  има решение, тогаш важи x  a k .

p 1
a
Решение. Од Ојлеровиот критериум имаме дека a 2 k 1  a 2
    1 mod p  .
 p

Значи a 2k  a  x 2  mod p  , од каде следува дека x  a k .

Пример 7.2. Докажи дека за секој природен број n секој прост делител на бројот
n4  n2  1 е од облик 12k  1.

Решение. Нека p е прост делител на бројот n4  n2  1. Забележуваме дека важи

    n2  1   1,0  mod 4  и
2 2
n4  n2  1  n2  1  n2 и n4  n2  1  n2  1  3n2 . Јасно е дека

n2  0,1 mod 4 . Според тоа  n2  1  n2  1 mod 4  од каде следува дека p  1 mod 4  .


2

39
200 ТЕОРИЈА НА БРОЕВИ - подготвителни задачи

Важи n4  n2  1   n2  1  3n2  1 mod 3 па оттука p  1 mod 3 . Од p  1 mod 4  и


2

p  1 mod 3 следува дека постојат природни броеви s и t така што p  4s  1  3t  1.

Јасно 4s  3t , од каде следува дека t  4k. Значи p  3t  1  12k  1.

Пример 7.3. Нека p и q се непарни прости броеви за кои p  q  7 pq е полн квадрат.


Докажи дека барем еден од броевите p и q е од облик 4k  1.

Решение. Нека a е природен број за кој важи p  q  7 pq  a 2 . Јасно е дека важи

 p  q
a 2  p  mod q  и a 2  q  mod p  . Значи       1.
 q   p

p 1 q 1
 p  q 
Од правилото на квадратни реципрочности добиваме  1

2 2        1 , од каде
 q   p
p 1 q 1
следува дека барем еден од броевите и е парен. Јасно барем еден од броевите
2 2
p и q е од облик 4k  1.

Пример 7.4. Докажи дека равенката x 2  17 y 2  12 нема целобројни решенија.

Решение. Јасно е дека x 2  12  mod17  . Исто така важи:

31 17 1  2 
 12   3  4   17  2  2
           1 2 2       11  1.

      
17 17 17 3    3
17

Значи конгруентната равенка x 2  12  mod17  нема решение, односно горната равенка


нема целобројно решение.

 205 
Пример 7.5. Пресметај  .
 407 

Решение. Бидејќи 407  37 11 и 205  41 5 од Теорема 7.2. б) добиваме:

 205   205   205   205   41 5   41 5   41   5   41   5 


                 .
 407   37 11   37   11   37   11   37   37   11   11 

Сега, бидејќи 41  4  mod 37  и 41  8  mod11 имаме:

40
200 ТЕОРИЈА НА БРОЕВИ - подготвителни задачи

 205   41   5   41   5   2   5   2   5 
2 3

                   .
 407   37   37   11   11   37   37   11   11 

 a2 
За секој непарен прост број p и цел број a таков што НЗД  a, p   1 важи    1.
 p

 51 37 1
 41   2   5   37 
2
Оттука       1. Од Теорема 7.7. добиваме        1 4  1.
 37   37   37   5 

 5   37   2 
Значи          1, бидејќи x 2  0,1, 4  mod 5 .
 37   5   5 

 51111
 5  11   5   11   1 
Аналогно важи      1 4  1, т.е.          1.
 11  5   11   5   5 

 41   8   2   2   2 
2
Исто така имаме                1. На крај добиваме
 11   11   11   11   11 

 205   41   5   41   5 
               1  1   1 1  1.
 407   37   37   11   11 

41
8. РАВЕНКА НА ПЕЛ

Дефиниција 8.1. Равенката од облик x 2  dy 2  1, каде d е позитивен цел број ( не полн


квадрат) и x и y се цели броеви се нарекува равенка на Пел .

Лема 8.2. (Теорема на Дирихле). Нека  е ирационален број и нека n е природен број.
p 1
Тогаш постои цел број p и број q 1, 2,..., n така што    .
q  n  1 q
1
Доказ. Неравенството е еквивалентно со q  p  . Со  x го означуваме
n 1
децималниот дел на бројот x, т.е.  x  x   x . Ги разгледуваме n  2 броевите
0,   , 2  ,..., n  ,1 од интервалот 0,1. Ако овој интервал го поделиме на n  1
1
интервали со должина тогаш од принцип на Дирихле, бидејќи разгледуваме n  2
n 1
броеви, следува дека постојат два броја s  и t  , кои припаѓат на ист интервал.
1
Значи s   t   и 0  s  n  1, 0  t  n  1. Сега нека s   t   q  p за
n 1
1
некој цел број p и q  s  t  n  1. Оттука јасно е дека важи q  p  s   t   .
n 1

Лема 8.3. Ако  е произволен реален број, тогаш постојат бесконечно многу парови од
p 1
природни броеви  p, q  за кои важи    2.
q q
Доказ. Следува директно од теорема на Дирихле.

Теорема 8.4. Равенката на Пел има решение во множеството природни броеви.


Доказ. Ако во Лема 8.3. ставиме   d добиваме дека постои цел број n така што
n  2 d  1 и равенката x  dy  n има бесконечно многу природни решенија  x, y  .
2 2

Важи  x, y    a, b  mod n  ,  a, b   0,0 ,  0,1 ,....,  n 1, n 1. Оттука јасно е дека
постојат  x1 , y1  и  x2 , y2  различни решенија на x 2  dy 2  n за кои важи x1  x2  mod n  и
y1  y2  mod n  . Нека z1  x1  y1 d , z2  x2  y2 d и притоа z1  z2 .
 x x  dy1 y2 x1 y2  x2 y1 
Ќе докажеме дека  1 2 ,  е нетривијално решение на равенка на Пел.
 n n 

42
200 ТЕОРИЈА НА БРОЕВИ - подготвителни задачи

 x1 x2  dy1 y2   x1 y2  x2 y1  x12 x22  2dx1 x2 y1 y2  d 2 y12 y22  dx12 y22  2dx1 x2 y1 y2  dx22 y12
2 2

  d   
 n   n  n2


 
x12 x22  dy22  dy12 x22  dy22    nx 2
1

2

 ndy12 n x1  dy1
2
n2
 2  1.

2 2 2
n n n n

Теорема 8.5. Ако a, b е најмалото решение во множеството природни броеви на


равенката x  dy  1, тогаш сите решенија на равенката во множеството природни
2 2

броеви се од облик

xn , y n    a  b   a  b d  , a  b d   a  b d 
 n n n n

d .
2 2 d 
 

a  b d   a  b d  a  b d   a  b d 
n n n n

Од xn  и yn  добиваме дека важи


2 2 d

  и x  y d  a  b d  .
n n
xn  yn d  a  b d n n

 d y  x  d y    a  db   1  1.
n
Според тоа следува xn2  dyn2  xn  n n n
2 2 n

Јасно е дека секој пар генериран од горната формула е решение на равенка на Пел.

Забелешка. Ако d е полн квадрат тогаш равенката x 2  dy 2  1 има само тривијално


решение 1, 0  .
Дефиниција 8.6. За равенката x 2  dy 2  k , каде k  1 е цел број, велиме дека е равенка од
Пелов тип. Ваквите равенки може да немат целобројно решение. Пример x 2  3 y 2  2.

Својство 8.7. Нека  x0 , y0  е најмало решение за равенката x 2  dy 2  1, и нека  a0 , b0  е

 .
2
најмало решение за равенката x 2  dy 2  1. Важи x0  y0 d  a0  b0 d

Теорема 8.8. Равенката x 2  dy 2  1 нема решение во множеството цели броеви ако


d  3 mod 4.
Решение. Јасно е дека d има прост делител p од облик 4k  3, бидејќи во спротивно ќе
важи d  1mod 4. Бидејќи p dy следува дека p x  1 . Секој прост делител поголем од
2 2

43
200 ТЕОРИЈА НА БРОЕВИ - подготвителни задачи

три на број од облик x 2  1 има остаток 1 при делење со 4. Последното е противречно со


p  4k  3.

Теорема 8.9. Ако d  1, 2  mod 4  е цел број кој не е полн квадрат, тогаш равенката
x 2  dy 2  1 има решение ако и само ако x0  1 mod 2d  , каде  x0 , y0  е најмалото
решение на равенката x  dy  1.
2 2

Доказ. Нека равенката x 2  dy 2  1 има решение и нека  a0 , b0  е нејзиното најмало


решение. Исто така, нека парот  x0 , y0  е најмало решение на равенката на Пел
x 2  dy 2  1. Од Својство 8.7. добиваме x0  dy0  a02  2a0b0 d  db02 , т.е. x0  a02  db02 .
Значи x0  a02  db02  1  2db02  1 mod 2d  .
Обратно, нека претпоставиме дека x0  1 mod 2d  , каде  x0 , y0  е најмалото решение на
равенката x 2  dy 2  1. Следува дека постои цел број k така што x0  2kd  1. Ако
замениме во равенката добиваме  2kd  1  dy02  1, од каде следува y02  4k  kd  1 .
2

Јасно е дека y0 се дели со 2, т.е. y0  2 y1. Оттука важи y12  k  kd  1 . Бидејќи


НЗД  k , kd  1  1 следува дека k  m2 и kd  1  n2 .
На крај добиваме 1  n2  kd  n2  dm2 , од каде следува дека парот  n, m  е решение на
равенката x 2  dy 2  1.

Примери

Пример 8.1. Најди ги сите решенија - парови од природни броеви x, y  на равенката
x 2  2 y 2  1.
Решение. Со проверка се добива дека 3, 2 е најмало можно решение на равенката
x 2  2 y 2  1. Според тоа за сите решенија на дадената равенка имаме

xn , y n    3  2   3  2 2  , 3  2 2   3  2 2 
 n n n n

2 .
2 2 2 
 

Пример 8.2. Докажи дека постојат бесконечно многу тројки од последователни цели
броеви така што секој од нов е збир од два квадрати.

44
200 ТЕОРИЈА НА БРОЕВИ - подготвителни задачи

Решение. Првата таква тројка се броевите 8  22  22 , 9  32  02 и 10  32  12. Ќе ја


разгледаме тројката последователни броеви x 2  1, x 2 и x 2  1. Бидејќи равенката на Пел
x 2  2 y 2  1 има бесконечно многу природни решенија  x, y  следува дека броевите
x  1  y  y , x  x  0 , x  1  x  1 се бесконечно тројки од последователни цели
2 2 2 2 2 2 2 2 2

броеви за кои важи условот на задачата.

Пример 8.3. Најди ги сите природни броеви n така што важи


 n  n  n 
   2     за некој природен број k  n.
 k  1   k   k  1
n! 2n ! n!
Решение. Имаме   .
 n  k  1! k  1!  n  k !k !  n  k  1! k  1!
Ако последниот израз го помножиме со
 n  k  1! k  1! добиваме:
n!
k  k  1  2  n  k  1 k  1   n  k  n  k  1  n2  3n  2  2k 2  2k.
Добиената равенка е еквивалентна со равенката од Пелов тип  2n  3  2  2k  1  1.
2 2

Најмалото решение во множеството природни броеви на равенката x 2  2 y 2  1 е парот

  
 1 2 i  1 2 i 1 2 i  1 2
     
i

 a, b   1,1 . Оттука добиваме  xi , yi    , .



 2 2 2 
 
Лесно се докажува дека xi и yi се непарни броеви. На крај имаме 2n  3  xi ,
xi  3
т.е. n  .
2

Пример 8.4. Докажи дека ако m  2  2 28n2  1 е цел број за некој природен број n,
тогаш m е полн квадрат.
 m2
2

Решение. Јасно е дека важи  m  2   4 28n2  1 т.е. 


2
  2 

  28n  1. Значи парот
2

 m2 
 , n  е решение на равенката на Пел x 2  28 y 2  1. Лесно се проверува дека 127, 24 
 2 
е најмалото решение на равенката. Според тоа добиваме дека

   127  24     8  3 7   .
2
  8  3 7
k k k k
m  2  127  24 28 28

45
9. НИЗА НА ФИБОНАЧИ

Дефиниција 9.1.

Fn n1

Низата дефинирана на следниот начин: F1  F2  1, Fn 2  Fn1  Fn , за секој
n N , ја нарекуваме низа на Фибоначи.

Теорема 9.2. НЗД Fn1 , Fn   1, за секој n  1.

Решение. Тврдењето ќе го докажеме со помош на математичка индукција.

За n  1 важи НЗД F2 , F1   1. Нека за секој 2  n  k важи НЗД Fn1 , Fn   1.

Ќе докажеме дека НЗД Fk  2 , Fk 1   1. Нека важи спротивно, НЗД Fk  2 , Fk 1   d  2.

Јасно, d Fk  2 и d Fk 1 . Бидејќи Fk  Fk  2  Fk 1 следува дека d Fk . Според тоа


НЗД Fk 1 , Fk   d  1 , што е противречност со НЗД Fk 1 , Fk   1. Значи, останува дека
НЗД Fk  2 , Fk 1   1.

Теорема 9.3. Fmn  Fm1 Fn  Fm Fn1 , за сите m  0 и n  0.

Теорема 9.4. За m  1, n  1, Fm Fmn .

Доказ. Тврдењето ќе го докажеме со помош на математичка индукција по n.

За n  1 јасно е дека Fm Fm . Нека за секој 2  n  k важи Fm Fmk . Сега од Теорема 9.3


имаме Fmk 1  Fmk m  Fmk 1 Fm  Fmk Fm1 . Јасно е дека важи Fm Fmk 1Fm , и од индуктивната
претпоставка Fm Fmk Fm1 следува дека Fm Fmk 1 Fm  Fmk Fm1 т.е. Fm Fmk 1.

Теорема 9.5. Важат следните идентитети:

1) F1  F2  ....  Fn  Fn2  1 .
2) Идентитет на Касини: Fn2  Fn1Fn1   1
n 1
.

n n 1 5 1 5
3) Формула на Бине: Fn  каде   ,  .
5 2 2

Лема 9.6. Ако m  nq  r, m, n  0 тогаш НЗД Fm , Fn   НЗД Fn , Fr .

46
200 ТЕОРИЈА НА БРОЕВИ - подготвителни задачи

Теорема 9.7. Најголемиот заеднички делител на два фибоначиеви броеви е број на


Фибоначи. Важи НЗД Fm , Fn   FНЗД m, n  .

Доказ. Нека m  nq1  r1 . Од Лема 9.6. имаме НЗД Fm , Fn   НЗД Fn , Fr1 .  
   
Нека n  r1q2  r2 . Тогаш важи НЗД Fn , Fr1  НЗД Fr1 , Fr2 . Ако ја повториме ова постапка

ќе добиеме дека постои природен број k така што rk 2  rk 1qk  rk и rk 1  rk qk 1  0.


 
Притоа rk  НЗД m, n. Значи НЗД Fm , Fn   НЗД Frk 1 , Frk . Бидејќи rk rk 1 добиваме дека
Frk Frk 1 . Според тоа добиваме НЗД F , F   НЗД F
m n rk 1 ,F  F
rk rk  FНЗД m, n  .

Теорема 9.8. Fm Fn ако и само ако m n.

Доказ. Нека Fm Fn . Тогаш важи Fm  НЗД Fm , Fn   FНЗД m,n  .

Оттука m  НЗД m, n ,од каде следува дека m n. Ако m n тогаш m  НЗД m, n , од каде
добиваме Fm  FНЗД m, n   НЗД Fm , Fn  . Јасно Fm Fn .

Теорема 9.9. Докажи дека збирот од квадратите на првите n фибоначиеви броеви е


определен со формулата F12  F22  ....  Fn2  Fn Fn1 .

Доказ. Важи F12  F1  F2  1 и Fk2  Fk  Fk  Fk Fk 1  Fk 1   Fk Fk 1  Fk 1 Fk ,

за секој k  2, 3, ..., n. Според тоа ја добиваме формулата:


F12  F22  ....  Fn2  F1 F2  F2 F3  F1 F2   F3 F4  F2 F3   ...  Fn Fn1  Fn1 Fn   Fn Fn1 .

Примери
Пример 9.1. Предлог ИМО 1988. Нека Fn  е низа на Фибоначи. Најди го најголемиот
заеднички делител на 1960 -тиот и 1988 -тиот член од низата на Фибоначи.

Решение. Од Теорема 9.7. имаме НЗД F1960, F1988  FНЗД 1960,1988  F28  256.

Пример 9.2. а) Докажи дека Fn2  Fn21  F2n1.

б) Докажи дека 3 Fn ако и само ако 4 n.

Решение. а) Доказот следува директно од Теорема 9.3. со замена m  n  1.

47
200 ТЕОРИЈА НА БРОЕВИ - подготвителни задачи

б) Нека 3 Fn . Важи Fn4  Fn3  Fn2  2Fn2  Fn1  3Fn1  2Fn . Бидејќи 3 Fn следува дека

важи 3 Fn 4 . Од 3 F4  3 следува дека 3 F8 ,3 F12 ,.. Јасно е дека 4 n.

Сега нека 4 n. Бидејќи 3 F4 и Fn4  Fn3  Fn2  2 Fn2  Fn1  3Fn1  2 Fn добиваме дека
3 F8 ,3 F12 ,.. т.е. 3 Fn .

Пример 9.3. Ако x2  x  1, n  2 тогаш докажи дека xn  Fn x  Fn1.

Решение. Ќе докажеме со помош на математичка индукција по n. За n  2 добиваме

x 2  F2 x  F1  x  1. Сега, нека претпоставиме за n  2 е исполнето x n1  Fn1 x  Fn2 .

Имаме

xn  x  xn1  x   Fn1 x  Fn2   x 2 Fn1  xFn2   x  1 Fn1  xFn2  x  Fn1  Fn2   Fn1 

 xFn  Fn1.

n
n
Пример 9.4. (Цезаро) Докажи дека  k  2 F k
k  F3n .
k 0  

Решение. Од Теорема 9.5. 3) добиваме

 n  k  r  1  r   1  n n k k n n k k 
k
n k
n n k

  
k 0  k 
2 Fk     
k 0  k 
2  
     2 r     2 1  r   
 5  5  k 0  
k k 0  
k 


1
5
 n n

1  2r   1  2 1  r   . .....(1).

1 5 1 5
Од формула на Бине заменивме r  и 1 r  . Бидејќи r 2  r  1 имаме дека
2 2

r 3  r 2  r   r  1  r  r 2  r  r  1  r  2r  1 како и 1  r   1  2 1  r  .
3

Ако замениме во (1) добиваме:

n
   
n
1  2r   1  2 1  r   
1 1 3n
 k  2 F r  1  r   F3n .
n

k n 3n
k
k 0   5 5

48
10. НИЗА НА ФАРЕЈ
a
Дефиниција 10.1. Низа на Фареј Fn е низа од рационални броеви така што
b
0  a  b  n и НЗД a, b  1 .

 0 1  0 1 1  0 1 1 2 1  0 1 1 1 2 3 1
На пример: F1   ,  , F2   , ,  , F3   , , , ,  , F4   , , , , , ,  ,..
 1 1   1 2 1   1 3 2 3 1   1 4 3 2 3 4 1

a c a c
Теорема 10.2. Нека 0    1. Елементите и се соседни елементи во Fn ако и
b d b d
само ако bc  ad  1.

p1 p 2 p
Теорема 10.3. Ако , и 3 се три последователни членови на низа на Фареј, тогаш
q1 q 2 q3
p1  p3 p 2
 .
q1  q3 q2

p1  p3 p2
Доказ. Имаме   q2  p1  p3   p2  q1  q2   q2 p1  q2 p3  p2 q1  p2 q2  0.
q1  q3 q2

p1 p
Бидејќи и 2 се соседни елементи на низа на Фареј, од Теорема 10.2 добиваме дека важи
q1 q2

p2 q1  p1q2  1. Од исти причини важи и p3q2  p2 q3  1.

Добиваме q2 p1  q2 p3  p2 q1  p2q3   p2q1  1  1  p2q3   p2q1  p2q3  0.

Примери
Пример 10.1. Нека a1 , a2 , ...., am се именителите на дропките кои се членови на низа на
1 1 1
Фареј од ред n. Докажи дека   ....   1.
a1a 2 a 2 a3 a m1 a m

0 b b b b 1
Решение. Нека Fn    1 , 2 ,.... m1 , m   . Од Теорема 10.2. имаме дека
 1 a1 a 2 am 1 am 1
bi 1ai  bi ai 1  1, каде i  0,1,..., m  1.

49
200 ТЕОРИЈА НА БРОЕВИ - подготвителни задачи

1 1 1 b a b a b a b a b a b a
Добиваме   ....   2 1 1 2  3 2 2 3  ....  m m1 m1 m 
a1a2 a2 a3 am1am a1a2 a2 a3 am1am

b b  b b  b b  b b 1 0
  2  1    3  2   ...   m  m1   m  1    1.
 a2 a1   a3 a2   am am1  am a1 1 1

p
Пример 10.2. Предлог ИМО 1967. Определи дропка , каде p и q се природни броеви
q
помали од 100 и која е најблиску до 2.

41 99 p 41 p 99
Решение. Важи  2  . Нека е дропка за која важи   .
29 70 q 29 q 70

70 q 29
Тогаш е исполнето   . Бидејќи 99  29  41 70  1 , од Теорема 10.2 следува дека
99 p 41
70 29
дропките и се соседни членови од низа на Фареј од ред n  100. Оттука добиваме
99 41
99
дека p  41  99  100. Со проверка се добива дека дропката е поблиску до 2.
70

50
200 ТЕОРИЈА НА БРОЕВИ - подготвителни задачи

ЗАДАЧИ
1. С.Ф. Дали постои природен број чиј збир на цифри на неговиот четврти степен е 2012
?

2. С.Ф. Во множеството природни броеви реши ја равенката

x6  y 6  z 6  11010100.

3. С.Ф. Во множеството цели броеви реши ја равенката x 5  y 5  z 5  1  6 n

ако важи x  y  z  2012.

4. С.Ф. Предлог ЈММО 2012. Најди ги сите цели броеви a, b, c, d кои не се делат со 5 и
за кои важи a4  b4  c4  d 4  2012.

5. Македонија 1999. Докажи дека равенката x  y    y  z   z  x   30 нема решение


3 3 3

во множеството цели броеви.

6. ММО 1994. Нека a1 , a 2 ,....., a1994 се цели броеви такви што

a1  a2  .....  a1994  19941994.

Да се определи остатокот при делење на a13  a 23  ....  a1994


3
со 6.

7. Балтички натпревар 2012. Најди ги сите тројки од цели броеви a, b, c  за кои е
исполнето a  b  c  20122012.
2 2 2

8. С.Ф. а) Во множеството природни броеви реши ја равенката

ab  10a  5b  1962,

б) Во множеството природни броеви реши ја равенката a 2  1  2b ,

в) Во множеството природни броеви реши ја равенката

1 1 1 4
   .
x y z 5

9. Македонија 2013. Нека a и b се заемно прости броеви.

Докажи дека равенката ax  by  ab нема решение во множеството природни броеви.

51
200 ТЕОРИЈА НА БРОЕВИ - подготвителни задачи

10. ЈММО 2009. Нека m и n се цели броеви за кои важи m  m 2  n 2  8n  2mn  16 .


Докажи дека m е полн квадрат.

11. С.Ф. Бројот 20122011 претстави го како збир од кубови на седум природни броеви.

12. С.Ф. Нека x и y се цели броеви такви што бројот x 2  xy  4y 2 е делив со 25.
Докажи дека x и y се делат со 5.

13. ЈСМО 2009. Најди ги сите двоцифрени броеви ab за кои важи ab a0b .

14. ЈСМО 2009. Дадени се природните броеви а, b, n такa што a 2  2nb 2 е полн квадрат.
Докажи дека бројот a 2  nb 2 може да се претстави како збир од квадрати на два природни
броеви.

15. Докажи дека бројот 147  2 47  347  4 47  547  6 47 се дели со 7.

16. ЈБМО 2004. Нека x и y се природни броеви за кои 3x  4 y и 4 x  3 y се полни


квадрати. Докажи дека x и y се деливи со 7 .

17. ИМО 1964. а) Најди ги сите природни броеви n така што бројот 2 n  1 е делив со 7.

б) Најди ги сите природни броеви n така што бројот 2 n  1 не е делив со 7.

18. С.Ф. Најди ги сите трицифрени броеви чиј збир на цифри е поголем или еднаков од
производот на цифрите на бројот.

nn  1
19. Предлог ИМО 1970. Определи природен број n за кој е трицифрен број со
2
исти цифри.

20. Дали постојат три природни броеви поголеми од еден така што квадратот на секој
минус еден се дели со секој од останатите броеви.
21. PUTNAM 1988. Докажи дека секој сложен природен број може да се претстави во
облик xy  yz  zx  1, каде x, y, z се природни броеви.

1
22. Ако n е совршен број докажи дека  d  2.
d n

23. Докажи дека еден природен број е полн квадрат ако и само ако бројот на неговите
природни делители е непарен број.

52
200 ТЕОРИЈА НА БРОЕВИ - подготвителни задачи

24. Докажи дека секој непарен совршен број има најмалку три различни прости делители.

25. Докажи дека  n  


n
за секој природен број n.
2

 
26. Ако a  b mod n , докажи дека a n  b n mod n 2 . Дали важи обратно?

27. Тајван 1997. Определи ги сите природни броеви k така што постои функција

f : N  Z за која важи:

а) f 1997   1998,

б) За секој a, b N , f ab  f a   f b  kf НЗД a, b.

28. Грција 1968. Ако за рационалниот број x вредноста на изразот 2 x 4  3x  1 е цел


број тогаш и x е цел број. Докажи!

4n  2
29. Романија 2001. Најди ги сите цели броеви n така што бројот е рационален.
n5

30. Белгија 2005. Најди ги сите вредности за ненегативниот цел број n така што
n  n  2005 е природен број.

31. Најдете ги сите природни броеви a, b, c за кои се исполнети условите:

1 a 2  1 и b 2  1 се прости броеви,

   
2 a  1 b  1  c  1 .
2 2 2

32. Романија 2002. Нека n е парен природен број и нека a, b се заемно прости
природни броеви. Најди ги a и b такви што a  b a  b .
n n

5125  1
33. Предлог ИМО 1992. Докажи дека бројот е сложен .
5 25  1

34. Предлог ИМО 1979. Најдете ги сите природни броеви n за кои бројот 28  211  2 n
е полн квадрат.

35. Романија 2008. Нека p е прост број, p  3 , и нека a, b се цели броеви така што
p a  b и p a  b . Докажи дека p 2 a  b или p 3 a 3  b 3 .
2 3 3

53
200 ТЕОРИЈА НА БРОЕВИ - подготвителни задачи

36. ЈБМО 1999. За секој ненегативен цел број n дефинираме An  23n  36n  2  56 n  2 .
Најдете го најголемиот заеднички делител за броевите A0 , A1,...., A1999.

37. Дадени се произволни 2012 природни броеви. Докажи дека можеме да избериме
неколку од нив така што збирот од нивните четврти степени е делив со 2012 .

38. Четири цели броеви се запишани во четири различни точки на една кружница. За еден
број, негов соседен е бројот што е прв до него во насока обратна од насоката на вртење на
стрелките на часовникот. Во еден чекор секој од броевите се заменува со разликата меѓу
него и неговиот соседен број. Дали по 2012 вакви чекори може да се добијат четири
броеви a, b, c, d , така што броевите ab  cd , ac  bd , bc  ad се прости?

39. Грција. Докажи дека равенката 7 x 3  13 y  5 нема целобројни решенија.

40. БМО 1998. Докажи дека равенката x 2  4  y 5 нема решение во множеството цели
броеви.

41. С.Ф. Реши ја равенката 2 n  3n  4 n  5n  4 во множеството природни броеви.

42. ЈСМО 2008. Одреди го најмалиот збир на цифрите на број од облик 3n 2  n  1 , каде
n е природен број.

43. Предлог ИМО 1967. Дали постои цел број чиј куб е еднаков на 3n2  3n  7 , каде n
е цел број?

44. Предлог ЈБМО 2000. Одреди ги сите природни броеви x, y, z  за кои е исполнето
xy  yz  zx  xyz  2.

45. Виетнам 1978. Најди ги сите трицифрени броеви abc така што

2abc  bca  cab.

46. Предлог ЈБМО 2000. Најдете ги сите точни кубови кои не се делат со 10 и притоа
при бришење на последните три цифри пак се добива точен куб.

47. ЈБМО 2000. Нека x, y се позитивни реални броеви за кои е исполнето равенството
x3  y3  x  y   30 xy  2000 . Докажи дека важи x  y  10 .
3

48. Нека m, n се цели броеви за кои важи 0  m  2n . Докажи дека бројот 2 2n2  2 m2  1
е полн квадрат ако и само ако m  n .

54
200 ТЕОРИЈА НА БРОЕВИ - подготвителни задачи

49. Докажи дека за секои цели броеви a, b може да се најде цел број c така што не
постојат цели броеви m, n за кои m 2  am  b  2n 2  2n  c .

50. Романија 2002. Нека a е цел број. Докажи дека за секој реален број x, x 2  3 ,

броевите 3  x2 и 3
a  x 3 не можат истовремено да бидат рационални броеви.

51. а) Докажи дека бројот nn  2n  4n  6 не е квадрат на природен број.

б) С. Ф. Докажи дека производот на шест последователни природни броеви не може да


биде куб на некој природен број.
52. Предлог ИМО 1984. Нека n е природен број и a1 , a2 ,...., a2 n се различни цели

броеви. Најди ги сите цели броеви x за кои важи x  a1   x  a2   ...  x  a2n    1 n! ,
n 2

каде n! 1  2  ...  n .

53. Најди ги сите четирицифрени броеви кои претставени во каноничен облик го


исполнуваат условот: Збирот од нивните прости делители е еднаков со збирот на
степените на простите делители.

54. Предлог ИМО 1984. а) Докажи дека не постои тројка од цели броеви m, n, p  која ја
задоволува равенката 4mn  m  n  p 2

б) Докажи дека постојат бесконечно многу тројки од цели броеви m, n, p  кои ја
задоволуваат равенката 4mn  m  n  p  1 . 2

55. С.Ф. Предлог ЈММО 2013. Нека p1 , p2  5 се прости брoеви за кои важи 6 p1  p2 .

Докажи дека бројот 1p1 p2 1  2 p1 p2 2  3 p1 p2 3  4 p1 p2 4  5 p1 p2 5  6 p1 p2 6 е сложен.

56. Романија 2010. Нека a, b, c и d се природни броеви и нека p  a  b  c  d .

Докажи дека ако p е прост број, тогаш p не е делител на бројот ab  cd .

57. Индија 2012. Нека p1  p2  p3  p4 и q1  q2  q3  q4 се прости броеви, така што


p4  p1  8 и q4  q1  8 . Нека p1  5, q1  5 . Докажи дека 30 p1  q1 .

58. Предлог ИМО 1991. Реши ја равенката 3m  4 n  5 k во множеството природни


броеви.

55
200 ТЕОРИЈА НА БРОЕВИ - подготвителни задачи

59. ИМО 1986. Множеството А  2,5,13 има својство да за секој a, b  A, a  b , бројот


ab  1 е полн квадрат . Докажи дека за секој цел број c  A , множеството A  c го нема
својството на множеството A .

60. Најди ги сите природни броеви n така што n  11  sn , каде sn  го означува збирот
на цифри на бројот n .

61. ЈБМО 1997. Нека n1 , n2 ,...., n2011, n2012 се природни броеви за кои важи
n  n  ...  n
2
1
2
2
2
2011 n 2
2012 . Докажи дека најмалку два од дадените броеви се парни.

1
62. Предлог ИМО 1999. Докажи дека секој позитивен рационален број  r  2 може да
2
a3  b3
се запише во облик , каде a, b, c и d се природни броеви.
c3  d 3

63. Предлог ИМО 1985. Нека a, b, c се позитивни реални броеви. Најди ги сите
позитивни реални броеви x, y, z за кои важи x y  z  abc и
 
4 xyz  a 2 x  b 2 y  c 2 z  abc .

64. Предлог ИМО 1992. Докажи дека за секој природен број m постојат бесконечно
парови природни броеви x, y  за кои се исполенти следните услови:

1 x и y се заемно прости броеви,

2 y x  m ,
2

3 x y  m .
2

65. Канада 2011. Разгледуваме 70  цифрен број со својство секоја од цифрите 1, 2, ..,7

се појавува по точно 10 пати, 8,9,0 не се појавуваат во неговиот декаден запис.

Докажи дека во множеството од сите такви броеви не постојат два така што едниот го
дели другиот.

66. Во множеството природни броеви реши ја равенката 2 x x  y y  3z z .

67. Сингапур 2007. Најди ги сите парови од ненегативни цели броеви x, y  кои ја
задоволуваат равенката 14 y   y x x y
 2007.

56
200 ТЕОРИЈА НА БРОЕВИ - подготвителни задачи

68. Кореа 2012. Најди ги сите тројки m, p, q  така што m е природен број, p, q се
прости броеви и важи 2 p  1  q .
m 2 5

69. ЈАР 2012. Нека p и k се природни броеви така што p е прост број и k  1 .

Докажи дека постои најмногу еден пар x, y  од природни броеви така што важи

x k  px  y k .

70. Кореа 2005. Нека a и b се заемно прости природни броеви. Најди ги сите природни
броеви кои можат да бидат вредност за најголем заеднички делител на броевите a  b и
a 2005  b 2005
.
ab

71. УСАМО 1972.

НЗСa, b, c  НЗД a, b, c 


2 2
Докажи дека  .
НЗСa, b НЗСb, c НЗСc, a  НЗД a, b НЗД b, c НЗД c, a 

72. Холандија 1996. Нека c е ненегативен цел број, и дефинираме a n  n 2  c, n  1.

Нека d n е најголемиот заеднички делител на a n и a n 1 .

а) Нека c  0. Докажи дека d n  1,  n  1.

б) Нека c  1. Докажи дека d n  1, 5,  n  1.

в) Докажи дека d n  4c  1,  n  1.

bc ca ab
73. Германија 2008. Нека a, b и c се природни броеви така што , и
bc ca ab

се природни броеви. Докажи дека НЗД a, b, c   1 .

a a 2  b2
74. Романија 1999. Нека a, b, c се ненулти цели броеви, a  c, така што  .
c c 2  b2
Докажи дека a 2  b2  c2 неможе да биде прост број.

75. Докажи дека низата 1,11,111,... содржи бесконечно поднизи чии членови се попарно
заемно прости.

76. ЈБМО 1998. Докажи дека бројот 1111....111222....22225 запишан со 1997 единици,
1998 двојки е полн квадрат.

57
200 ТЕОРИЈА НА БРОЕВИ - подготвителни задачи

77. Нека p е прост број таков што сите цифри му се единици. Докажи дека бројот на
цифрите на p е прост број.

78. Бразил 1991. Докажи дека постои природен број n  2 така што 1991 199
 991 .
...
n деветки

79. Индонезија 2007. Нека m и n се два природни броеви. Ако постојат бесконечно
многу цели броеви k така што k 2  2kn  m 2 е полн квадрат, тогаш m  n . Докажи!

80. Пан Африка 2012. Најдете ги сите природни броеви m и n така што n m  m

го дели бројот m 2  2m .

81. Нека a, b, c....k се природни броеви такви што a 2b  1, b 2c  1,...., k 2a 1.

Докажи дека a  b  c  ...  k  1 .

82. Балтички натпревар 2011. Определи ги сите парови  p, q  од прости броеви за кои
p 2  q 3 и q 2  p 3 се полни квадрати.

83. С.Ф. Најди ги сите a, b, c, d  N за кои a8  b8  c8  d 8  a 2  b 2  c 2  d 2  5  252013

и збирот на цифри на бројот a 6  b6  c 6  d 6  a 4  b 4  c 4  d 4 изнесува 2014.

84. УМО 1998. Нека a е цел број. Докажи дека не постојат цели броеви b и c, c  1, и за
кои важи a  12  a  22  ...  a  992  b c .

1 1 1
85. Докажи дека збирот S    ...  , каде n  1 , не може да биде цел број.
2 3 n

86. Русија 2001. Нека a и b се различни природни броеви такви што бројот a 2  ab  b 2
е делител на бројот aba  b. Докажи дека a  b  3 ab .

87. Ако НЗД b, c   1, докажи дека НЗД a, bc   НЗД a, b  НЗД a, c .

 2n 
88. Докажи дека n  1  , за секој природен број n.
n 

89. Нека n е природен број. Докажи дека производот на n последователни природни


броеви е делив со n !.

58
200 ТЕОРИЈА НА БРОЕВИ - подготвителни задачи

n
n
90. Чешка 2004. Најди ги сите природни броеви n така што  k ! е цел број.
k 1

91. Предлог ИМО 2005.

Нека a, b, c, d , e и f се природни броеви и нека S  a  b  c  d  e  f . Ако S ги дели


броевите abc  def и ab  bc  ca  de  ef  df тогаш S е сложен број. Докажи!

92. С.Ф. Дали постои природен број n кој има шест природни делители чии збир од
нивните шести степени е 5000054?

93. Јапонија 2009. Најди ги сите природни броеви n така што 8 n  n се дели со

2 n  n.

94. Канада 1999. Определи го бројот на реални решенија на равенката

a a a


 2    3    5   a.

95. Докажи дека  n  n 1  n  2    


9n  8 за n  0,1, 2, ....

96. Канада 1999. Најди ги реалните решенија на равенката 4 x 2  40x  51  0.


 x  2k 
97. ИМО 1968. Нека x е реален број. Докажи дека  k 1 
 x.
k 0  2 

98. Индонезија 2008. Нека m, n  1 се природни броеви за кои важи n 4 m  1 и 2 m n  1 .


Докажи дека n  2m  1.

99. Докажи дека бројот на членови на низа на Фареј од ред n е 1   1   2  ...   n.

100. С.Ф. Нека a и b се природни броеви за кои важи НЗД  а, 2013  НЗД  b, 2013  1

и a  b. Докажи дека бројот a1200  b1200 има најмалку три прости делители.

1 1 1
101. Најди го бројот на парови од природни броеви x, y  за кои важи   ,
x y n

каде n е природен број.

59
200 ТЕОРИЈА НА БРОЕВИ - подготвителни задачи

102. Предлог ИМО 1993. За природниот број n велиме дека го има својство P ако
важи: Ако n го дели a n  1 за некој цел број a, тогаш и n 2 го дели a n  1 .

а) Докажи дека секој прост број има својство P.

б) Докажи дека постојат бесконечно многу сложени броеви n кои го имат


својството P.

103. УСАМО 1998. Нека претпоставиме дека множеството 1, 2, ....,1998 е поделено на
дисјунктни парови ai , bi  1  i  999 , и ai  bi  1 или ai  bi  6 за секој
i  1, 2,....,999. Докажи дека збирот a1  b1  a2  b2  ...  a999  b999 завршува на 9.

104. С.Ф. Избран е сложен природен број за прв член на една конечна геометриска
прогресија со количник фиксиран прост делител на првиот член. Докажи дека низата од
бројот на делители на членовите од геометриската прогресија образува аритметичка
прогресија.

105. Италија 2012. Нека x1 , x 2 , x3 ,.... е низа дефинирана со следната рекурентна формула

 x1  4

 x n 1  x1 x 2 x3    x n  5, n  1

Најди ги сите парови природни броеви a, b  така што x a xb е точен квадрат.

106. Албанија 2011.

Низата a n  е дефинирана со: a1  1 и an  na1  a2  ...  an1 ,  n  1.

а) Докажи дека за секој парен број n , a n се дели со n !.

б) Најди ги сите непарни броеви n за кои a n се дели со n !.

107. Предлог ИМО 1992. Нека  n  е бројот на единици во бинарната репрезентација на


природниот број n. Докажи дека:

  n    n   1
а) Важи неравенството  n2    2
.

б) Горното неравенство преминува во равенство за бесконечно многу природни


броеви n.

60
200 ТЕОРИЈА НА БРОЕВИ - подготвителни задачи

108. ИМО 2005. Ја разгледуваме низата a1 , a 2 , ... дефинирана со a n  2 n  3 n  6 n  1 за


секој природен број n. Определи ги сите природни броеви кои се заемно прости со секој
член од низата.

109. Предлог ИМО 1997. Нека p е прост број и нека f x  е полином од степен d со
целобројни коефициенти така што:

1) f 0  0, f 1  1.

2) за секој природен број n , остатокот при делење на f n  со p е 0 или 1 .

Докажи дека d  p  1.

110. Нека p i е i - тиот прост број. Докажи дека p1k  p 2k  ...  p nk  n k 1 , за секој пар од
природни броеви n, k .

111. ММО 2007. Природните броеви a, b и c се попарно различни за кои важи

a b  c  bc, b c  a  ca, c a  b  ab.

Докажи дека најмалку еден од броевите a, b, c не е прост број.

112. ИМО 2001. Нека a  b  c  d се природни броеви за кои важи

ac  bd  b  d  a  c b  d  a  c .

Докажи дека ab  cd не е прост број.

113. УСАМО 2003. Докажи дека за секој природен број n постои n  цифрен број делив
со 5 n и притоа сите негови цифри се непарни.

114. Докажи дека за секој k  N , постои цел број m така што ниеден од броевите

m  1, m  2, ...., m  k не е слободен квадрат.

115. Докажи дека ако постојат бесконечно многу ненегативни цели броеви n така што
2n  1 и 3n  1 се полни квадрати, тогаш n се дели со 40.

116. Најди ги сите тројки од последователни природни броеви кои се страни на


триаголник чија плоштина е природен број.

61
200 ТЕОРИЈА НА БРОЕВИ - подготвителни задачи

117. За низата an   n2   n  1  докажи дека постојат бесконечно многу природни


2
 
броеви n така што n2   n  1 е полн квадрат, an  an1  1 и an1  an  1.
2

118. Виетнам 1985. Во множеството цели броеви реши ја равенката

x 3  y 3  2 xy  8.

119. Виетнам 2003. Најди го најголемиот природен број n така што равенката

x  12  y12  x  22  y22  ....  x  n2  yn2


има решение  x, y1 , y2 ,..., yn  во множеството цели броеви.

120. Предлог ИМО 2004. Нека k е фиксиран природен број поголем од 1 , и нека
m  4k 2  5. Докажи дека постојат природни броеви a и b така што сите членови на
 xn n1

низата дефинирана со x0  a, x1  b, xn 2  xn1  xn за n  0,1, 2, ... се заемно
прости со m.
n 3
121. Докажи дека n го дели бројот N   r  r ! ако и само ако n е прост број.
r 1

122. Докажи дека секој прост број од облик p  4k  1 може да се претстави како збир од
квадрати на два природни броја.

123. Предлог ИМО 2010. Најдете го најмалиот природен број n за кој постои множество

s1 , s 2 , ...., s n  кое се состои од n различни природни броеви така што важи

 1  1  1 51
1  1      1    .
 s1  s 2   s n  2010

124. Претстави го бројот 459 како збир од квадрати на четири природни броеви.

125. Докажи дека секој цел број може да биде претставен како збир од кубови на пет цели
броеви, не- неопходно различни.

62
200 ТЕОРИЈА НА БРОЕВИ - подготвителни задачи

126. БМО 2001. Нека a, b, n се природни броеви такви што 2 n  1  ab. Нека k  N е
таков што ab  a  b  1 се дели со 2k и ab  a  b  1 не се дели со 2k 1. Докажи дека k
е парен број.

127. ММО 2011. Најди ги сите природни броеви n така што секој природен број запишан
со n  1 единица и една седмица е прост број.

128. Русија 2005. Определи го најмалиот природен број кој неможе да се претстави во
2 a  2b
облик , каде a, b, c, d се природни броеви.
2c  2 d

  n 
 n n   
129. Докажи дека множеството A     ,   ,..., n  1   содржи непарен број на
 
  1  2   2  
 
непарни броеви, ( n е непарен број).

130. ИМО 1967. Нека k, m и n се природни броеви така што m  k  1 е прост број
поголем од n  1. Нека c s  ss  1. Докажи дека бројот cm1  ck cm 2  ck     cm n c k 
се дели со c1c 2    c n .

131. Нека p е прост број. Докажи дека ако постои цел број x за кој p x 2  x  3 тогаш
постои цел број y за кој p y 2  y  25.

132. (ТСТ- Индија). Докажи дека за секој природен број n , n 7  7 не е полн квадрат.
p 1
133. Докажи дека ако p  2 n  1, n  2, е прост број, тогаш 3 2
 1 се дели со p.

134. БМО 2009. Реши ја равенката 3 x  5 y  z 2 во множеството природни броеви.

135. Чешка 2009. Природниот број n го нарекуваме добар ако и само ако постојат точно
четири природни броеви k1 , k 2 , k 3 , k 4 така што n  k i n  k i2 1  i  4 .

Докажи дека

а) 58 е добар број.

б) 2 p е добар ако и само ако p и 2 p  1 се прости броеви  p  2 .

136. Докажи дека равенката a 2  b3  c4 има бесконечно многу решенија во множеството


природни броеви.

63
200 ТЕОРИЈА НА БРОЕВИ - подготвителни задачи

137. ИМО 2002. Нека n  2 е природен број , со делители 1  d1  d2  ...  dk  n.

Докажи дека d1d2  d2 d3  ...  dk 1dk  n2 , и определи кога d1d2  d2 d3  ...  dk 1dk n2 .

138. ММО 2009. Во множеството на цели броеви реши ја равенката

x2010  2006  4 y 2009  4 y 2008  2007 y.

139. Јапонија 2011. Најди ги сите петорки од природни броеви a, n, p, q, r  така што
  
важи a n  1  a p  1 a q  1 a r  1 . 
140. Јапонија 2001.

Најди ги сите природни броеви N така што N  a1  1a2  1......an  1 каде a i се
цифрите на бројот N во неговата декадна репрезентација.
103
141. Сингапур 2008. Најди ги сите прости броеви p, за кои p го дели бројот n
n 1
p 1
.

1
142. Хонг Конг 1999. Најди ги сите позитивни рационални броеви r  1 такви што r r 1 е
рационален број.

143. Украина 2002. Во множеството цели броеви реши ја равенката

n 2002  mm  nm  2n  ..  m  2001n.

144. Албанија 2010. Дадена е низата на Фибоначи дефинирана со a1  a 2  1 и


an 2  an1  an ,  n  1.

а) Докажи дека a 2010 се дели со 10.

б) Дали a1005 се дели со 4 ?

145. БМО 2005. Најди ги сите прости броеви p за кои p 2  p  1 е точен куб на
природен број.

146. Јапонија 1996. Нека m и n се природни броеви такви што НЗД m, n  1.

Пресметај НЗД  5n  7n ,5m  7m  .

64
200 ТЕОРИЈА НА БРОЕВИ - подготвителни задачи

x4 1 y 4 1
147. Виетнам 2007. Нека x  1, y  1 се цели броеви така што  е цел број.
y 1 x 1
Докажи дека бројот x 4 y 44  1 се дели со x  1.

148. ИМО 2000. Определи дали постои природен број n така што n има точно 2000
различни прости делители, и 2 n  1 се дели со n.

149. ИМО 1981. Одреди ја најголемата вредност на изразот m 2  n 2 ако m и n се


природни броеви помали од 1981 за кои важи

n 2  mn  m 2  1.

a n2
150. Предлог ИМО 1994. Нека a0  1994 и a n 1  за секој природен број n.
an  1

Докажи дека 1994  n е најголемиот цел број помал или еднаков на an , 0  n  998.

151. Предлог ИМО 1990.

Нека f 0  f 1  0 и f n  2  4 n 2 f n  1  16 n1 f n  n  2 n ,


2

n  0,1, 2,.. . Докажи дека броевите f 1989, f 1990, f 1991 се делат со 13.

152. Низата a0 , a1 , a2 ,... е дефинирана со: a0  0, a1  1, an2  2an1  an . Докажи дека 2k ak


ако и само ако 2k n.

x y zu
153. Предлог ИМО 2001. Го разгледуваме системот  .
 2 xy  zu

x
Најди ја најголемата вредност за реалната константа m така што m  за секое решение
y
природни броеви x, y, z, u  , така што x  y.

154. Германија 2010. Определи ги сите парови од природни броеви m, n  за кои важи

3m  7 n  2.

155. Хонг Конг 2008. Нека n  4 е сложен број и притоа важи n  n n  1 , каде   n 

65
200 ТЕОРИЈА НА БРОЕВИ - подготвителни задачи

е Ојлерова функција, а  n  е функцијата која го пресметува збирот на сите природни


делители на бројот n . Докажи дека n има најмалку три различни прости делители.

156. Предлог ИМО 2008. Најди ги сите цели броеви a и b кои се решение на равенката

a7 1 5
 b  1.
a 1

157. Предлог ИМО 2000. Определи ги сите природни броеви n  2 за кои важи следниот
услов: За сите цели броеви a, b заемно прости со n,

a  b mod n ако и само ако ab  1mod n.

158. Виетнам 2002. Најди ги сите природни броеви n за кои равенката

x  y  u  v  n xyuv

има решение  x, y, z, u  во множеството цели броеви.

159. Предлог ИМО 2005. Нека a и b се природни броеви така што a n  n го дели
b n  n за секој природен број n. Докажи дека a  b.

160. ИМО 2009. Нека n е природен број и a1 , a 2 , ...., a k ( k  2 ) се различни цели броеви
од множеството 1, 2, ...., n така што n го дели ai ai 1  1 , за секој i  1,...., k  1.

Докажи дека n не го дели бројот a k a1  1.

161. Предлог ИМО 2006. Низата f 1, f 2, f 3,... е дефинирана со формулата

1 n n n


f n      ....    ,
n   1   2  n

каде x  го означува целиот дел од x.

а) Докажи дека f n  1  f n за бесконечно многу природни броеви n,

б) Докажи дека f n  1  f n за бесконечно многу природни броеви n.

162. Хрватска 1999. Во множеството реални броеви реши ја равенката

 x  2x  4x  8x  16x  32x  12345.

66
200 ТЕОРИЈА НА БРОЕВИ - подготвителни задачи

163. Предлог ИМО 1988. Нека a е најголемиот позитивен корен на равенката


   
x 3  3x 2  1  0. Докажи дека a 1788 и a 1988 се деливи со 17.

164. Предлог ИМО 1998. Определи ги сите парови a, b  од реални броеви така што важи

abn  ban за сите природни броеви n.

165. Предлог ИМО 1996. Најди ги сите природни броеви a и b за кои важи

 a 2  b 2   a 2  b 2 
     ab.
 b   a   ab 

166. Предлог ИМО 2009. Нека f е неконстантна функција од множеството природни


броеви во множеството природни броеви, така што a  b го дели f a   f b за сите
различни природни броеви a и b. Докажи дека бројот на сите прости делители на
броевите од множеството  f 1 , f  2  , f  3 ,... е бесконечен.

167. Предлог ИМО 2010.

Најди го најмалиот природен број n така што постојат полиноми f 1 , f 2 , ..., f n со


рационални коефициенти и притоа важи x 2  7  f1 x   f 2 x   ....  f n x  .
2 2 2

168. Предлог ИМО 2004. Нека f : N  N е функција за која важи: За секој m, n  N


бројот m 2  n  се дели со f m  f n. Докажи дека f n  n, за секој природен број n.
2 2

169. Предлог ИМО 2008. Нека n е природен број и p е прост број. Докажи дека ако

a, b, c се цели броеви за кои важи a n  pb  b n  pc  c n  pa , тогаш a  b  c.

170. Предлог ИМО 2011. Го разгледуваме полиномот Px   x  d1 x  d 2     x  d 9 ,


каде d1 , d 2 , ...., d 9 се девет различни цели броеви. Докажи дека постои цел број N таков
што за сите цели броеви x  N бројот Px  се дели со прост број поголем од 20.
2001
171. Канада 2003. Најди ги последните три цифри на бројот 20032002 .

172. Предлог ИМО 1991. Најди го најголемиот природен број k така што 1991k го дели
 19921991 .
1992 1990
бројот 19901991

67
200 ТЕОРИЈА НА БРОЕВИ - подготвителни задачи

173. Предлог ИМО 2011. Нека p е непарен прост број. За секој цел број a, го
a a 2
a p 1
дефинираме бројот S a    ...  . Нека m и n се цели броеви така што
1 2 p 1
m
S 3  S 4  3S 2  . Докажи дека p го дели m.
n

174. Предлог ИМО 2012. Определи ги сите цели броеви m  2 така што секој природен
m m  n 
број n таков што  n  го дели биномниот коефициент  .
3 2  m  2n 

175. Предлог ИМО 1994. Нека M е подмножество од множеството 1, 2, ....,15 така што
производот од три различни елементи од M не е полн квадрат. Определи го
максималниот број на елементи во M .

176. Предлог ИМО 1996. Конечната низа од цели броеви a0 , a1 ,...., a n ја нарекуваме
квадратна ако за секој i 1, 2, ...., n важи ai  ai 1  i .
2

а) Докажи дека за секои два цели броеви b и c , постои природен број n и квадратна
низа таква што a0  b и a n  c.

б) Најди го најмалиот природен број n за кој постои квадратна низа таква што a0  0 и
a n  1996.

177. СМО 2007. Одреди ги сите парови од природни броеви x, n  кои се решение на
равенката x 3  2 x  1  2 n.

178. Ако x е природен број докажи дека предпоследната цифра на секој повеќецифрен
прост делител на бројот 5x 2  1 е парна.

179. Докажи дека за n  5, f n  f n1  1 има најмалку n 1 прости делители.


( Бројот f n  2 2  1 е број на Ферма).
n

180. Предлог ИМО 1992. Дали постои множество од 1992 природни броеви такво што
збирот на елементите на секое негово подмножество е квадрат, куб или повисок степен на
некој природен број.

181. Предлог ИМО 2008. Нека а0 , а1 , а2 ,... е низа од природни броеви така што
најголемиот заеднички делител на два последователни членови е поголем од претходниот
член, односно НЗД  аi , ai 1   ai 1. Докажи дека аn  2n за сите n  0.

68
200 ТЕОРИЈА НА БРОЕВИ - подготвителни задачи

182. Предлог ИМО 2011. Нека f е функција од множеството цели броеви во


множеството на природни броеви таква што за секои два цели броеви m и n важи

f m  n f m  f n. Ако за сите m, n важи f m  f n тогаш докажи дека f m f n.

183. Предлог ИМО 2007. Нека b, n  1 се цели броеви. Претпоставуваме дека за секој
k  1 постои цел број a k така што b  a kn е делив со k . Докажи дека b  A n за некој цел
број A.

184. ИМО 2003. Нека p е прост број. Докажи дека постои прост број q така што за секој
цел број n бројот n p  p не се дели со q.

185. Предлог ИМО 1994. Нека k е природен број. Докажи дека постојат бесконечно
многу полни квадрати од облик n  2 k  7 , каде n е природен број.

186. Виетнам 1997. Докажи дека за секој природен број n постои природен број k така
што 19 k  97 се дели со 2 n .

187. Предлог ИМО 2007. Нека k е природен број. Докажи дека бројот 4k 2  1  
2
има
природен делител од облик 8kn  1 ако и само ако k е парен број.

188. Предлог ИМО 2007. Најди ги сите парови k, n  од природни броеви за кои 7 k  3n
го дели k 4  n 2 .

189. Предлог ИМО 2006. За x  0,1 нека y  0,1 е број чија n -та цифра после
n
децималната запирка е 2 -тата цифра после децималната запирка на бројот x . Докажи
дека ако x е рационален број тогаш и y е рационален број.

190. ИМО 2006. Определи ги сите парови цели броеви x, y  така што

1  2 x  22 x1  y 2 .

191. Канада 2009. Најди ги сите парови од цели броеви a, b  така што 3a  7 b е полн
квадрат.

192. БМО 2013. Определи ги сите природни броеви x, y и z така што x5  4 y  2013z.

193. ИМО 1997. Најди ги сите парови од природни броеви x, y за кои важи x y  y x .
2

69
200 ТЕОРИЈА НА БРОЕВИ - подготвителни задачи

194. Предлог ИМО 2010. Најди ги сите парови m, n  од ненегативни цели броеви за кои

 
m 2  2  3n  m 2 n1  1 .

195. Предлог ИМО 2004. Нека со  n  го означуваме бројот на природни делители на


природниот број n. Докажи дека постојат бесконечно многу природни броеви a така што
равенката  an   n нема решение n.

196. Предлог ИМО 2000. За природниот број n , со d  n  го означуваме бројот на сите


природни делители на бројот n. Најди ги сите природни броеви n така што d  n   4n.
3

197. Предлог ИМО 2005. Најди ги сите природни броеви n  1 за кои постои единствен
цел број a така што 0  a  n ! и a n  1 се дели со n !.

198. Предлог ИМО 1989. Нека m е непарен природен број, m  3. Најди го најмалиот
го дели m n  1.
1989
природен број n таков што 2

199. Предлог ИМО 1998. Определи ги сите природни бреови n за кои постои природен
број m така што 2n  1 е делител на m2  9.

200. Предлог ИМО 2012. За целиот број a велиме дека е пријателски ако равенката
m 2
 
 n n 2  m  am  n има решение во множеството на природни броеви.
3

а) Докажи дека постојат најмалку 500 пријателски броеви во множеството


1, 2, ...., 2012.
б) Провери дали бројот a  2 е пријателски.

70
ЗАДАЧИ И
РЕШЕНИЈА

71
72
200 ТЕОРИЈА НА БРОЕВИ - подготвителни задачи

1. С.Ф. Дали постои природен број чиј збир на цифри на неговиот четврти степен е 2012 ?

Решение. Остатокот при делење на збирот на цифрите на еден број со 3 е ист со


остатокот при делење на тој број со 3 . Бидејќи остатокот при делење на 2012 со 3 е 2
добиваме дека четвртиот степен на таквиот природен број при делење со 3 има остаток
2 . Меѓутоа остатокот при делење на било кој четврти степен на цел број со 3 е 0 или 1 .
Оттука се добива противречност, па следува дека таков природен број не постои.

2. С.Ф. Во множеството природни броеви реши ја равенката

x6  y 6  z 6  11010100.

Решение. За секој природен број a, остатокот при делење на а 6 со 8 е 0 или 1 .

Оттука, остатокот при делењето на x 6  y 6  z 6 со 8 е еден од броевите 0,1,2 или 3.

Но, бидејќи остатокот при делење на 11010100 со 8 е 4, следува дека равенката нема
решение.

3. С.Ф. Во множеството цели броеви реши ја равенката x 5  y 5  z 5  1  6 n

ако важи x  y  z  2012.

Решение. Бидејќи 6 n завршува на цифрата 6 следува дека x5  y 5  z 5 завршува на 5 ,


т.е. 5 x 5  y 5  z 5 . Бидејќи 5 a 5  a , за секој цел број а , следува дека важи
5 x  y  z   x  y  z  . Последното не е можно бидејќи 5 x 5  y 5  z 5 , но 2012 не
5 5 5

се дели со 5 . Оттука следува дека дадената равенка нема решение во множеството на


цели броеви.

4. С.Ф. Предлог ЈММО 2012. Најди ги сите цели броеви a, b, c, d кои не се делат со 5 и
за кои важи a4  b4  c4  d 4  2012.

Решение. Од НЗД a,5  1 и од теорема на Ферма имаме: a 51  1mod 5  a 4  1mod 5 .

Аналогно се добива дека b4  1 mod5 , c4  1 mod5 , d 4  1 mod5 .

Сега, a 4  b4  c4  d 4  4  mod 5 . Но 2012  2mod 5 , па добиваме дека дадената


равенката нема решение во множеството цели броеви.

5. Македонија 1999. Докажи дека равенката x  y 3   y  z 3  z  x3  30 нема решение


во множеството цели броеви.

73
200 ТЕОРИЈА НА БРОЕВИ - подготвителни задачи

Решение. Воведуваме смени m  x  y, n  y  z, p  z  x и ги добиваме равенките


m  n  p  0 , m  n  p  30 . Бидејќи p  m  n имаме:
3 3 3

30  m3  n3   m  n  3m2n  3mn2  3mnm  n  3mnp .


3

m  n  p  0
Според тоа го добиваме следниот систем 
mnp  10 .

Значи m, n, p   1,2,5,10. Со проверка се утврдува дека системот нема решение.


Провери!

6. ММО 1994. Нека a1 , a 2 ,....., a1994 се цели броеви такви што

a1  a2  .....  a1994  19941994.

Да се определи остатокот при делење на a13  a 23  ....  a1994


3
со 6.

Решение. Бидејќи a 3  a  a  1aa  1 за секој цел број a , следува дека 6 a  a ,


3

т.е. важи ai3  ai mod 6.

Според тоа имаме a13  a23  ....  a1994


3
 a1  a2  .....  a1994 19941994  4 mod 6

т.е. бараниот остаток е 4.

7. Балтички натпревар 2012. Најди ги сите тројки од цели броеви a, b, c  за кои е
исполнето a  b  c  20122012.
2 2 2

Решение. Бидејќи 20122012 е делив со 4 имаме дека a 2  b 2  c 2  0 mod 4 . Бидејќи за


секој цел број x важи x 2  0,1mod 4 следува дека a 2  b 2  c 2  0mod 4. Јасно a, b и
c се парни броеви, па постојат цели броеви a1 ,b1 и c1 за кои е исполнето a  2a1 , b  2b1
и c  2c1 .

Оттука имаме a12  b12  c12  5030503. Ако последната равенка ја разгледуваме по модул
осум добиваме дека a12  b12  c12  7 mod 8. Бидејќи за секој цел број x важи
x  0,1,4mod 8 , следува дека остатокот при делење на a  b  c со осум е број од
2 2
1 1
2 2
1

множеството 0,1, 2, 3,4, 5, 6 . Според тоа конгруентната равенка a12  b12  c12  7 mod 8
нема решение.

74
200 ТЕОРИЈА НА БРОЕВИ - подготвителни задачи

8. С.Ф. а) Во множеството природни броеви реши ја равенката

ab  10a  5b  1962,

б) Во множеството природни броеви реши ја равенката a 2  1  2b ,

в) Во множеството природни броеви реши ја равенката

1 1 1 4
   .
x y z 5

Решение. a) Дадената равенка ја трансформираме на следниот начин:

ab  10a  5b  1962  a10  b   5b  50  2012  0 


 ab  10  5b  10  2012  a  5b  10  2012 .

Ги добиваме следните системи:

a  5  1 a  5  2
1  2 
b  10  2012 b  10  1006

a  5  4 a  5  503 a  5  1006 a  5  2012


3  4  5  6 
b  10  503 b  10  4 b  10  2 b  10  1

од каде се добиваат следните решенија:

a, b  6,2022, 7,1016, 9,513, 508,14, 1011,12, 2017,11.

б) Дадената равенка е еквивалентна со а 2  2 b  1 . Ако b  2 тогаш 4 2 , па затоа


b

остатокот при делење на 2  1 со 4 е 3 . Од друга страна остатокот на квадратот на секој


b

природен број при делење со 4 е 0 или 1 . Значи случајот кога b  2 повлекува дека
равенката нема решение. Ако b  1 се добива a  1 , што е единствено решение на
равенката.

в) Без губење на општоста земаме x  y  z и добиваме

4 1 1 1 1 1 1 3
        4 x  15.
5 x y z x x x x

1 4
Според тоа x  1, 2, 3. Ако x  1 равенката нема решение бидејќи 1 .
x 5

75
200 ТЕОРИЈА НА БРОЕВИ - подготвителни задачи

1 1 3 1 1 3
Ако x  2 добиваме   . Следува дека   т.е. 3z  20.
y z 10 z z 10

Значи z 2,...,6. Со проверка се добива дека во овој случај равенката нема решение.

4 1 2
Ако x  3 тогаш важи   т.е. 7 y  30. Значи y  3, 4. Ако замениме x  3 и y  3
5 3 y
15
добиваме z  , што не е природен број. На ист начин, кога x  3, y  4 добиваме дека
2
13
z . Значи дадената равенка нема решение во множеството природни броеви.
60

9. Македонија 2013. Нека a и b се заемно прости броеви.

Докажи дека равенката ax  by  ab нема решение во множеството на природни броеви.

Решение. Важи НЗД  а, b   1. Нека постојат природни броеви x и y така што

ax  by  ab. Јасно a by и бидејќи НЗД  а, b   1 следува дека a y. оттука јасно е дека

y  a. На ист начин се докажува дека b x , од каде x  b.

Добиваме ab  ax  by  ab  ab  2ab, што не е можно. Според ова следува дека равенката


нема решение.

10. ЈММО 2009. Нека m и n се цели броеви за кои важи m  m 2  n 2  8n  2mn  16 .


Докажи дека m е полн квадрат.

Решение. Бидејќи 9m  m  8m  m2  n2  8n  2mn  16  8m   m  n  4 


2
следува дека
9m е полн квадрат, па оттука и m е полн квадрат.

11. С.Ф. Бројот 20122011 претстави го како збир од кубови на седум природни броеви.

Решение. Го разгледуваме следното запишување на 2012 :

2012  1000  1000  8  1  1  1  1  103  103  23  13  13  13  13 .

Според тоа добиваме:

76
200 ТЕОРИЈА НА БРОЕВИ - подготвителни задачи

2012 2011  2012 2010  2012  2012 670   10 3  10 3  2 3  13  13  13  13  


3

 2012 670  10  2012 670  10  2012 670  2  2012 670  
3 3 3 3

 2012 670   2012 670   2012 670  .


3 3 3

12. С.Ф. Нека x и y се цели броеви такви што бројот x 2  xy  4y 2 е делив со 25.
Докажи дека x и y се делат со 5.

Решение. Бидејќи x 2  xy  4y 2 е делив со 25 постои цел број z за кој важи


x 2  xy  4 y 2  25z т.е. x  2 y   55z  xy  ... 1 . Значи 5 x  2 y  , па оттука 5 x  2 y ,
2 2

т.е. x  2 y  5m за некој цел број m . Ако замениме во 1 добиваме 5m 2  5z  xy ,


следува 5 xy , односно еден од броевите x или y е делив со 5 . Но, од 5 x  2 y следува
дека ако едниот број е делив со 5 тогаш и другиот број е делив со 5 .

13. ЈСМО 2009. Најди ги сите двоцифрени броеви ab за кои важи ab a0b .

Решение. Бидејќи ab a0b следува дека 10a  b 100a  b  10a  b 1010a  b  9b ,

па затоа 10a  b 9b . Со непосредно проверување за b  0,1, 2,...,9

се добива дека 10,15,18, 20, 30, 40, 45, 50, 60, 70, 80 и 90 се бараните двоцифрени броеви.

14. ЈСМО 2009. Дадени се природните броеви а, b, n такa што a 2  2nb 2 е полн квадрат.
Докажи дека бројот a 2  nb 2 може да се претстави како збир од квадрати на два природни
броеви.

Решение. Нека постои природен број c така што a 2  2nb 2  c 2 .

Следува дека c 2  a 2  2nb 2 , па заклучуваме дека c  a . Бидејќи c  a c  a  е парен


број добиваме дека c и a имаат иста парност.

c 2  a 2 a 2  c 2 a 2  2ac  c 2  a 2  2ac  c 2
a 2  nb 2  a 2    
2 2 4
ac ac
2 2

    .
 2   2 

ac ac
Бидејќи a и c имаат иста парност следува дека и се природни броеви.
2 2

77
200 ТЕОРИЈА НА БРОЕВИ - подготвителни задачи

15. Докажи дека бројот 147  2 47  347  4 47  547  6 47 се дели со 7.

Решение. Ако n е непарен број, за произволни природни броеви x, y важи


x y x  y .
n n

Според тоа 1  6 147  6 47 , 2  5 2 47  5 47 и 3  4 347  4 47.

Од 7 147  6 47 , 7 2 47  5 47 и 7 347  4 47 следува дека 7 147  2 47  347  4 47  5 47  6 47.

16. ЈБМО 2004. Нека x и y се природни броеви за кои 3x  4 y и 4 x  3 y се полни


квадрати. Докажи дека x и y се деливи со 7 .

Решение. Најпрво ќе го докажеме следното својство: Ако a и b се природни броеви за


кои 7 a 2  b 2 тогаш 7 a и 7 b .

Нека 7 a 2  b2 . Бидејќи за секој цел број x важи x 2  0,1, 2 или 4  mod 7  добиваме дека
мора a 2  0  b2  mod 7  т.е. a  b  0  mod 7  .

Нека 3x  4 y и 4 x  3 y се квадрати на природни броеви. Тогаш постојат природни


броеви a и b за кои 3x  4 y  a 2 и 4 x  3 y  b2 . Бидејќи a 2  b2  7x  y  следува
7 a 2  b 2 , па 7 a и 7 b . Оттука добиваме 49 a 2  b2  7  x  y  т.е. 7 x  y ...(1).
Бидејќи 3x  4 y  a 2 добиваме 3  x  y   7 y  a 2 , и бидејќи 7 a следува 7 3  x  y  т.е.
7 x  y ...(2). Од (1) и (2) следува дека 7  x  y    x  y   2 x т.е. 7 x. Сега јасно е,
бидејќи 7 x  y, важи 7 y.

17. ИМО 1964. а) Најди ги сите природни броеви n така што бројот 2 n  1 е делив со 7.

б) Најди ги сите природни броеви n така што бројот 2 n  1 не е делив со 7.

Решение. Лесно се покажува дека 2 3k  1mod 7, 2 3k 1  2 mod 7 и 2 3k  2  4 mod 7.

а) Бидејќи се барат сите природни броеви n за кои 2 n  1mod 7 следува дека

n  3k , каде k N .

78
200 ТЕОРИЈА НА БРОЕВИ - подготвителни задачи

2, n  3k

б) Јасно е дека важи 2 n  1  3, n  3k  1 па затоа за секој природен број n бројот 2 n  1
5, n  3k  2

не се дели со 7.

18. С.Ф. Најди ги сите трицифрени броеви чиј збир на цифри е поголем или еднаков од
производот на цифрите на бројот.

Решение. Нека abc е таков трицифрен број. Тогаш a  b  c  abc . Бидејќи a, b, c  9


добиваме 0  abc  a  b  c  27 . Јасно е дека сите трицифрени броеви што содржат една
или две нули го исполнуваат условот на задачата. Истото важи ако бројот содржи три или
две единици.

Сега да претпоставиме дека едната цифра на бројот е единица. Нека c  1 . Тогаш важи
a  b  1  ab кое е исполенто за а  2, b  2, c  1 a  2, b  3, c  1 . Јасно е дека и сите
други броеви добиени со преместување на цифрите на 221 и 231 се дел од бараните
броеви. Сега да претпоставиме дека а, b, c  2 . Бројот 222 не ги исполнува условите на
задачата. Ако две од цифрите се двојки , без губење на општоста земаме b  c  2 , и треба
да важи а  4  4а , што е можно само за а  1 .

Ако една од цифрите е два т.е. нека а  2, b, c  3 , тогаш треба b  c  2  2bc  18 , што
не е можно. Ако сите цифри се поголеми или еднакви на три тогаш важи abc  27 .

Од условот a  b  c  abc добиваме дека неравенката нема решение во овој случај.

nn  1
19. Предлог ИМО 1970. Определи природен број n за кој е трицифрен број со
2
исти цифри.

n 2 n  n nn  1
Решение. Јасно е дека важи    999 , па затоа n  2  999  44,7 .
2 2 2
Секој трицифрен број составен од исти цифри е делив со 111 па оттука и со 37 . Значи,
37 n или 37 n  1 . Од n  44,7 следува n  37 или n  36 . Со непосредна проверка се
добива дека n  36, па следува дека бараниот трицифрен број е бројот 666.

20. Дали постојат три природни броеви поголеми од еден така што квадратот на секој
минус еден се дели со секој од останатите броеви.

79
200 ТЕОРИЈА НА БРОЕВИ - подготвителни задачи

Решение. Нека претпоставиме дека a  b  c се природни броеви кои го задоволуваат


условот на задачата. Бидејќи a 2  1 се дели со b , следува дека a и b се заемно прости.
Бидејќи c 2  1 се дели со a и b , и бидејќи a и b се заемно прости следува c 2  1 се
дели со ab . Оттука c 2 1  ab . Но a  c, b  c па следува ab  c 2 , што е спротивно со
c 2 1  ab . Според тоа следува дека не постојат такви природни броеви.

21. PUTNAM 1988. Докажи дека секој сложен природен број може да се претстави во
облик xy  yz  zx  1, каде x, y, z се природни броеви.

Решение. Нека z  1. Добиваме xy  yz  zx  1  xy  y  x  1  x  1 y  1  mn,

каде m  x  1  2, n  y  1  2. Значи xy  yz  zx  1 е сложен број.

Според тоа, секој сложен број s  mn, каде m, n  2 , можеме да го претставиме во облик
xy  yz  zx  1, каде x  m  1, y  n  1, z  1.

1
22. Ако n е совршен број докажи дека  d  2.
d n

n
Решение. Ако d е делител на бројот n , тогаш и е делител на n.
d

Бидејќи n е совршен број за збирот на неговите делители важи  n  2n.

1 1 d 1 2n
Сега имаме,  d  n   n  n   d 
d n d n d n d n n
 2.

23. Докажи дека еден природен број е полн квадрат ако и само ако бројот на неговите
природни делители е непарен број.

Решение. Нека n  p11  p22  pkk . Тогаш d n  1  1 1   2     1   k . Ако бројот на
сите природни делители на бројот n е непарен тогаш d n  1mod 2. Оттука
i  0  mod 2  , i  1, 2,.., k , па заменуваме  i  2 i . Добиваме n  p12   p22     pk2  т.е.
1 2 k


n  p11  p2 2    pk k 
2
е полн квадрат.

80
200 ТЕОРИЈА НА БРОЕВИ - подготвителни задачи

Oбратно, ако n е полн квадрат тогаш i  0  mod 2  , за секој i  1, 2, ..., k. Според тоа
d n  1  1 1   2     1   k   1  01  0    1  0  1mod 2 , т.е. бројот на различни
природни делители на n е непарен број.

24. Докажи дека секој непарен совршен број има најмалку три различни прости делители.

Решение. Нека претпоставиме дека n е непарен совршен број кој има еден прост
делител, т.е. нека n  p k , каде p  3 и k е природен број.

Јасно, броевите 1, p, p 2 ,...., p k се сите делители на n па нивниот збир изнесува

 
  n   1  p  ...  p k 1  p k 
pk  1
p 1
 p k  p k  p k  2 p k  2n.

Бидејќи  n  2n следува дека бројот не е совршен.

Сега, нека n има два прости делители p, q  3 , т.е. n  p a q b , a, b  1.

Добивме

 n    p a q b     p a   q b   1  p  .....  p a 1  q  ...  q b  

 1 1 1   1 1 1  
1  1
 p a 1   2  ...  a   q b 1   2  ...  b   p a q b   s   s 
 p p p   q q q  s 0 p s 0 q

1 1 1 1 15
 paqb    paqb    p a q b  2 p a q b  2n.
1 1 1 1 8
1 1 1 1
p q 3 5

Оттука следува дека секој непарен совршен број има најмалку три различни прости
делители.

25. Докажи дека  n  


n
за секој природен број n.
2

Решение. Нека n  2 p1 p2 ... pk е канонична факторизација на бројот n


0 1 2 k

каде p1 , p2 , ..., pk се прости броеви и  0 ,1 , 2 , ..., k  1.

Имаме

81
200 ТЕОРИЈА НА БРОЕВИ - подготвителни задачи

 1  1  1   1  2 0 p11 p 2 2 ... p k k
 n   n1  1  1  .....1    2  1 p1  1  ... p k  1 
 2  p1  p2   p k  2 p1 p 2 .... p k
 2 0 1 p11 1 p 2 2 1 ... p k k 1   p1  1  ... p k  1.

За pi  3 имаме pi2  3 pi т.е. pi2  1  3 pi   pi  1  pi  pi  1 


2
pi .

 n   2 0 1
p11 1 p 2 2 1 ... p k k 1   p1  1  ... p k  1  2 0 1 p11 1 p 2 2 1 ... p k k 1 p1 p2    pk 
0 1 1 1
2 1  2  k 
 p 1
2
p 2
2
 p k
2
.
2

1 i
Ако искористеме дека  i   за секој природен број  i , добиваме дека важи:
2 2

2 0 p11 p2 2    pk k


 n  
n
 .
2 2

Ако  0  0 тогаш  n   n 
n
.
2

26. Ако a  b mod n , докажи дека a n  b n mod n 2 . Дали важи обратно?  


Решение. Бидејќи a  b mod n следува дека постои природен број c така што a  nc  b.

Од Биномната формула добиваме

n n   n
a n  b n  b  nc   b n  b n   b n1nc   b n2 n 2 c 2  ...   n n c n  b n 
n

 
1  
2  n
  n  n 
 n 2  b n1c   b n2 c 2  ....   c n n n2 .
  
2  
n 

Значи, n 2 a n  b n  т.е. a n  b n mod n.

Обратно не важи бидејќи, на пример, 34  14 mod 4 2   но 3 и 1 немат ист остаток при


делење со 4.

27. Тајван 1997. Определи ги сите природни броеви k така што постои функција

f : N  Z за која важи:

а) f 1997   1998,

82
200 ТЕОРИЈА НА БРОЕВИ - подготвителни задачи

б) За секој a, b N , f ab  f a   f b  kf НЗД a, b.

 
Решение. Ако ставиме a  b добиваме f a 2  k  2 f a .

Оттука важи

 
f a4  f a2    k  2 f a   k  2k  2 f a  k  2
2 2 2
f a .

Од друга страна добиваме:

       
f a 4  f a 3  a  f a 3  f a   kf a   f a 2  f a   kf a   f a   kf a  
 k  2 f a   2 f a   2kf a   3k  4 f a .

Значи k  22 f a   3k  4 f a . Ако ставиме a  1997 јасно f a   1998  0 па следува

k 2  4k  4  3k  4, од каде следува k  0.

28. Грција 1968. Ако за рационалниот број x вредноста на изразот 2 x 4  3x  1 е цел


број тогаш и x е цел број. Докажи!

m
Решение. Бидејќи x е рационален број имаме x , каде m , n Z , n  0,
n
НЗД m, n  1 и нека 2 x 4  3x  1  k , k  Z .

m
Ако во последното равенство замениме x  добиваме
n
4
m m
2   3   1  k т.е. 2m 4  3mn3  n 4  kn4 .
 
n n

Значи, 2m 4  kn4  3mn3  n 4  n 3 kn  3m  n. Јасно, kn  3m  n е цел број, па затоа

n 3 n 3 kn  3m  n т.е. n3 2m 4 .

Бидејќи НЗД m, n  1 следува дека n 3 2 , од каде добиваме дека n  1.

m
Значи x    m Z .
n

83
200 ТЕОРИЈА НА БРОЕВИ - подготвителни задачи

4n  2
29. Романија 2001. Најди ги сите цели броеви n така што бројот е рационален.
n5

Решение. Нека a и b се заемно прости природни броеви така што

4n  2 a 2 2b 2  5a 2 22b 2
 т.е. n   5  2 .
n  5 b2 4b  a
2 2
4b  a 2

   
Бидејќи НЗД b2 , 4b2  a 2  НЗД b2 , a 2  1 следува дека 4b 2  a 2 22 . Ако a е парен
број тогаш 4b 2  a 2 е делив со 4 , што не е можно. Значи a е непарен број, па оттука
остатокот при делење на 4b 2  a 2 со 4 е 3 . Затоа 4b 2  a 2  1 или 4b 2  a 2  11 . Во
првиот случај се добива b  0 што не е можно. Во вториот случај имаме
2b  a  1, 2b  a  11 од каде се добива a  5, b  3 и n  13 .

30. Белгија 2005. Најди ги сите вредности за ненегативниот цел број n така што
n  n  2005 е природен број.

Решение. Ќе ја користиме следната лема:

Ако s, t се природни броеви така што s  t  N тогаш s и t се полни квадрати.

Доказ. Нека s, t  N , s  t  a  N и нека t не е полн квадрат. Тогаш t е ирационален

 s   a  t 
2 2
број. Добиваме s   a 2  t  2a t т.е. 2a t  a 2  t  s.

a2  t  s
Бидејќи a  0 добиваме t т.е. t е рационален број, што е спротивно со
2a
претпоставката дека t е ирационален број.

Значи, n  a 2 и n  2005  b 2 каде a и b се природни броеви. Со одземање на двете


равенки се добива b  a b  a   2005 . Од 2005  1  2005  5  401 ги добиваме следните
b  a  1 b  a  5 b  a  401 b  a  2005
системи од линеарни равенки     .
b  a  2005 b  a  401 b  a  5 b  a  1

Со нивно решавање се добиваат следните решенија:

a, b  1002,1003, 198,203, 203,198, 1003,1002 .

84
200 ТЕОРИЈА НА БРОЕВИ - подготвителни задачи

31. Најдете ги сите природни броеви a, b, c за кои се исполнети условите:

1 a 2  1 и b 2  1 се прости броеви,

   
2 a  1 b  1  c  1 .
2 2 2

Решение. Без губење на општоста земаме дека a  b .

   
Тогаш c  1  a  1 b  1  a  1 a  1  a  1 .
2 2 2 2 2 2
   
2

Оттука c 2  c 2  1   a 2  1 т.е. c  a 2  1. Јасно е дека важи a 2  1 c 2  1, a 2  1 a 2  1 па


2

важи и a  1
2
c  1   a  1  a
2 2 2
 1  c  a  c  a  .

 
Забележуваме дека 0  c  a  c  a  a 2  1  a  2 a 2  1 . Бидејќи a 2  1 е прост број
следува c  a  a  1, c  a  1 . Со одземање на последните две равенки се добива
2

a  2, b  1, c  3 .

32. Романија 2002. Нека n е парен природен број и нека a, b се заемно прости
природни броеви. Најди ги a и b такви што a  b a  b .
n n

  
Решение. Имаме a n  b n  a 2  b 2 a n2  a n4 b 2  ...  b n2 бидејќи n е парен број.

Јасно е дека a  b a  b , па затоа следува a  b a  b .


2 2 n n

Според тоа добиваме


a  b a n  b n  a n  b n  2a n и a  b a n  b n  a n  b n  2b n .
 
Бидејќи НЗД 2а n ,2b n  2 добиваме дека a  b 2 , па затоа a  b  1 .

5125  1
33. Предлог ИМО 1992. Докажи дека бројот е сложен .
5 25  1


Решение. Нека a  525. Од a 5  1  a  1 a 4  a 3  a 2  a  1 добиваме 
5125  1 a5  1
 
 a 4  a 3  a 2  a  1  a 2  3a  1  5a  a  1 
2 2

5 1 a 1
25

 
 a 2  3a  1  513  a  1 a 2  3a  1  513  a  1 . 

85
200 ТЕОРИЈА НА БРОЕВИ - подготвителни задачи

5125  1
Бидејќи a 2  3a  1  513  a  1  a 2  3a  1  513  a  1  1 следува дека бројот
5 25  1
можеме да го претставиме како производ на два различни природни броеви различни од
еден. Според тоа дадениот број е сложен.

34. Предлог ИМО 1979. Најдете ги сите природни броеви n за кои бројот 28  211  2 n
е полн квадрат.

Решение 1. Нека 2n  28  211  x2 т.е. 2n   3  24   x 2 . Оттука јасно е дека важи


2

x  3  24  2a и x  3  24  2b , и притоа a  b и a  b  n.

Ако ги одземиме добиените две равенки имаме 2b  2a  6  24  3  25 , т.е.

2a  2ba  1  3  25. Сега јасно е дека a  5 и 2b5  1  3, од каде следува b  7.

Значи n  a  b  12.

Решение 2. Со проверка лесно се утврдува дека за n  8 , 28  211  2 n не е полн квадрат.


Нека претпоставиме дека 8 11 n 8

n  8 . Добиваме, 2  2  2  2 9  2
n 8

. Оттука 9  2 n8
мора да биде полн квадрат. Претпоставуваме дека постои природен број x така што
92 n 8
 x , односно 2
2 n 8
 x  9  x  3x  3 . Од последното заклучуваме дека x  3
2

и x  3 се броеви од облик 2 k .

Земаме природни броеви a, b такви што x  3  2 a , x  3  2 b , каде a  b  n  8 .

Со одземање на последните две равенки добиваме 2  3  6  x  3  x  3  2 a 2ba  1 .  


Очигледно е дека a  1, b  3 е решение на последната равенка, од каде n  12.

35. Романија 2008. Нека p е прост број, p  3 , и нека a, b се цели броеви така што
p a  b и p 2 a 3  b 3 . Докажи дека p 2 a  b или p 3 a 3  b 3 .

Решение. Нека претпоставиме дека a  b не се дели со p 2 . Сега доволно е да докажеме


дека p 3 a 3  b 3 .

86
200 ТЕОРИЈА НА БРОЕВИ - подготвителни задачи

Бидејќи p 2 a  b  3aba  b од претпоставката, следува дека p 3ab . Бидејќи p  3


3

следува дека p a или p b . Од условот p a  b следува дека p a и p b , па оттука


p 3 a 3 и p 3 b 3 , што заедно повлекува дека p 3 a 3  b 3 .

36. ЈБМО 1999. За секој ненегативен цел број n дефинираме An  23n  36n  2  56 n  2 .
Најдете го најголемиот заеднички делител за броевите A0 , A1,...., A1999.

Решение. Лесно се проверува дека A0  1  9  25  35 и A1 не се дели со 5 . Затоа


најголемиот заеднички делител на броевите A0 , A1,...., A1999 е 7 или 1 . Бидејќи остатокот
при делење на 2 3 ,36 и 56 со 7 е 1 следува дека остатокот при делење на
6n  2 6n  2
An  2  3
3n
5 со 7 е ист со остатокот на бројот 1  32  52  35 при делење со 7,
а тој е 0. Значи сите броеви A1 , A2 ,...., A1999 се делат со 7. Според тоа
НЗД  А0 , А1 ,...., А1999   7 .

37. Дадени се произволни 2012 природни броеви. Докажи дека можеме да избериме
неколку од нив така што збирот од нивните четврти степени е делив со 2012 .

Решение. За S1  a14 , S 2  a14  a24 , .... , S 2012  a14  a24  ...  a2012
4
постојат две можности.

1 Барем еден од S1 , S 2 ,..., S 2012 е делив со 2012.

2  Ниеден од броевите S1 , S 2 ,..., S 2012 не е делив со 2012.

Во случај 1 веднаш се добива дека ако S i е делив со 2012 тогаш a1 , a2 ,...., ai се


бараните броеви кои го задоволуваат условот на задачата.

Во случај 2  остатокот на секој S i при делење со 2012 е број од множеството


1,2,...,2011 . Бидејќи S1 , S 2 ,..., S 2012 се 2012 броеви, а постојат 2011 можни остатоци при
нивното делење со 2012 , од принцип на Дирихле следува дека барем два броја при делење
со 2012 имаат исти остаток. Нека претпоставиме дека тоа се броевите S i и S j , и нека
i  j . Тогаш 2012 S i  S j , односно 2012 a 4j 1  a 4j  2  ...  ai4 .

38. Четири цели броеви се запишани во четири различни точки на една кружница. За еден
број, негов соседен е бројот што е прв до него во насока обратна од насоката на вртење на
стрелките на часовникот. Во еден чекор секој од броевите се заменува со разликата меѓу

87
200 ТЕОРИЈА НА БРОЕВИ - подготвителни задачи

него и неговиот соседен број. Дали по 2012 вакви чекори може да се добијат четири
броеви a, b, c, d , така што броевите ab  cd , ac  bd , bc  ad да се прости?

Решение. Нека m, n, p, q се четири цели броеви запишани во четири различни точки од


кружницата, во насока на вртење на стрелките на часовникот. По првиот чекор добиените
броевите се m  q, n  m, p  n, q  p .

По вториот чекор ги добиваме броевите m  2q  p, n  2m  q, p  2n  m, q  2 p  n ,

по третиот m  3q  3 p  n, n  3m  3q  p, p  3n  3m  q, q  3 p  3n  m и во четвртиот
чекор ги добиваме броевите
2m  4q  6 p  4n, 2n  4m  6q  4 p, 2 p  4n  6m  4q,
2q  4 p  6n  4m.

Забележуваме дека добиените четири броја се парни. Оттука, бидејќи 2012 се дели со 4 ,
следува дека по 2012 вакви постапки ќе добиеме парни броеви. Бидејќи a, b, c, d се
парни следува дека ab  cd , ac  bd , bc  ad не можат да бидат прости броеви.

39. Грција. Докажи дека равенката 7 x 3  13 y  5 нема целобројни решенија.

Решение. Нека претпоставиме дека x 0 и y 0 се цели броеви за кои важи 7 x03  13 y0  5.

Јасно е дека 7 x03  5 mod 13 , 14 x03  10 mod 13 т.е. x03  3 mod 13. По стапенување на
четврти степен последната конгруенција добива облик x012  81mod 13. Јасно е дека x 0 не
се дели со 13, па затоа НЗД x0 ,13  1. Од малата теорема на Ферма следува дека
x012  1mod 13. Значи 80  0 mod 13 , што не е точно. Затоа горната равенка нема
целобројни решенија.

40. БМО 1998. Докажи дека равенката x 2  4  y 5 нема решение во множеството цели
броеви.

Решение. Од малата теорема на Ферма имаме y 10  1 mod 11 , кога y не се дели со 11

или y10  0  mod11 кога y се дели со 11. Значи y 10  0,1mod 11 , па според тоа
y 5  1, 0,1mod 11.

88
200 ТЕОРИЈА НА БРОЕВИ - подготвителни задачи

Важи x 2  0,1, 3, 4, 5, 9 mod 11, односно x2  4  2, 4,5,7,8,9  mod11 . Очигледно е дека


x 2  4 и y 5 имат различни остатоци при делење со 11 , па според тоа дадената равенка
нема решение во множеството цели броеви.

41. С.Ф. Реши ја равенката 2 n  3n  4 n  5n  4 во множеството природни броеви.

Решение. Јасно е дека n  1, 2 се решение на равенката. Нека претпоставиме дека n  2 .


Дадената равенка е еквивалентна со равенката:


2 n  3n  4  5n  4 n  5  4 5n1  5n2  4  5n3  4 2  ....  4 n1 , 
т.е. 3n  5n1  5n2  4  ...  5  4 n2  2 2n1  2  2 n1  1  3 .

Бројот 5n1  5n2  4  ..  5  4 n2 завршува на цифрата 5 и ќе го замениме со m .


n 1
 2
Добиваме 3  m  2  1  3 . Бројот 3 n завршува на цифрата 1, 3, 7 или 9 додека
n

m  2  1  3 завршува на цифрата
n 1 2
бројот 3, 7 или 9 . Ако 3 n завршува на 3 тогаш

n  4k  1. Сега
n 1

2 n1  1  2 4k  1 завршува на 5 па затоа m  2  1  1 завршува на 2

цифрата 1 , што не е можно. Ако 3 n завршува на цифрата 7 тогаш n  4k  3 па оттука


n 1
 2
2 n1  1  2 4k 2  1 завршува на цифрата 3 , т.е. m  2  1  1 завршува на цифрата 5 ,
што исто така не е можно. Значи равенката нема решение за n  2 .

42. ЈСМО 2008. Одреди го најмалиот збир на цифрите на број од облик 3n 2  n  1 , каде
n е природен број.

Решение. За n  1 бројот е 5 . За n  8 го добиваме бројот 201 , чиј збир на цифри е 3 .


Ќе докажеме дека ова е најмалиот можен збир на цифри.

Јасно 3n 2  n  1  2n 2  n 2  n  1  2n 2  nn  1  1 е непарен број, па збирот на цифри на


ваквите броеви не може да биде 1 , бидејќи такви броеви се од облик 10 m кои се парни.
Од непарноста на 3n 2  n  1 следува дека тој број не е од облик 2  10 m . Останува да
провереме дали бројот 3n 2  n  1 е од облик 10 m  1 , односно дали равенката
3n 2  n  1  10 m  1 има решенија во множеството природни броеви.

Бидејќи 3n 2  n  n3n  1  10 m  2 m 5 m најпрво земаме n  2 m . Тогаш 3  2 m  1  5 m .

89
200 ТЕОРИЈА НА БРОЕВИ - подготвителни задачи

Но, бидејќи 5 m  3  2 m  1, m  2 добиваме дека равенката нема решение. Сега, ако


n  5 m , тогаш 3  5 m  1  2 m што очигледно не е можно. Ако земеме n  1, 3n  1  10
m

добиваме 4  10 m , што не е можно. Според тоа равенката нема решение, па затоа


најмалиот збир на цифри на број од облик 3n 2  n  1 е 3 .

43. Предлог ИМО 1967. Дали постои цел број чиј куб е еднаков на 3n2  3n  7 , каде n
е цел број?

Решение. Нека претпоставиме дека постојат цели броеви n, k за кои


 
k 3  3n 2  3n  7  3 n 2  n  2  1. Јасно, остатокот при делење на k 3 со 3 е 1 , па затоа
постои цел број s за кој k  3s  1. Ако замениме во k 3  3n2  3n  7 имаме
 
3s 3s 2  3s  1  n2  n  2 ... 1 . Бидејќи n2  n  2 не е делив со 3 следува дека 1 не е
можно, па според тоа не постои цел број чиј куб е еднаков на 3n 2  3n  7.

44. Предлог ЈБМО 2000. Одреди ги сите природни броеви x, y, z  за кои е исполнето
xy  yz  zx  xyz  2.

Решение. Без губење на општоста претпоставуваме дека x  y  z. Следува дека


xy  yz  zx  3xy, односно 2  xyz  3xy. Од последната неравенка добиваме xy 3  z   2
па следува z  3, т.е. z  1 или z  2 . Ако z  1 имаме x  y  2, па следува дека
x  y  1. Ако z  2 тогаш xy  2 y  2 x  2 xy  2 т.е. x  2 y  2  2 . Решението на
последната равенка е x  4, y  3. Значи 1,1,1 ,  4,3, 2 ,  4, 2,3 , 3, 4, 2  , 3, 2, 4  , 2,3, 4  и
 2, 4,3 се решенија на дадената равенката.

45. Виетнам 1978. Најди ги сите трицифрени броеви abc така што

2abc  bca  cab.

Решение. Имаме 2100a  10b  c   100b  10c  a   100c  10a  b,

од каде следува 7a  3b  4c. Ако две од цифрите a, b, c се еднакви следува дека и третата
цифра е еднаква на останатите две. Нека претпоставиме дека a  b  c  a.

a b 4
Имаме 7a  3b  4c  3a  3b  4c  4a  3a  b  4c  a    .
ca 3

90
200 ТЕОРИЈА НА БРОЕВИ - подготвителни задачи

Нека a  b  4k , c  a  3k. Со собирање имаме c  b  7k. Бидејќи a, c се цифри важи


 9  c  b  9, па следува k  1. Ако k  1 добиваме a  b  4, c  a  3,

па b  a  4  0, a  4 и a  c  3  6 . Значи a  4, 5, 6 и од a  b  4, c  a  3, следува
b  0,1, 2 и c  7, 8, 9 , соодветно. Добиваме дека 407, 518, 629 се три од бараните броеви.

На ист начин, за k  1 ги добиваме броевите 370, 481, 592.

46. Предлог ЈБМО 2000. Најдете ги сите точни кубови кои не се делат со 10 и притоа
при бришење на последните три цифри пак се добива точен куб.

Решение. Нека а 3 е таков број и b 3 е бројот што се добива по отстранувањето на


последните три цифри на а . Постои природен број c  1,2,...,999 така што
3

a  1000b  c  10b   c, па оттука важи c  a3  10b  10b  1  10b .


3 3 3 3 3 3

Со мало средување добиваме:

1000  c  1000b3  300b2  30b  1  1000b3  300b 2  30b  1 


 300b2  30b  999  0  b  2.

Ако b  0 добиваме дека важи a3  c , од каде a3 1,8, 27,64,125, 216,343,512,729.

Ако b  1 имаме a 3  1000  c , од каде a3 1331,1728 .

47. ЈБМО 2000. Нека x, y се позитивни реални броеви за кои е исполнето равенството
x3  y3  x  y   30 xy  2000 . Докажи дека важи x  y  10 .
3

Решение. Воведуваме смени x  y  a, xy  b . Добиваме:

 
x 3  y 3  x  y   30 xy  x  y  x 2  xy  y 2  x  y   30 xy 
3 3

 2
 3
 
 x  y  x  y   3xy  x  y   30 xy  a a 2  3b  a 3  30b 
a  3ab  a  30b  2a  3ab  30b.
3 3 3

Оттука добиваме 2a3  3ab  30b  2000  2  103 или запишано во еквивалентна форма

3b10  a   210  a  100  10a  a 2 . 
Ако 10  a  0 следува дека важи 200  20a  2a 2  3b , што не е можно, бидејќи
a 2  x  y   x 2  2 xy  y 2  4 xy  4b . Останува 10  a  0 т.е. x  y  10.
2

91
200 ТЕОРИЈА НА БРОЕВИ - подготвителни задачи

48. Нека m, n се цели броеви за кои важи 0  m  2n . Докажи дека бројот 2 2n2  2 m2  1
е полн квадрат ако и само ако m  n .

Решение. Јасно 2 2 n2  2 n1   2


е полн квадрат. Ако 2 2n2  2 m2  1 е полн квадрат тогаш

важи 
2 2n2  2 m2  1  2 n1  1 
2
од каде следува дека m  n . Очигледно е дека
2 n2 m 2
2 2  1 е непарен број па затоа земаме

2 2 n 2  2 m 2  1  2k  1  4k 2  4k  1  2 2 n  2 m  k k  1 
2

 2 m 2 2 nm  1  k k  1.

 
Очигледно 2 m 2 2 nm  1 е производ на два последователни броеви. Тоа е можно само кога
2 nm
2 2 m
односно n  m . Ако постои друго такво претставување тогаш ќе имаме дека
 
броевите 2 и 2 mk 2 2nm  1 се последователни, што не е можно заради нивната иста
k

парност.

49. Докажи дека за секои цели броеви a, b може да се најде цел број c така што не
постојат цели броеви m, n за кои m 2  am  b  2n 2  2n  c .

Решение. Нека a е непарен број. Забележуваме дека парноста на бројот


m  am  b  mm  a   b е иста со парноста на b . Од друга страна парноста на бројот
2

2n 2  2n  c  2nn  1  c е иста со парноста на бројот c . Ако c го избериме да има


спротивна парност од b тогаш m 2  am  b и 2n 2  2n  c имат различна парност, па
равенката m 2  am  b  2n 2  2n  c нема решение. Сега нека претпоставиме дека a е
парен број. Бидејќи 2n 2  2n  c  2nn  1  c и nn  1 се дели со 2 следува дека
остатокот при делење на 2n 2  2n  c со 4 е ист со остатокот при делење на c со 4 . Од
2
 a a2
друга страна m 2  am  b   m     b . Бидејќи секој полн квадрат има остаток 0
 2 4
или 1 при делењето со 4 следува дека остатокот на m 2  am  b при делење со 4 е
a2 a2
b или b   1 . Тоа е еден од броевите 0,1,2,3 . Затоа за c земаме вредност која е
4 4
a2 a2
различна од b  и b  1 . Според тоа горната равенката нема решение.
4 4

50. Романија 2002. Нека a е цел број. Докажи дека за секој реален број x, x 2  3 ,

броевите 3  x2 и 3
a  x 3 не можат истовремено да бидат рационални броеви.

92
200 ТЕОРИЈА НА БРОЕВИ - подготвителни задачи

Решение. Нека претпоставиме спротивно, т.е. нека m  3  x 2 и n  3 a  x 3 се


рационални броеви. Тогаш важи x 2  3  m2 и x3  a  n3 , па според тоа
3

a  n   3 m 2
 3  m . Бидејќи левата страна на последното равенство е рационален
2

број следува дека и q  3  m е рационален број.


2

Со квадрирање се добива m 2  q 2  3 . Бидејќи m, q се рационални броеви постојат цели

u v
броеви u, v и w така што m  и q  , и приоа НЗД  u, w  НЗД  v, w  1.
w w

Ако замениме во m 2  q 2  3 добиваме дека 3w2  u 2  v 2 . Јасно, 3 u 2  v 2 од каде


2
следува 3 u и 3 v . Затоа 9 3w т.е. 3 w . Од 3 u , 3 v и 3 w следува дека u, w и v, w
не се заемно прости, што е спротивно на нашата претпоставката.

51. а) Докажи дека бројот nn  2n  4n  6 не е квадрат на природен број.

б) С. Ф. Докажи дека производот на шест последователни природни броеви не може да


биде куб на некој природен број.

  
Решение. а) Од nn  2n  4n  6  nn  6n  2n  4  n 2  6n n 2  6n  8 , со смена
k  n 2  6n , добиваме nn  2n  4n  6  k k  8  k 2  8k .

Од друга страна, k  3  k 2  6k  9  k 2  8k  k 2  8k  16  k  4 . Бидејќи k 2  8k се


2 2

наоѓа помеѓу квадратите на два последователни броеви, k  3 и k  4 , следува дека истиот


не е квадрат на природен број.

б) Нека n, n  1, n  2, n  3, n  4 и n  5 се дадени шест последователни природни броеви.

Ќе докажеме дека nn  1n  2n  3n  4n  5 е број кој се наоѓа меѓу кубовите на два
последователни природни броја.

Имаме,

nn  1n  2n  3n  4n  5 = n2  5nn2  5n  4n2  5n  6 .

Земаме смена A  n2  5n  4 и добиваме

nn  1n  2n  3n  4n  5 =  A  4A A  2  A3  2 A2  8 A . Јасно е дека е исполнето


A3  2 A2  8 A  A3 . Бидејќи n е природен број следува дека A  10 .

93
200 ТЕОРИЈА НА БРОЕВИ - подготвителни задачи

Сега  A  13  A3  3A2  3A  1  A3  2 A2  8 A . Ова следува бидејќи последното


неравенство е еквивалентно со A  11A  1 кое е исполнето за A  11 . За n  2 важи
2

A  18 . За n  1 добиваме nn  1n  2n  3n  4n  5  720 кој не е куб на природен
број. Значи, за секој природен број n важи  A  1  nn  1n  2n  3n  4n  5  A3 ,
3

па затоа следува дека nn  1n  2n  3n  4n  5 не е куб на природен број.

52. Предлог ИМО 1984. Нека n е природен број и a1 , a2 ,...., a2 n се различни цели
броеви. Најди ги сите цели броеви x за кои важи x  a1   x  a2   ...  x  a2n    1 n! ,
n 2

каде n! 1  2  ...  n .

Решение. Нека x е цел број за кој ја важи x  a1   x  a2   ...  x  a2n    1 n! .


n 2

Јасно, x  a1, x  a2 ,...., x  a2n се 2n броеви различни меѓу себе, за кои важи

x  a1   x  a2   ...  x  a2n    1n n!2   1 1  2  2  ....   n  n .


Следува дека секој множител x  ai добива некоја вредност од множеството
 n,n  1,....,1,1,...n  1, n. Од  n   n  1  ....   1  1  2  ....  n  1  n  0 добиваме
a1  a 2  ....  a 2 n
дека 2nx  a1  a2  ....  a2n , односно x  е единствено решение.
2n

53. Најди ги сите четирицифрени броеви кои претставени во каноничен облик го


исполнуваат условот: Збирот од нивните прости делители е еднаков со збирот на
степените на простите делители.

Решение. Нека n  p11 p2 2  ...  pk k е канонична факторизација на бројот n , каде pi се
прости броеви. Бидејќи 214  16384  10000 и pi  2

       1  2 ... k
добиваме 2  n  p1 1 p2 2  ..  pk k  2 1 2 2  ...  2 k  2
14
, па според тоа имаме дека
1   2  ...   k  13 . Бидејќи p1  p2  ...  pk  1   2  ...   k следува дека важи
p1  p2  ...  pk  13 . Сега ги разгледуваме сите можности за простите делители p i :
2,3,5, 2,3,7, 2,5, 2,7, 2,11, 3,5, 3,7, 5,7,2,3,5,7,11,13 , од каде со проверка се добиваат
бараните четирицифрени броеви.

94
200 ТЕОРИЈА НА БРОЕВИ - подготвителни задачи

54. Предлог ИМО 1984. а) Докажи дека не постои тројка од цели броеви m, n, p  која ја
задоволува равенката 4mn  m  n  p 2

б) Докажи дека постојат бесконечно многу тројки од цели броеви m, n, p  кои ја
задоволуваат равенката 4mn  m  n  p  1 . 2

Решение. Нека претпоставиме спротивно, т.е. нека постојат цели броеви m, n, p  за кои
важи 4mn  m  n  p 2 . Ако последната равенка ја помножиме со 4 и додадеме 1 ја
добиваме равенката 4m  14n  1  2 p   1 . Барем еден прост делител на број од облик
2

4m  1 е од облик 4k  1. Меѓутоа, сите прости делители на број од облик x 2  1 , во


нашиот случај 2 p   1 , се од облик 4k  1каде k е цел број. Оттука следува дека не
2

постои тројка m, n, p  за која важи 4mn  m  n  p 2 .

Во вториот случај, ако во 4mn  m  n  p 2  1 замениме m  1 добиваме 3n  p 2 , па ако


земеме n  3k 2 следува дека p  3k , т.е. тројката 1,3k 2
,3k  е решение на
4mn  m  n  p  1 . Според тоа добиваме дека равенката има бесконечно решенија.
2

55. С.Ф. Предлог ЈММО 2013. Нека p1 , p2  5 се прости брoеви за кои важи 6 p1  p2 .

Докажи дека бројот 1p1 p2 1  2 p1 p2 2  3 p1 p2 3  4 p1 p2 4  5 p1 p2 5  6 p1 p2 6 е сложен.

Решение. Бидејќи секој прост број поголем од три може да се запише во облик 6k  1 и
бидејќи 6 p1  p2 следува дека едниот број е од облик 6m  1 а другиот од облик 6n  1.

Значи p1 p2  1mod 6 , од каде следува дека p1 p2  1  6s . Забележуваме дека


1 p1 p2 1
6 p1 p2 6
 1  (1) 6 s 5
 1  1  0 mod 7.

Сега ќе докажеме дека 7 2 p1 p2 2  3 p1 p2 3  4 p1 p2 4  5 p1 p2 5. Ако ја искористиме малата


теорема на Ферма a 6  a 71  1mod 7 , за a  2, 3, 4, 5, 6 добиваме:

2 p1 p2 2  3 p1 p2 3  4 p1 p2 4  5 p1 p2 5  2 p1 p2 11  3 p1 p2 12  4 p1 p2 13  5 p1 p2 14 


  2
  s
 
 26 s1  36 s 2  46 s 3  56 s4  26  2  36  9  46  64  56  625 
s
  s

 2  9  64  625  700  0 mod 7 .

Јасно е дека 7 1p1 p2 1  2 p1 p2 2  3 p1 p2 3  4 p1 p2 4  5 p1 p2 5  6 p1 p2 6 т.е. бројот е сложен.

95
200 ТЕОРИЈА НА БРОЕВИ - подготвителни задачи

56. Романија 2010. Нека a, b, c и d се природни броеви и нека p  a  b  c  d .

Докажи дека ако p е прост број, тогаш p не е делител на бројот ab  cd .

Решение. Јасно е дека a  b  c  d b  a  b  c  d   ab  b2  bc  bd . Нека претпоставиме


дека p е прост делител на бројот ab  cd .

Според тоа добиваме

p ab  b2  bc  bd  ab  cd  b  d b  c  .

Бидејќи p е прост број следува дека p b  d или p b  c. Последното не е можно бидејќи

p  a  b  c  d  b  d и p  a  b  c  d  b  c.

57. Индија 2012. Нека p1  p2  p3  p4 и q1  q2  q3  q4 се прости броеви, така што


p4  p1  8 и q4  q1  8 . Нека p1  5, q1  5 . Докажи дека 30 p1  q1 .

Решение. За секој прост број p  3 важи p  1(mod 6) . Ако p1  1(mod 6)

тогаш p4  3 (mod 6) , што не е можно, бидејќи и p 4 е прост број. Значи, p1  1(mod 6) .

Аналогно важи q1  1(mod 6) , па оттука p1  q1   1   1  0 (mod 6)...1 .

Два од броевите p1  2, p1  4, p1  6 се прости, бидејќи се непарни и се наоѓаат помеѓу


p1 и p4  p1  8 . Бидејќи p1  4  3 (mod 6) добиваме дека ваквиот број не е прост.
Значи, p2  p1  2, p3  p1  6 . Аналогно и q2  q1  2, q3  q1  6 .

Ако p1  2 (mod 5) следува дека p4  0 (mod 5) , што не е можно. Ако p1  3 (mod 5)


следува дека p2  0 mod 5 , а ако p1  4 (mod 5) тогаш p3  0 mod 5 , што исто така не е
можно. Останува p1  1(mod 5) . Аналогно и q1  1(mod 5) . Затоа добиваме
p1  q1  1  1  0 mod 5....(2) . Од 1 и 2 следува 30 p1  q1 .

58. Предлог ИМО 1991. Реши ја равенката 3m  4 n  5 k во множеството природни


броеви.

96
200 ТЕОРИЈА НА БРОЕВИ - подготвителни задачи

Решение. Од 5 k   1 mod 3,4 n  1mod 3 имаме  1  1mod 3 , од каде следува k е


k k

парен број т.е. k  2K . Слично добиваме дека 3m   1 mod 4 и 5 k  1mod 4 па имаме


m

 1m  1mod 4 од каде следува m е парен број, т.е. m  2M . Добиваме

  
2 2n  4 n  5 k  3m  5 2 K  32 M  5 K  3M  5 K  3M ... 1 .

Јасно, постојат природни броеви a и b така што 5 K  3M  2 a и 5 K  3M  2b . Ако ги


одземеме последните две равенки добиваме 2 a 2ba  1  2  3M , па следува дека a  1 и
2 b1  1  3M . Бидејќи 3M  1  3mod 8 следува дека 2 b1  2mod 8 или 2 b1  4mod 8 .
Останува b  1  1, b  2 или b  1  2, b  3 . Од 1 имаме 2n  a  b , па следува дека
b  3, n  2 , M  1, K  1, m  2, k  2 . Значи 2,2,2 е единствено решение на равенката.

59. ИМО 1986. Множеството А  2,5,13 има својство да за секој a, b  A, a  b , бројот


ab  1 е полн квадрат . Докажи дека за секој цел број c  A , множеството A  c го нема
својството на множеството A .

Решение. Нека претпоставиме спротивно дека за секој a, b  2,5,13, c, a  b важи ab  1


е полн квадрат. Значи, постојат цели броеви m, n, p така што
2c  1  m 2 , 5c  1  n 2 ,13c  1  p 2 . Јасно m е непарен број . Бидејќи секој полн квадрат
при делење со 4 има остаток 0 или 1 јасно е дека m2  1 не е делив со 4 , па затоа
m2  1
c е непарен број. Од непарноста на c следува n и p се парни, па постојат n1 и
2
p1 за кои n  2n1, p  2 p1 .

Важи 8c  13c  1  5c  1  p 2  n 2   p  n p  n  4 p1  n1  p1  n1 

т.е. 2c   p1  n1  p1  n1  . Бидејќи десната страна е делива со 2 следува дека броевите p1


и n1 имаат иста парност, па оттука p1  n1 и p1  n1 се парни. Оттука следува дека бројот
 p1  n1  p1  n1  е делив со 4 , но бидејќи c е непарен следува 2c не е делив со 4 .
Доаѓаме до противречност на претпоставката.

60. Најди ги сите природни броеви n така што n  11  sn , каде sn  го означува збирот
на цифри на бројот n .

Решение. Јасно е дека n не може да биде едноцифрен број.

97
200 ТЕОРИЈА НА БРОЕВИ - подготвителни задачи

Нека n е двоцифрен број, т.е. нека n  xy . Важи 10 x  y  11x  y  или x  10 y  0 , од


каде следува дека x  y  0. Оттука не постои двоцифрен број кој ги исполнува условите
на задачата. Ако бараниот број е трицифрен т.е. ако n  xyz  100 x  10 y  z добиваме
дека 100 x  10 y  z  11x  y  z  или 119 x  y   x  y  z   11x  y  z  . Оттука
x  y  z се дели со 11 . Бидејќи x, y, z се цифри следува дека  8  x  y  z  18 , од каде
x  y  z  0 или x  y  z  11 .

Ако важи x  y  z  0 , и замениме во 119 x  y   x  y  z   11x  y  z  , добиваме


дека 88x  11z, z  8x што е можно само кога x  1, z  8 . Со замена во x  y  z  0
следува y  9 , па 198 е баран трицифрен број. Ако важи x  y  z  11 и замениме во
119 x  y   x  y  z   11x  y  z  добиваме дека 8x  z  1 , т.е. x  1, z  9 од каде
следува y  1 , што не е можно. Ако бараниот број има k  4 цифри тогаш со помош на
математичка индукција лесно се докажува дека 10k 1  11 9k , и оттука следува дека
n  10 k 1  11  9k  11  sn . Значи единствен број со ваква особина е бројот 198.

61. ЈБМО 1997. Нека n1 , n2 ,...., n2011, n2012 се природни броеви за кои важи
n  n  ...  n
2
1
2
2
2
2011 n2
2012 . Докажи дека најмалку два од дадените броеви се парни.

Решение. Ако меѓу броевите n1 , n2 ,....., n2011 има барем два парни броеви тогаш решението
следува веднаш. Затоа нека претпоставиме спротивно:

1 Нека помеѓу броевите n1 , n2 ,....., n2011 има еден парен број.

2  Нека помеѓу броевите n1 , n2 ,....., n2011 нема ниеден парен број.

1 Во овој случај бројот n1  n2  ...  n2011 е парен бидејќи е збир од 2010 непарни броеви
2 2 2

и еден парен, што јасно е парен. Значи следува дека и n2012


2
, односно и бројот n2012 е
парен. Оттука имаме најмалку два парни броја помеѓу n1 , n2 ,...., n2011, n2012 .

2  Ако сите броеви n1 , n2 ,....., n2011 се непарни тогаш остатокот при делење на нивните
квадрати со 8 е 1 , односно, остатокот при делење на n12  n22  ...  n2011
2
со 8 е 2011 ,
односно 3 . Бидејќи n12  n22  ...  n2011
2
 n2012
2
следува дека остатокот на квадратот на n2012
при делење со 8 е 3 , што не е можно, бидејќи остатокот на секој квадрат при делење со
8 е 0,1 или 4 .

98
200 ТЕОРИЈА НА БРОЕВИ - подготвителни задачи

Значи, случајот 2  не е можен, па затоа следува дека меѓу броевите n1 , n2 ,....n2011, n2012 има
барем два парни броеви.

1
62. Предлог ИМО 1999. Докажи дека секој позитивен рационален број  r  2 може да
2
a3  b3
се запише во облик , каде a, b, c и d се природни броеви.
c3  d 3

Решение. Воочуваме дека m  n  2m  n  9m m 2  mn  n 2 и


3 3
 
m  n  2n  m
3 3

 9n m  mn  n
2 2
 за сите природни броеви m и n . Значи,

m m  n   2m  n 
3 3
 .
n m  n 3  2n  m 3

m a3  b3

Ако ставиме смени a  m  n, b  2m  n, c  m  n и d  2n  m добиваме .
n c3  d 3
Очигледно a и c се природни броеви. Но b и d се природни ако и само ако 2m  n и
1 m
2n  m . Оттука следува дека  r   2 .
2 n

63. Предлог ИМО 1985. Нека a, b, c се позитивни реални броеви. Најди ги сите
позитивни реални броеви x, y, z за кои важи x y  z  abc и
 
4 xyz  a 2 x  b 2 y  c 2 z  abc .

bc ca ab


Решение. Земаме смени x   m, y   n, z   p,
2 2 2

bc ca ab


каде m  ,n , p . Од условот на задачата x  y  z  a  b  c добиваме
2 2 2
дека m  n  p  0 . Ако ги замениме x, y, z во abc  a 2 x  b 2 y  c 2 z  4 xyz имаме
 
2 am 2  bn 2  cp 2  2mnp  0 ... 1 . Ако mnp  0 тогаш 1 не е можно, па затоа mnp  0 .
Значи, два од овие три броја се ненегативни, па без губење на општоста земаме m, n  0 .
Заменувајќи p  m  n во 1 добиваме 2a  c  2nm 2  b  c  2mn 2  2cmn  0 .

Последното равенство е можно само ако m  n  0. Следува дека и p  0 , па според тоа


bc ca ab
добиваме x  , y , z .
2 2 2

99
200 ТЕОРИЈА НА БРОЕВИ - подготвителни задачи

64. Предлог ИМО 1992. Докажи дека за секој природен број m постојат бесконечно
парови природни броеви x, y  за кои се исполенти следните услови:

1 x и y се заемно прости броеви,

2 y x  m ,
2

3 x y  m .
2

Решение. Нека претпоставиме дека парот x, y  , каде x  y , ги задоволува условите на


задачата. Ќе докажеме дека и парот  y, z  исто така ги задоволува условите на задачата,
y m
2
каде z  . Бидејќи еден пар кој ги задоволува условите на задачата е 1,1 ќе
x
следува дека можеме да наоѓаме бесконечно парови за кои ќе важи истото. Јасно,
y2  m y m
z  y    y  0 . Ќе покажеме дека НЗД z, y   1 . Нека претпоставиме
x x x
спротивно, нека НЗД z, y   d  1 . Постои прост број p така што p d . Оттука следува
дека p z и p y. Тогаш p z y 2  m , од каде поради p y следува p m.

Бидејќи p y x 2  m и p m следува дека p x 2 , и бидејќи p е прост број добиваме p x.


Значи p x и p y, што не е можно поради НЗД  x, y   1.

y2  m
Добиваме дека НЗД z, y   1 . Од x y 2  m следува дека z  е цел број,
x

па оттука z y  m . Останува да докажеме дека важи y z  m . Бидејќи x и y се заемно


2 2

прости следува дека доволно е да докажеме y x z  m .


2 2
 
2 y  2y m  m 
 
4 2 2
Имаме x z  m  x   m   y 4  2 y 2 m  m 2  mx 2
2 2
2
 x 

     
 y 2 y 2  2m  m x 2  m , па следува y x z  m т.е. y z  m .
2 2 2

65. Канада 2011. Разгледуваме 70  цифрен број со својство секоја од цифрите 1, 2, ..,7

се појавува по точно 10 пати, 8,9,0 не се појавуваат во неговиот декаден запис.

100
200 ТЕОРИЈА НА БРОЕВИ - подготвителни задачи

Докажи дека во множеството од сите такви броеви не постојат два така што едниот го
дели другиот.

Решение. Нека a и b се броеви од даденото множество за кои важи a b .

За секој цел број n важи n  snmod 9 па следува дека a  101  2  ...  7  1mod 9 .

Исто така и b  1 mod 9 . Од a b следува дека постои природен број k  1 така што
b
b  ka . Бидејќи b  77..766..6....11..1 следува дека 1  k   6.
a

Оттука добиваме дека 1  b  ka  k mod 9 . Ова не е можно бидејќи k  2,3,4,5,6.

66. Во множеството природни броеви реши ја равенката 2 x x  y y  3z z .

Решение. Јасно е дека ако ставиме x  y  z добиваме 3x x  3x x , односно равенката


важи за сите природни броеви x  y  z . Нека претпоставиме дека x, y, z  е решение на
равенката и нека z е најголем меѓу броевите.

Добиваме дека 3z z  2 z z  z z  2 y y  x x  3z z , па оттука равенката нема решение. Ако


земеме дека y е најголем меѓу броевите тогаш y  3, y  1  z па затоа y y  y  y y 1  3z z .
Ако x е најголем тогаш следува x x  x  x x1  3z z . Според тоа не постои друга тројка од
природни броеви која е решение на равенката 2 x x  y y  3z z .

67. Сингапур 2007. Најди ги сите парови од ненегативни цели броеви x, y  кои ја
задоволуваат равенката 14 y x  y x  y  2007.

Решение. Ако x  0 добиваме y y  2006, што не е можно за ниеден ненегативен цел број
y. За y  0 задачата нема смисла. Затоа x, y  1. Ако x  3 тогаш

14 y   y x  y  14 y   14x  143  2744  2007.


x x

Останува x  1 или x  2. Ако x  1 ја добиваме равенката y 14  y y   2007  32  223.

Лесно се проверува дека равенката нема решение.

Ако x  2 имаме y 2 196  y y   2007 од каде се добива y  3. Значи  2,3 е единствено


решение на равенката.

101
200 ТЕОРИЈА НА БРОЕВИ - подготвителни задачи

68. Кореа 2012. Најди ги сите тројки m, p, q  така што m е природен број, p, q се
прости броеви и важи 2 p  1  q .
m 2 5

Решение. Од q  1 q 5  1 следува дека q  1 2 m p 2 . Оттука q  1  2 n p k каде n  m и


2m p 2 q5  1
k  0,1,2 . Бидејќи 2 m  n p 2 k    q 4  q 3  q 2  q  1 е непарен број
2n p k q 1
добиваме m  n . Важи 2 m p 2  1  2 m p k  1  2 5m p 5k  1 па затоа p 25k  2 4m  1 .
5

Следува 2  5k  0 односно k  0 . Значи q  2 m  1 . Ако замениме во дадената равенка


добиваме p 2  2 4m  5  23m  10  2 2m  10  2 m  5 . Ако m  2 добиваме дека p 2  5 mod 8
што не е можно, бидејќи p 2  0,1,4mod 8 . Добиваме дека m  1 т.е. q  3 и
3 1
5
p  121  11 . Значи 1,11,3 е единствената тројка која ја задоволува горната
2
равенка.

69. ЈАР 2012. Нека p и k се природни броеви така што p е прост број и k  1 .

Докажи дека постои најмногу еден пар x, y  од природни броеви така што важи

x k  px  y k .

Решение. Јасно е дека x  y. Нека НЗД x, y   d . Постојат природни броеви x1 и y1

такви што x  x1d и y  y1d и притоа НЗД x1 , y1   1 . Добиваме p 


 
d k 1 y1k  x1k
.
x1

 
Од НЗД x1 , y1   1 следува НЗД x1 , y1k  x1k  1 па затоа x1 d k 1 .

Бидејќи y1k  x1k  1 и p е прост број следува дека x1  d k 1 и p  y1k  x1k .

Од y1  x1 y1k  x1k и y1  x1  y1k  x1k имаме y1  x1  1 т.е. y1  1  x1 .

Значи p  1  x1 k  x1k . Функцијата f t   1  t k  t k е монотоно растечка кога t  0.

Според тоа најмногу за една вредност на x1 е можно p  1  x1 k  x1k .

Оттука следува дека постои најмногу еден пар x, y  од природни броеви така што важи
x k  px  y k .

102
200 ТЕОРИЈА НА БРОЕВИ - подготвителни задачи

70. Кореа 2005. Нека a и b се заемно прости природни броеви. Најди ги сите природни
броеви кои можат да бидат вредност за најголем заеднички делител на броевите a  b и
a 2005  b 2005
.
ab

 a 2005  b 2005 
Решение. Нека d  НЗД  a  b,  . Од d a  b добиваме a  b mod d  .
 ab 

a 2005  b 2005
Исто така  a 2004  a 2003b  ....  ab 2003  b 2004  0 mod d  . Ако во последната
ab

конгруенција замениме a  b mod d  добиваме b 2004  b 2004  ...  b 2004  0 mod d 

т.е. d 2005b 2004, како и d 2005a 2004 . Ако 2005 не се дели со d добиваме дека d a и
d b , што не е можно бидејќи a и b се заемно прости броеви. Значи d 2005 , од каде

следува дека d 1,5, 401, 2005.

71. УСАМО 1972.

НЗСa, b, c  НЗД a, b, c 


2 2
Докажи дека  .
НЗСa, b НЗСb, c НЗСc, a  НЗД a, b НЗД b, c НЗД c, a 

НЗСa, b НЗСb, c НЗСc, a  НЗД a, b НЗД b, c НЗД c, a 


Решение. Ќе докажеме  . .(1).
НЗСa, b, c  НЗД a, b, c 
2 2

Нека p е произволен прост број и нека  ,  и  се највисоките степени на p за кои


p  a, p  b и p  c . Без губење на општостта претпоставуваме дека      .

Највисокиот степен ( x ) на p за кој p x го дели НЗСа, b , НЗСb, c  , НЗСc, a  и


НЗСa, b, c  е  ,  ,  и 2 , соодветно. Според тоа највисокиот степен на p за кој p x
2

го дели бројот од левата страна на (1) е       2   .

На ист начин, највисокиот степен на p за кој p x го дели НЗД а, b , НЗД b, c  ,
НЗД c, a  и НЗД a, b, c 2 е  ,  ,  и 2 , соодветно. Според тоа највисокиот степен на
p за кој p x го дели бројот од десната страна на (1) е       2   .

103
200 ТЕОРИЈА НА БРОЕВИ - подготвителни задачи

72. Холандија 1996. Нека c е ненегативен цел број, и дефинираме a n  n 2  c, n  1.

Нека d n е најголемиот заеднички делител на a n и a n 1 .

а) Нека c  0. Докажи дека d n  1,  n  1.

б) Нека c  1. Докажи дека d n  1, 5,  n  1.

в) Докажи дека d n  4c  1,  n  1.

Решение. а) Нека c  0. Тогаш добиваме d n  НЗД аn , an1   НЗД n 2 , n 2  2n  1 


  
 НЗД n 2 , n 2  2n  1  n 2  НЗД n 2 , 2n  1  1. 
б) Нека c  1. Ако n  1 тогаш d1  НЗД  а1 , а2   НЗД  2,5  1.

Нека n  2.

Важи d n  НЗД аn , an1   НЗД n 2  1, n 2  2n  2  НЗД n 2  1, n 2  2n  2  n 2  1 

     
 НЗД n 2  1, 2n  1  НЗД n 2  1  2n  1, 2n  1  НЗД n 2  2n, 2n  1  НЗД nn  2, 2n  1.

Бидејќи n и 2n  1 се заемно прости броеви добиваме дека:

НЗД nn  2, 2n  1  НЗД n  2, 2n  1.

Значи,

НЗД nn  2, 2n  1  НЗД n  2, 2n  1  НЗД n  2, 2n  1  n  2  НЗД n  2, n  3 


 НЗД n  2, n  3  n  2  НЗД n  2, 5 1, 5.

в) Бидејќи d n a n и d n a n 1 следува дека d n a n  an 1 , d n a n  a n 1 .

Оттука d n 2a n  a n 1  и d n  a n  a n 1  , па следува


2

d n 2a n  2an1  an2  2an an1  an21 .

Од a n  n 2  c и a n1  n 2  2n  c  1 имаме 2an  2an1  an2  2an an1  an21  4c  1.

Според тоа добиваме дека d n 4c  1, од каде следува дека d n  4c  1.

bc ca ab
73. Германија 2008. Нека a, b и c се природни броеви така што , и
bc ca ab

104
200 ТЕОРИЈА НА БРОЕВИ - подготвителни задачи

се природни броеви. Докажи дека НЗД a, b, c   1 .

Решение. Нека НЗД a, b  d1 , НЗД b, c   d 2 , НЗД a, c   d 3 . Постојат природни броеви
a1 , b1 , b2 , c2 , a3 , b3 за кои е исполнето НЗД  a1 , b1   НЗД  b2 , c2   НЗД  a3 , c3   1 и
a  a1d1 , b  b1d1 , b  b2 d 2 , c  c2 d 2 , a  a3 d 3 и c  c3 d 3 .

b2 c 2 d 2 a3 c3 d 3 abd
Следува дека броевите , и 1 1 1 се природни.
b2  c 2 a3  c3 a1  b1

Бидејќи НЗД b2  c2 , b2 c2   1, НЗД a3  c3 , a3 c3   1 и НЗД a1  b1 , a1b1   1 добиваме дека
b2  c2 d 2 , a3  c3 d 3 и a1  b1 d1 . Значи d 2  b2  c2 , d 3  a3  c3 и d1  a1  b1 .

Ќе докажеме дека НЗД d1 , d 2   1 . Нека претпоставиме дека НЗД d1 , d 2   1 .

Од b1d1  b2 d 2 следува дека d1 b2 и d 2 b1 , т.е. b2  d1 и b1  d 2 .

Според тоа добиваме b1  b2  d1  d 2  a1  b1   b2  c2  , т.е. 0  a1  c1 , што не е можно.

Значи, НЗД d1 , d 2   t  2 . Важи t d1 , t d 2 , па затоа t a, t b и t c .

a a 2  b2
74. Романија 1999. Нека a, b, c се ненулти цели броеви, a  c, така што  .
c c 2  b2
Докажи дека a 2  b2  c2 неможе да биде прост број.

a a 2  b2
Решение. Идентитетот  го запишуваме во облик  a  c   b2  ac   0.
c c 2  b2

Бидејќи a  c следува дека b2  ac  0.

Добиваме a 2  b2  c2  a 2  ac  c2  a 2  2ac  c2  b2   a  c   b2   a  c  b  a  c  b  .
2

Бидејќи a 2  b2  c2  3 е прост број следува дека важи еден од четирите случаи:

1) a  c  b  1, a  c  b  a 2  b2  c 2 ,

2) a  c  b  1, a  c  b    a 2  b2  c 2  ,

3) a  c  b  a 2  b2  c2 , a  c  b  1,

105
200 ТЕОРИЈА НА БРОЕВИ - подготвителни задачи

4) a  c  b    a 2  b2  c 2  , a  c  b  1.

Во првиот и третиот случај добиваме дека  a  1  b2   c  1  1 од каде следува дека


2 2

a  c  1, што е противречност со a  c.

Во вториот и четвртиот случај добиваме дека  a  1  b2   c  1  1 оде каде следува


2 2

a  c  1.

75. Докажи дека низата 1,11,111,... содржи бесконечно поднизи чии членови се попарно
заемно прости.

10n  1
Решение. Општиот член на дадената низа го запишуваме во облик an  .
9

Нека m и n се заемно прости броеви. Бидејќи НЗД  xm  1, x n  1  x НЗД  m, n  1 имаме:

 10m  1 10n  1  1
НЗД  am , an   НЗД 
 9
,
9  9
m
 n 1
  НЗД 10  1,10  1  10
9
 
НЗД  m , n 
 1  1. 
Спред тоа поднизата a p  , каде p е прост број, е низа чии членови се попарно заемно
прости бреови бидејќи НЗД  p, q   1 , за сите прости броеви p и q.

76. ЈБМО 1998. Докажи дека бројот 1111....111222....22225 запишан со 1997 единици,
1998 двојки е полн квадрат.
n

99...9 10n  1
Решение 1. Важи 111....11   .
n 9 9

Добиваме 11...1122...225  11...1100...0  22...220  5  101999 11...11  10  2 11...11  5 


1997 1998 1997 1999 1998 1997 1998

2
 101997  1   101998  1  103996  101999  25  101998  5 
 101999     20   5   .
 9   9  9  3 

Бидејќи 3 101988  5 следува дека бараниот број е квадрат на природен број.

Решение 2. Имаме аn an1 ...a1a0  a0  10a1  ...  10 n1 an1  10 n an .

106
200 ТЕОРИЈА НА БРОЕВИ - подготвителни задачи

1111 .... 111222 .... 22225  10 3995  10 3994  ...  10 1999  2  10 1998  10 1997  ...  10 
 5  10 1999  10 1996  10 1995  ...  10  1  2 10 10 1997  10 1996  ...  10  1  5 
2
10 1997  1 10 1998  1 10 3996  10 1999  25  10 1998  5 
 10 1999   20  5    .
10  1 10  1 9  3 

Бидејќи збирот на цифри на бројот 101998  5 е 6 следува дека е делив со 3 , па оттука


101998  5
бројот 1111....111222....22225 е квадрат на природниот број .
3

x n 1  1
Притоа ја користивме формулата 1  x  x  ....  x  , x  1.
2 n

x 1

77. Нека p е прост број таков што сите цифри му се единици. Докажи дека бројот на
цифрите на p е прост број.

Решение 1. Нека бројот на цифри на простиот број p е сложен, т.е. нека е a  b , каде
a, b  1. За декадното претставување на p имаме:

p  11
    
11  1  10  10 2  ....  10 ab1  1  10  10 2  ...  10 a 1  10 a 1  10  10 2  ...  10 a 1  ...
...
abединици

    
 10 a b1 1  10  10 2  ...  10 a 1  1  10  10 2  ...  10 a 1 1  10 a  10 2 a  ...  10 a b1 .

Значи p е производ на два природни броеви поголеми од еден, што не е можно.

Следува дека бројот на цифри на бројот p е прост.

Решение 2. Нека p  11...11 е прост број и нека n  a  b, a, b  1.


n

 
10n  1 10ab  1 10  1
 
a

  
b 1 b2
Имаме 11...11     10a  10a  ...  10a  1 .
n 9 9 9

Бидејќи a  1 имаме 10a  1  9 и 9 10a  1. Оттука следува дека p е сложен број, што е
спротивно на претпоставката. Значи n е прост број.

107
200 ТЕОРИЈА НА БРОЕВИ - подготвителни задачи

78. Бразил 1991. Докажи дека постои природен број n  2 така што 1991 199
 991 .
...
n деветки

10 n  1
 
Решение. Имаме 199...91  1  10 n1  9 10 n  10 n1  ...  10  1  10 n1  9  10 
10  1
1 

 10 n1  10 n1  9  2  10 n1  9 . Од 2  10 n1  9  0 mod 1991 важи 2  10 n1  9 mod 1991
т.е. 10 n1  1000 mod 1991, 10 n2  103  1000 mod 1991, 10 n2  1mod 1991.

Од Ојлеровата теорема имаме 10 1991  1mod 1991 .

 1  1 
Бидејќи 1991  11  181 имаме  1991  1991  1    1    10  180  1800 .
 11   181 

Следува дека n  2  1800k , т.е. n  1800k  2 , каде k  1 .

79. Индонезија 2007. Нека m и n се два природни броеви. Ако постојат бесконечно
многу цели броеви k така што k 2  2kn  m 2 е полн квадрат, тогаш m  n . Докажи!

Решение. Ќе разгледаме два случаи:

m 2  n 2  2n  1
1) Нека m  n . Доволно е да избериме k  и добиваме:
2

 k  n  k 2  2kn  n2  k 2  2kn  m2  k 2  2kn  2k  n2  2n  1   k  n  1 .


2 2

Оттука за бесконечно цели броеви k , бројот k 2  2kn  m2 не е полн квадрат.

2) Нека m  n. На ист начин како во претходниот случај избираме


n  m  2n  1
2 2
k и добиваме:
2

 k  n 1  k 2  n2  1  2kn  2k  2n  k 2  2kn  m2  k 2  2kn  n2   k  n  .


2 2

Оттука за бесконечно цели броеви k , бројот k 2  2kn  m2 не е полн квадрат.

Останува дека m  n.

108
200 ТЕОРИЈА НА БРОЕВИ - подготвителни задачи

80. Пан Африка 2012. Најдете ги сите природни броеви m и n така што n m  m

го дели бројот m 2  2m .

Решение. Бидејќи nm  m m2  2m и m2  2m  0 следува дека важи n m  m  m 2  2m .

Ако n  1 добиваме 1  m m2  2m  1  m 1  m   4 1  m   3  1  m 3.
2

Имаме 1  m  1,  3. Бидејќи m е природен број следува дека m  2 или m  4.

Ако n  2 тогаш 2 m  m  n m  m  m 2  2m , односно m 2  3m  2 m .

Со помош на математичка индукција ќе докажеме дека 2 m  m 2  3m , за m  6.

За m  6 имаме 64  54. Нека за k  6 важи 2 k  k 2  3k .

Важи

 
2 k 1  2  2 k  2 k 2  3k  2k 2  6k  k 2  5k  k 2  k  k 2  5k  42  k 2  5k  4 
 k  1  3k  1.
2

Оттука следува дека 2 m  m 2  3m , за m  6.

На крај, лесно се проверува за кои вредности n важи nm  m m2  2m ако m1, 2,3, 4,5.

Добиваме m, n  1, 2, 1, 4, 2, 2, 3, 2, 4, 2.

81. Нека a, b, c....k се природни броеви такви што a 2b  1, b 2c  1,...., k 2a 1.

Докажи дека a  b  c  ...  k  1 .

Решение. Нека p е најмал прост делител на 2 b  1 и нека претпоставиме дека b  1.


Јасно 2 b  1mod p  . Од теорема на Ферма следува дека 2 p 1  1mod p  .

Ако A  НЗД  b, p  1 тогаш 2 A  1mod p  . Бидејќи A p  1 следува дека A  p  1  p.

Од A p следува дека A 2c  1. Ако q е прост делител на A добиваме дека постои прост


делител ( q  p ) на 2 c  1. Ако истата анализа ја повториме за броевите 2c  1,..., 2a  1

ќе докажиме дека постои прост делител p l на 2 b  1 , помал од p . Од минималноста на


p добиваме противречност со тоа дека p l е помал прост делител од p . Значи b  1.

109
200 ТЕОРИЈА НА БРОЕВИ - подготвителни задачи

82. Балтички натпревар 2011. Определи ги сите парови  p, q  од прости броеви за кои
p 2  q 3 и q 2  p 3 се полни квадрати.

Решение. Ако p  q тогаш имаме p 2  p 3  a 2 т.е. p 2  p  1  a 2 .

Јасно, p  1  b 2 . Оттука следува p  b  1b  1 , и бидејќи


p е прост број имаме
b  1  1, b  2, p  3. Сега нека p  q. Без губење на општоста земаме дека p  q.
Очигледно q  3. Од p 2  q 3  a 2 добиваме дека q 3  a  p a  p .

Бидејќи НЗД  a  p, a  p   НЗД  a  p, 2 p   НЗД  q, 2 p   1 добиваме дека a  p и


a  p се заемно прости броеви, од каде следува дека a  p  1 и a  p  q 3 . Со одземање
 
на последните две равенки се добива 2 p  q 3  1  q  1 q 2  q  1  2q , што е спротивно
на претпоставката p  q. Останува дека 3, 3 е единствениот пар кој ги задоволува
условите на задачата.

83. С.Ф. Најди ги сите a, b, c, d  N за кои a8  b8  c8  d 8  a 2  b 2  c 2  d 2  5  252013

и збирот на цифри на бројот a 6  b6  c 6  d 6  a 4  b 4  c 4  d 4 изнесува 2014.

  
Решение. Од 3 a 3  a и 5 a 5  a следува 15 a 3  a a 5  a , односно 15 a8  a 2  a 6  a 4 .
Значи a8  a 2  a 6  a 4 mod 15, b8  b 2  b6  b 4 mod 15, c8  c 2  c 6  c 4 mod 15 и

d 8  d 2  d 6  d 4 mod 15.

Со собирање на овие конгруентни равенки добиваме

a8  b8  c8  d 8  a 2  b 2  c 2  d 2  a 6  b6  c 6  d 6  a 4  b 4  c 4  d 4 mod 15.

Лесно се проверува дека 25n  10 mod 15,  n  N , па затоа 15 5  252013.

Бидејќи 2014 не се дели со 3 следува дека бројот a 6  b6  c 6  d 6  a 4  b 4  c 4  d 4 не


се дели со 3 , од каде следува дека тој број не се дели со 15.

Според тоа се добива противречност со 15 a8  b8  c8  d 8  a 2  b 2  c 2  d 2 .

Значи, не постојат природни броеви a, b, c, d кои го задоволуваат условот на задачата.

110
200 ТЕОРИЈА НА БРОЕВИ - подготвителни задачи

84. УМО 1998. Нека a е цел број. Докажи дека не постојат цели броеви b и c, c  1, и за
кои важи a  12  a  22  ...  a  992  b c .

Решение. Нека претпоставиме дека постојат цели броеви b и c така што c  1, за кои е
исполнето a  12  a  22  ...  a  992  b c . По квадрирање добиваме дека важи

a  2a  1  a  2  2a  4  ...  a  2  99a  99   99a  2a1  2  ...  99 


2 2 2 2 2

99  100 99  100  199


 1  2  ..  99   99a  2 
2 2 2
a2
 99a  9900a  33  50  199  2

2 6
 333a  300a  50  199.
2

 
Оттука се добива 33 3a 2  300a  50  199  b c . Сега јасно е дека 3 b. Бидејќи c  2 следува
2 c
 2

дека 3 b . Од 33 3a  300a  50  199  b c имаме дека 3 2 333a 2  300a  50  199 ,

што не е можно бидејќи 3a 2  300a  50  199 не се дели со 3.

1 1 1
85. Докажи дека збирот S    ...  , каде n  1 , не може да биде цел број.
2 3 n

Решение. Нека a е најголемиот цел број таков што 2 a  n и нека p  3  5  ...  k е


производ од сите непарни броеви кои не го надминуваат n. Ќе докажеме дека 2 a 1  p  S
не е цел број од каде ќе следува дека S не е цел број ( бидејќи 2 a 1  p е цел број).

Важи

2 a 1  3  5  ....  k  2 a 1  3  5  ....  k  2 a 1  3  5  ....  k  2 a 1  3  5  ....  k 


2 a 1  p  S    ... a
 ..  .
2 3 2 n

Бидејќи за секој m  n важи m  2 s  t , каде 2 s  2 a и t е непарен број , t  k , следува


дека 2 s t 2 a 1 3  5    k  . Сите собироци во горниот збир се цели броеви освен
2 a 1 3  5  ...  k 
.
2a

Оттука добиваме дека бројот 2 a 1  p  S не е цел број, од каде и S не е цел број.

86. Русија 2001. Нека a и b се различни природни броеви такви што бројот a 2  ab  b 2
е делител на бројот aba  b. Докажи дека a  b  3 ab .

111
200 ТЕОРИЈА НА БРОЕВИ - подготвителни задачи

Решение. Нека НЗД a, b  c . Постојат цели броеви a1 и b1 така што a  a1c ,

b  b1c и притоа НЗД a1 , b1   1. Ако замениме a  a1c и b  b1c во


a  ab  b aba  b добиваме дека a  a1b1  b a1b1 a1  b1 c.
2 2 2
1 1
2

     
Бидејќи НЗД a12  a1b1  b12 , a1  НЗД b12 , a1  1 , НЗД a12  a1b1  b12 , b1  НЗД a12 , b1  1 и  
 2
1
2
1   2
1  
НЗД а  a1b1  b , a1  b1  НЗД a1 a1  b1   b , a1  b1  НЗД b , a1  b1  НЗД b , a1  1
1
2
  1
2

следува дека a  a1b1  b c. Значи c  a  a1b1  b .
2
1 1
2 2
1 1
2

Сега добиваме a  b  ca1  b1   c 2 a1  b1  c  c 2  1  a12  a1b1  b12   c 2 a1b1  ab.


3 3 3

Според тоа следува дека a  b  3 ab .

87. Ако НЗД b, c   1, докажи дека НЗД a, bc   НЗД a, b  НЗД a, c .

Решение. Нека НЗД a, bc   m, НЗД a, b  n и НЗД a, c   p. Јасно n b и p c , и бидејќи

НЗД b, c   1 следува НЗД n, p   1. Исто така n a и p a , и бидејќи n и p се заемно


прости следува дека np a. Од n b и p c добиваме np bc. Сега np a и np bc , па оттука
важи np НЗД a, bc   m. Од теорема на Безу следува дека постојат цели броеви x, y така
што n  ax  by, и цели броеви x1 , y1 за кои p  ax1  cy1 .

Имаме: np  ax  by ax1  cy1   a 2 xx1  acxy1  abyx1  bcyy1 . Бидејќи m a и m bc


следува дека m a xx1  acxy1  abyx1  bcyy1 т.е. m np. Важи m np и np m , од каде
2

следува m  np.

 2n 
88. Докажи дека n  1  , за секој природен број n.
n 

 2n  1 2n  1! n  1  2n  1  2n!  2n  1  2n! 


Решение. Имаме n  1   n  1 n !n  1! 
 n  n !n  1  n ! n !n !

 2n   2n 
 2n  1   . Според тоа важи n  1 2n  1    .
n n

Бидејќи НЗД n  1, 2n  1  НЗД n  1, 2n  1  n  1  НЗД n  1, n  1 следува дека

112
200 ТЕОРИЈА НА БРОЕВИ - подготвителни задачи

 2n 
n  1  .
n

89. Нека n е природен број. Докажи дека производот на n последователни природни


броеви е делив со n !.

Решение. Нека m, m  1, ...., m  n  1 се n последователни природни броеви.

mm  1m  2    m  n  1 1  2    m  1  mm  1m  2    m  n  1


 
n! n !1  2    m  1


m  n  1!   m  n  1  N .
n ! m  n  1  n !  n 

n
n
90. Чешка 2004. Најди ги сите природни броеви n така што  k ! е цел број.
k 1

n
1 1 1 1 n ! n(n  1)  ...  3  ....nn  1  n  1
Решение. Бидејќи  k !  1!  2!  ...  n! 
k 1 n!

n  1S  n  1 n
n n  1S  n  1 . За бројот n n

n!
добиваме дека  k! 
k 1 n  1! 
k 1 k !
да биде цел треба

n  1! n  1S  n  1. Бидејќи n  1 n  1! и n  1 n  1S следува дека мора n  1 n  1.

Јасно n  1 n  1  2 па оттука n  1 2. Значи n  2 и n  3.

91. Предлог ИМО 2005.

Нека a, b, c, d , e и f се природни броеви и нека S  a  b  c  d  e  f . Ако S ги дели


броевите abc  def и ab  bc  ca  de  ef  df тогаш S е сложен број. Докажи!

Решение. Го разгледуваме полиномот Px   x  a x  bx  c   x  d x  ex  f  

 a  b  c  d  e  f x 2  ab  bc  ca  de  ef  fd x  abc  def  Sx 2  Rx  Q.

Од условот на задачата важи S Q и S R , па оттука следува дека S Px  , за секој цел


број x. Ако x  d имаме Pd   a  d b  d c  d  , односно S  a  d  b  d  c  d  .

113
200 ТЕОРИЈА НА БРОЕВИ - подготвителни задачи

Ако S е прост број тогаш од S  a  d  b  d  c  d  следува дека S a  d или S b  d


или S c  d . Но бидејќи S  a  d , b  d , c  d следува дека S мора да биде сложен број.

92. С.Ф. Дали постои природен број n кој има шест природни делители чии збир од
нивните шести степени е 5000054?

Решение 1. Бидејќи d  n   6  1 6  2  3 добиваме дека n  p5 или n  pq 2 , p  q.

Ако n  p5 имаме 16  p6  p12  p18  p24  p30  5000054 …. (1).

Бидејќи p  2 имаме 16  p6  p12  p18  p 24  p30  230   210   10243  10003  5000054.
3

Оттука равенката (1) нема решение во множеството прости броеви.

Сега, нека n  pq 2 . Од условот на задачата имаме

1  p6  q6  q12  p6 q6  p6 q12  5000054.

Ако p  5 добиваме:

1  p6  q6  q12  p6 q6  p6 q12  1  p6 1  q6  q12   56  212  46  212  224  5000054.

Ако p  2 тогаш 5000054  1  p6 1  q6  q12   65 1  q6  q12  . Последната равенка не е


можна бидејќи 5000054 не се дели со 65.

Ако p  3 тогаш 5000054  1  p6 1  q6  q12   1  36 1  q6  q12   730 1  q6  q12  .

Последната равенка не е можна бидејќи 5000054 не се дели со 10.

Останува дека не постои природен број n кој ги исполнува условите на задачата.

Решение 2. Бидејќи 6  2  3 следува дека n има најмногу два прости делители, т.е. нека
n  p1  p2 каде a  1, b  0 . Ако n е парен број тогаш n 6 е делив со 8 , а ако n е
a b

непарен тогаш n 6 има остаток 1 при делење со 8 . Нека d1 , d 2 ,..., d 6 се природни


делители на n . Бидејќи 5000054  5  10 6  54 има остаток 6 при делење со 8 добиваме
дека d16  d 26  d 36  d 46  d 56  d 66 има остаток 6 при делење со 8 , од каде следува дека
сите делители на n се непарни. Јасно 2 не е прост делител на n , т.е. pi  3 . Ако ниеден
7 тогаш од теорема на Ферма важи 7 d i  1 , па оттука следува дека
6
d i не се дели со

114
200 ТЕОРИЈА НА БРОЕВИ - подготвителни задачи

d16  d 26  d 36  d 46  d 56  d 66 има остаток 6 при делење со 7 , но бидејќи 5000054 при


делење со 7 има остаток 3 следува дека еден прост делител на n е 7 . Јасно е дека n
има два прости делители. Можни се два случаи n  7  p2
2
или n  7 2  p2 . Во првиот
случај сите делители на n се 1,7,7 p2 , p2 , p22 и 7 p 22 , додека во вториот случај природни
делители на бројот n се 1,7,49, p2 ,7 p2 и 49 p 2 . Вториот случај не е можен бидејќи
остатокот при делење на d16  d 26  d 36  d 46  d 56  d 66 со 7 е 2 .

Значи останува да се реши равенката по непознатата p 2 ,

1  7 6  7 6 p26  p26  p12 6 12



2  7 p2  1  7
6
 
2  p2  1  5  10  54 .
p12 6 6

Бидејќи 1  7 6 312  36  1  6  10 6 следува дека таков природен број n не постои.

93. Јапонија 2009. Најди ги сите природни броеви n така што 8 n  n се дели со

2 n  n.

 
Решение. Бидејќи a 3  b 3  a  b a 2  ab  b 2 јасно е дека a  b a 3  b 3 .

Значи 2 n  n 8 n  n 3 . Од условот на задачата 2 n  n 8 n  n , па според тоа важи

2 n  n 8 n  n 3   8 n  n т.е. 2 n  n n 3  n.

Ако n 3  n  0 следува n  1 е едно решение.

Ако n 3  n  0 јасно е дека n 3  n е природен број таков што n 3  n  2 n.

Сега со математичка индукција ќе докажеме дека 2 n  n 3 , n  10.

За n  10 важи 1024  1000. Нека за сите природни броеви помали или еднакви на k
важи 2 k  k 3 . Од претпоставката добиваме 2 k 1  2  2 k  2k 3 .

Ќе докажеме дека за k  10 важи 2k 3  k  13 .

Доволно е да докажеме дека k 3  3k 2  3k  1.

Имаме k 3  k  k 2  9k 2  3k 2  3k 2  3k 2  3k 2  3k  1. Значи 2 n  n 3 , n  10.

Останува да се провери за кои вредности n од множеството 2, 3, ...,9 важи 2 n  n n 3  n.

115
200 ТЕОРИЈА НА БРОЕВИ - подготвителни задачи

Добиваме дека n  1, 2, 4,6 се природни броеви кои го исполнуваат условот на задачата.

94. Канада 1999. Определи го бројот на реални решенија на равенката

a a a


 2    3    5   a.

a a a


Решение. Бројот a е цел број бидејќи е збир од целите броеви  ,   и  .
2 3 5

Нека ставиме a  30m  r каде m и r се цели броеви така што 0  r  30.

Сега ако во дадената равенка замениме за a  30m  r добиваме дека важи

 r  r  r
15m  2   10m  3   6m  5   30m  r. Ако искористиме a  b  a  b , кога a е
цел број а b е произволен реален број, за горната равенка добиваме:

15m  10m  6m   r    r    r   30m  r r  r  r 


т.е. m  r         .
2 3 5  2  3 5

Сега за произволна вредност r  0,1, ...,29 добиваме единствен m цел број.

Значи дадената равенка има точно 30 решенија.

95. Докажи дека  n  n 1  n  2    


9n  8 за n  0,1, 2, ....

Решение. Со директна проверка за n  0,1,2 се утврдува точноста на горниот идентитет.

За n  3 , користејќи неравенство меѓу аритметичка и геометриска средина за позитивните


броеви n, n  1 и n  2 добиваме дека важи:

n  n 1  n  2  3 3 nn  1n  2  3 3 nn  1n  2 .

3
 8
Ќе докажеме дека за секој n  3 важи nn  1n  2   n   .
 9

3
 1
Нека x  n  1 4 . Ја разгледуваме функцијата f x   x  1xx  1   x   .
 9

116
200 ТЕОРИЈА НА БРОЕВИ - подготвителни задачи

x 2 28
. Добиваме f ' x   x 
2 28
Имаме f x  
1
 x  0 , па затоа f е монотоно
3 27 729 3 27
x 2 28 4 2 28
растечка функција. Оттука f x  
1 1
 x   4  0.
3 27 729 3 27 729
3 3
 1  8
Според тоа n  n  1 n  2    n  1     n   . Сега од неравенство меѓу аритметичка
 9   9
и геометриска средина за броевите n, n 1 и n  2 добиваме:

 8
n  n 1  n  2  33 n  n 1   n  2  3  n    9n  8.
 9

Од неравенство меѓу аритметичка и квадратна средина за позитивните броеви

n, n  1 и n  2 добиваме дека важи:

n  n 1  n  2  3
 n  
2
 
2
n 1  n2 
2

3
3n  3
 9n  9 .
3 3

Значи важи 9n  8   n  n 1  n  2 
2
 9n  9 , од каде следува

 n  n 1  n  2    9n  8 .
96. Канада 1999. Најди ги реалните решенија на равенката 4 x 2  40x  51  0.

Решение. Забележуваме дека 2 x  32 x  17  4 x 2  40 x  51  4 x 2  40x  51  0 .

Бидејќи 2 x  17  2 x  3 и 2 x  32 x  17  0 добиваме дека 2 x  17  0 и 2 x  3  0.

и 1  x  8. Од 4 x 2  40x  51  0 имаме


3 17
Оттука следува x
2 2

40x  51  40x   51 
x ...(1) односно x     ....(2). Со проверка за x 1, 2, ....,8 во
2  2 
(2) добиваме дека за 2, 6, 7 или 8 е исполнет условот на задачата.

29 189 229 269


На крај со заменување во (2) се добиваат решенијата , , и .
2 2 2 2

117
200 ТЕОРИЈА НА БРОЕВИ - подготвителни задачи


 x  2k 
97. ИМО 1968. Нека x е реален број. Докажи дека  k 1 
 x.
k 0  2 

 1
Решение. Ако ставиме n  2 во идентитетот на Хермит имаме x   x    2 x т.е.
 2
 1
 x  2   2 x  x ...(1). Сега ако x го замениме со 2 k 1 во (1) го добиваме равенството
x

 x  2k   x 1   2x   x   x   x 
 k 1    k 1     k 1    k 1    k    k 1 .
 2  2 2 2  2  2  2 


 x  2k     x   x  
Оттука имаме  k 1     k 
    k 1   x, што требаше да се докаже.
k 0  2  k 0   2   2  

98. Индонезија 2008. Нека m, n  1 се природни броеви за кои важи n 4 m  1 и 2 m n  1 .


Докажи дека n  2m  1.

Решение. Од n 4 m  1 следува дека постои природен број a така што 4 m 1  an . Од


2m n  1 постои природен број b така што n  1  2m b . Според тоа важи
  
4 m  an  1  a 2 m b  1  1  2 m ab  a  1 . Јасно, a  1  0 mod 2 m  па затоа a  2 m c  1 , за
  
некој природен број c . Оттука важи 4 m  1  2 m c  1 2 m b  1  4 m bc  2 m c  b  1 т.е.

4 m bc  1  2 m b  c . Равенката 2 m bc 1  b  c при услов m  1 е можна само ако


b  c  1 . Затоа n  2 m b  1  2 m  1 .

99. Докажи дека бројот на членови на низа на Фареј од ред n е 1   1   2  ...   n.

a
Решение. Доказот следува од фактот дека ако е елемент од низата на Фареј тогаш
b
НЗД  a, b   1 , и за секој b, 1  b  n можните вредности на a за кои
a
е елемент од
b
низата на Фареј се точно  b . Оттука вкупниот број членови на низата на Фареј е
еднаква на 1   1   2  ...   n.

118
200 ТЕОРИЈА НА БРОЕВИ - подготвителни задачи

100. С.Ф. Нека a и b се природни броеви за кои важи НЗД  а, 2013  НЗД  b, 2013  1

и a  b. Докажи дека бројот a1200  b1200 има најмалку три прости делители.

 1  1  1
Решение. Бидејќи 2013  3 11  61 имаме   2013  2013  1    1    1    1200,
 3   11   61 

каде   x  е Ојлеровата функција. Од Ојлеровата теорема a n   1mod n имаме



2013 a1200  1. Аналогно и 2013 b1200  1. Сега јасно е дека 2013 a1200  1  b1200  1 ,   
односно 2013 a1200  b1200 . Значи 3,11 и 61 се прости делители на бројот a1200  b1200.

1 1 1
101. Најди го бројот на парови од природни броеви x, y  за кои важи   ,
x y n

каде n е природен број.

1 1 1
Решение. Од   добиваме nx  ny  xy т.е. x  n y  n  n 2 .
x y n

Ако n  1 имаме x  1 y  1  1 од каде се добива единствено решение x, y   2,2.

Нека n  2 и n  p11 p 2 2    p k k е канонична факторизација на бројот n.

Нека 1  d1  d 2  ...  d s  n 2 се сите делители на n 2 . Бидејќи n 2 е полн квадрат следува


дека s е непарен број, и притоа важи d1  d s  d 2  d s 1  ....  d s21  n 2 .
2

Постои еквиваленција помеѓу решенијата x, y  и бројот на делители на n 2 . Според тоа


1 1 1
вкупниот број на парови природни броеви x, y  за кои важи   е еднаков на
x y n
 n 2   21  12 2  1    2 s  1.

102. Предлог ИМО 1993. За природниот број n велиме дека го има својство P ако
важи: Ако n го дели a n  1 за некој цел број a, тогаш и n 2 го дели a n  1 .

а) Докажи дека секој прост број има својство P.

б) Докажи дека постојат бесконечно многу сложени броеви n кои го имат


својството P.

119
200 ТЕОРИЈА НА БРОЕВИ - подготвителни задачи

Решение. а) Нека n  p е прост број и нека p a p  1. Од малата теорема на Ферма


имаме a p 1  1mod p . Следува дека a  a p 1  a  a p  1mod p .

Значи, за секој природен број k важи a k  1mod p .

1  a  a 2  ...  a p 1  p  0 mod p  .

Од p a  1 и p 1  a  a 2  ....  a p 1 имаме p 2 a  11  a  a 2  ...  a p 1 

т.е. p 2 a p  1.

б) Нека n  p1 p 2   p k е производ од k различни прости броеви p i и нека n a n  1.

 n  n
  pi
p 
Јасно pi a 1 т.е. pi a
n  i
 1. Ако замениме bi  a pi добиваме дека

pi bipi  1 , па според а) имаме pi2 bipi  1 .

Значи за секој прост број p i важи pi2 a n  1. Бидејќи НЗД  pi , p j   1, i  j следува дека

p12  p22  pk2 a n  1 т.е. n2 a n  1.

103. УСАМО 1998. Нека претпоставиме дека множеството 1, 2, ....,1998 е поделено на
дисјунктни парови ai , bi  1  i  999 , и ai  bi  1 или ai  bi  6 за секој
i  1, 2,....,999. Докажи дека збирот a1  b1  a2  b2  ...  a999  b999 завршува на 9.

Решение. Бидејќи ai  bi  1 или ai  bi  6 следува дека ai  bi  1mod 5.

Според тоа добиваме s  a1  b1  a2  b2  ...  a999  b999  1  1  ..  1  999  4 mod 5.

За секои цели броеви a и b важи a  b  a  b  a  b mod 2.

Оттука s  a1  b1  a2  b2  ...  a999  b999  a1  b1  a2  b2  ....  a999  b999 

1998  1999
 1  2  .....  1998   1mod 2.
2

120
200 ТЕОРИЈА НА БРОЕВИ - подготвителни задачи

Од 5 s  4 следува дека 5 s  9 . Јасно и 2 s  9 па затоа 10 s  9 .

Значи бројот a1  b1  a2  b2  ...  a999  b999 завршува на цифрата 9.

104. С.Ф. Избран е сложен природен број за прв член на една конечна геометриска
прогресија со количник фиксиран прост делител на првиот член. Докажи дека низата од
бројот на делители на членовите од геометриската прогресија образува аритметичка
прогресија.

Решение. Нека a1  p11  p22  pkk е прв член на геометриската прогресија каде
p1 , p2 ,..., pk се прости делители, и нека pi е фиксиран количник за геометриската
прогресија.

На тој начин ја добиваме низата:

a1  p11  p22  pii  pkk , a2  p11  p22  pi1i  pkk ,...., an  p11  p22  pin1i  pkk .

Новата низа која се состои од бројот на делителите на низата ai i 1 е следна:


b1  1  1 1   2   1   i   1   k  , b2  1  1 1   2    2   i   1   k  ,..


..., bn  1  1 1   2    n   i   1   k  .

b1
Ако ставиме S  добиваме b1  Si  S .
1   i 
На ист начин се добива b2  S  2  i   Si  2S ,..., bn  S  n  i   Si  nS.

Оттука следува дека низата bi i 1 е аритметичка прогресија со разлика d  S .


105. Италија 2012. Нека x1 , x 2 , x3 ,.... е низа дефинирана со следната рекурентна формула

 x1  4

 x n 1  x1 x 2 x3    x n  5, n  1

Најди ги сите парови од природни броеви a, b  така што x a xb е точен квадрат.

121
200 ТЕОРИЈА НА БРОЕВИ - подготвителни задачи

Решение. Имаме x1  4, x2  4  5  9, x3  x1  x2  5  4  9  5  41. Со помош на математичка


индукција лесно се докажува дека ниеден од членовите на низата  xn n 1 не се дели со 5.

Ако претпоставиме дека xn не се дели со 5, за секој 1  n  k тогаш добиваме:

xk 1  x1 x2  xk  5  a1a2  ak  0  a  mod 5 , каде ai 1, 2,3, 4 и a 1, 2,3, 4 .

Нека x a , x b се членови од низата за кои x a xb е полн квадрат. Без губење на општоста


земаме дека a  b. Важи xa  x1 x2  xa 1  5  x1 x2  xb  xa 1  5 , па затоа следува

НЗД xa , xb   НЗД x1 x2  xb  xa 1  5, xb   НЗД xb , 5  1.

Значи x a xb е полн квадрат ако и само ако x a и x b се полни квадрати.

Остатокот при делење на еден полн квадрат со 36 е број од множеството


0,1, 4, 9,13,16, 25, 28.
Важи x2  9, x3  41, ...., xn  36  x4  xn1  5 , па оттука xn  5 mod 36, n  3. Бидејќи 5 не
е остаток при делење на полн квадрат со 36 следува дека n  2. Јасно x1 x2  36  6 2 е
единствено решение.

106. Албанија 2011.

Низата a n  е дефинирана со: a1  1 и an  na1  a2  ...  an1 ,  n  1.

а) Докажи дека за секој парен број n , a n се дели со n !.

б) Најди ги сите непарни броеви n за кои a n се дели со n !.

Решение. Со помош на математичка индукција лесно се докажува дека

11! 2  2! ...  n  n!   n  1! 1, n  2.

n  n!
Сега со помош на математичка индукција ќе докажеме дека a n  ,  n  2.
2

2  2!
Ако n  2 тогаш a 2  2a1  2  .
2

k  k!
Нека претпоставиме дека за секој k , 2  k  n  1 важи a k  .
2

122
200 ТЕОРИЈА НА БРОЕВИ - подготвителни задачи

 11! 2  2!  n  1!  n  1  1   n  n ! .
Имаме: an  n  a1  a2  ...  an1   n    ...  
 2 2 2 2 2

2k  2k !
а) Ако n  2k имаме a 2 k   k  2k !, па оттука 2k ! a 2k .
2

б) Ако n  2k  1 имаме a 2 k 1 
2k  1  2k  1!  2k  1  2k  1!. Бидејќи
2k  1
не е цел
2 2 2
број следува дека a 2 k 1 не се дели со 2k  1!. Останува дека за n  1 условот е исполнет.

107. Предлог ИМО 1992. Нека  n  е бројот на единици во бинарната репрезентација на


природниот број n. Докажи дека:

  n    n   1
а) Важи неравенството  n2    2
.

б) Горното неравенство преминува во равенство за бесконечно многу природни


броеви n.

Решение. а) Нека  n   k . Значи n  2 a1  2 a2  ...  2 ak , каде a i е позицијата (од десно


налево) на секоја единица во бинарната репрезентација на бројот n.

Важи n 2  2 2a1  2 2a2  ...  2 2ak  2 a1 a2 1  2 a1  a3 1  ....  2 ak 1 ak 1.

a1  a2  1, a1  a3  1,...., a2  a3  1, a2  a4  1, ......., ak 1  ak  1 се вкупно


 k  1 k броеви.
2

Бидејќи помеѓу броевите

2a1 , 2a2 , 2a3 ,......,2ak , a1  a2  1, a1  a3  1,...., a2  a3  1, a2  a4  1, ......., ak 1  ak  1

k  1k  k k  1
има најмногу k  различни броеви следува дека
2 2

k k  1  n  n   1
 n 2    .
2 2

б) Нека избериме n  2k , k  N . Бидејќи 2k10  100..0 во бинарен броен систем, следува


k  нули

дека   2 k
  1.

123
200 ТЕОРИЈА НА БРОЕВИ - подготвителни задачи

108. ИМО 2005. Ја разгледуваме низата a1 , a 2 , ... дефинирана со a n  2 n  3 n  6 n  1 за


секој природен број n. Определи ги сите природни броеви кои се заемно прости со секој
член од низата.

Решение. Ќе докажеме дека секој прост број p дели некој член a n на низата.

За n  2 имаме a2  2 2  32  6 2  1  48 , па оттука p a 2 кога p  2 или p  3.

Нека p5 е прост број. Од малата теорема на Ферма следува дека


2 p 1
3 p 1
6 p 1
 1mod p .

 
Тогаш добиваме дека 6a p 2  6 2 p 2  3 p 2  6 p 2  1  3  2 p 1  2  3 p 1  6 p 1  6

од каде следува 6a p 2  3  2  1  6  0 mod p  т.е. p 6a p 2 .

Бидејќи НЗД  p, 6  1 следува дека p a p  2 . Останува дека 1 е единствениот природен


број кој е заемно прост со секој член на дадената низа.

109. Предлог ИМО 1997. Нека p е прост број и нека f x  е полином од степен d со
целобројни коефициенти така што:

1) f 0  0, f 1  1.

2) за секој природен број n , остатокот при делење на f n  со p е 0 или 1 .

Докажи дека d  p  1.

Решение. Лема: За секој полином P од степен n важи следниот идентитет:


n 1
 n  1
  1 
i
Pi   0.
i 0  i 

Нека претпоставиме спротивно, т.е. нека степенот d на полиномот f е најмногу p  2.

p 1
 p  1  p  1
Ако во горната лема замениме n  p  2 добиваме   1 
i
 f i   0. Јасно,   е
i 0  i   i 
 p  1  p  1 p  2  ... p  i   1 2  ...   i   1 i !
i

цел број и        1 mod p 


i

 i  i ! i ! i !

124
200 ТЕОРИЈА НА БРОЕВИ - подготвителни задачи

i  p  1
p 1 p 1 p 1
па според тоа имаме 0    1   f  i     1 f  i   f  i  mod p . ..(1).
2i

i 0  i  i 0 i 0

Од 2) имаме f i   0 mod p  или f i   1mod p  , за секој i  0,1,..., p  1.

Бидејќи f 1  1 и поради (1) имаме дека мора f  i   1 mod p  за секој i 0,1,..., p  1

т.е. и f  0   0  1 mod p  , што не е можно.

110. Нека p i е i - тиот прост број. Докажи дека p1k  p 2k  ...  p nk  n k 1 , за секој пар од
природни броеви n, k .

Решение. За секој природен број i  2 важи pi 1  pi  2.

Според тоа добиваме

pn  2   pn  pn1    pn1  pn2   ....   p3  p2    p2  p1   2  2  ...  2  1  2  n  2   1

т.е. pn  2n  1.

Од неравенството меѓу средини од ред k и ред 1 за броевите p1 , p 2 ,..., p n имаме:

p k  p 2k  ....  p nk  p1  p 2  ...  p n 
k
p1k  p 2k  ....  p nk p  p 2  ...  p n
k  1 т.е. 1   .
n n n  n 

 2 1  2  ...  n   n 
k
 p  p2  ...  pn 
k

Добиваме p1  p2  ....  pn  n   1  n  


k k k

 n   n 

 2n  n  1 
k

 n  1  n  n k  n k 1.
 2 n 

111. ММО 2007. Природните броеви a, b и c се попарно различни за кои важи

a b  c  bc, b c  a  ca, c a  b  ab.

Докажи дека најмалку еден од броевите a, b, c не е прост број.

Решение. Нека претпоставиме дека a, b, c се прости броеви.

125
200 ТЕОРИЈА НА БРОЕВИ - подготвителни задачи

Без губење на општоста земаме дека a  b  c . Важи a  3 бидејќи ако a  2 тогаш


b  c  bc е непарен број кој се дели со два, што не е можно. Значи a  3 , b  5 и c  7.

Од a b  c  bc следува дека a a  b  c  ab  bc  ca  abc.

Аналогно важи и b a  b  c  ab  bc  ca  abc и c a  b  c  ab  bc  ca  abc .

Бидејќи a  b  c  ab  bc  ca  abc  a  1b  1c  1  1 и a, b, c се различни прости


броеви следува дека abc a  1b  1c  1  1 .

Значи,

1
a  1b  1c  1  1  a  1b  1c  1  a  1  b  1  c  1  1  1 1  1 1  1  
   
abc abc a b c  a  b  c
 1  1  1  4 6 8 64
 1  1  1        2.
 3  5  7  3 5 7 35

Последното не е можно, па според тоа следува дека барем еден од броевите a, b, c не е


прост број.

112. ИМО 2001. Нека a  b  c  d се природни броеви за кои важи

ac  bd  b  d  a  c b  d  a  c .

Докажи дека ab  cd не е прост број.

Решение. Од ac  bd  b  d  a  c b  d  a  c  добиваме ac  bd  a 2  d 2  c 2  b 2

т.е. a 2  ac  c 2  b 2  bd  d 2 .

Според тоа имаме:

ab  cd ad  bc  acb 2  bd  d 2   bd a 2  ac  c 2   ac  bd b 2  bd  d 2 .


Јасно ac  bd  ab  cd ad  bc. Нека ab  cd е прост број. Од условот на задачата
a  b  c  d следува ab  cd  ac  bd  ad  bc. Оттука НЗД ac  bd , ab  cd   1.

Добиваме дека ac  bd  ad  bc  , што не е можно бидејќи ac  bd  ad  bc.

126
200 ТЕОРИЈА НА БРОЕВИ - подготвителни задачи

113. САД 2003. Докажи дека за секој природен број n постои n  цифрен број делив со
5 n и притоа сите негови цифри се непарни.

Решение. Ќе докажеме со помош на математичка индукција. Јасно е дека тврдењето важи


за n  1. Нека N  a1a2 ...an е делив со 5n и се состои само од непарни цифри.

Да ги разгледаме броевите

N1  1a1...an  110n  5n m  5n 1 2n  m  ,

N2  3a1...an  3 10n  5n m  5n  3  2n  m  ,

N3  5a1...an  5 10n  5n m  5n  5  2n  m  ,

N4  7a1...an  7 10n  5n m  5n  7  2n  m  ,

N5  9a1...an  9 10n  5n m  5n  9  2n  m  .

Броевите 1 2n  m,3  2n  m,5  2n  m,7  2n  m и 9  2n  m дават различни остатоци при


делење со 5. Во спротивно разликата на некои два од нив би се делела со 5, што не е
можно. Според ова имаме дека барем еден од овие броеви е делив со 5 т.е. барем еден од
броевите N1 , N2 ,..., N5 е делив со 5n1 и јасно се состои само од непарни цифри. Со ова
задачата е решена.

114. Докажи дека за секој k  N , постои цел број m така што ниеден од броевите

m  1, m  2, ...., m  k не е слободен квадрат.

Решение. За природниот број n велиме дека е слободен квадрат ако не се дели со


квадрат на некој прост број.

Избираме k различни прости броеви p1 , p 2 , ..., p k .

 x  1 mod p12 

 x  2 mod p 2 
2
Го разгледуваме ситемот конгруентни равенки 
 ......
 x  k mod p 2 
 k .

127
200 ТЕОРИЈА НА БРОЕВИ - подготвителни задачи

Од кинеската теорема за остатоци следува дека системот има единствено решение по


модул  p1 p 2    p k  .
2

Од системот конгруентни равенки следува дека p12 x  1, p 22 x  2, .... p k2 x  k

односно броевите x  1, x  2, ...., x  k се делат со квадрат на прост број, па оттука следува


дека ниеден од нив не е слободен квадрат.

115. Докажи дека ако постојат бесконечно многу ненегативни цели броеви n така што
2n  1 и 3n  1 се полни квадрати, тогаш n се дели со 40.

Решение. Нека постојат природни броеви a и b така што 2n  1  a 2 и 3n  1  b 2 .


Следува a 2  b 2  2 mod 5. Бидејќи за секој цел број x важи

 0  mod 5  , x  5k ,

x   1 mod 5  , x  5k  1,5k  4,
2


 4  mod 5  , x  5k  2, x  5k  3.

следува дека a 2  1  b2  mod 5 . Јасно 2n  0 mod 5 т.е. 5 n.


Од друга страна важи 3a 2  2b 2  1 . Нека ставиме a  u  2v и b  u  3v , и последната
равенка добива облик на равенка на Пел u 2  6v2  1 која има бесконечно многу решенија
во множеството на природни броеви. Бидејќи a 2  2n  1 следува a е непарен природен
број, па земаме a  2k  1.
Оттука n  2k k  1 . Од b 2  3n  1 добиваме дека и b е непарен број, па земаме
b  4s  1. Според тоа 3n  4s  12  1  16s 2  8s, од каде следува дека 8 n.

Бидејќи 5 n и 8 n следува дека 40 n.

116. Најди ги сите тројки од последователни природни броеви кои се страни на


триаголник чија плоштина е природен број.

Решение. Нека a  1, a и a  1 се должини на страните. Од Хероновата формула лесно се

пресметува дека P 

a 3 a2  4 
. Бидејќи P е цел број јасно е дека a мора да биде
4
 
парен број и 3 a 2  4 полн квадрат, па според тоа постојат цели броеви m и n така што

128
200 ТЕОРИЈА НА БРОЕВИ - подготвителни задачи

a  2m, a 2  4  3n 2 . Сега важи 4m 2  4  3n 2 , па оттука n  2k. Последнта равенка се


сведува на равенка на Пел m 2  3k 2  1 чие почетно решение е m, k   2, 1.

Според тоа сите решенија на равенката се ms 


2  3   2  3 
s s

и
2

 2  3    2  3  . Значи, страните на триаголникот се 2m


s s

ks  s  1, 2ms и 2ms  1.
2 2

117. За низата an   n2   n  1  докажи дека постојат бесконечно многу природни


2

 
броеви n така што n2   n  1 е полн квадрат, an  an1  1 и an1  an  1.
2

Решение. Нека n 2  n  12  m 2 . Важи 4n 2  4n  2  2m 2 т.е. 2n  12  2m 2  1.


Ако земеме x  2n  1 ја добиваме равенката од Пелов тип x 2  2m 2  1 , која има
почетно решение  x, m  1,1 . Оттука x 2  2m 2  1 има бесконечно решенија
природни броеви и секој x е непарен број. Важи a n  m како и

an1    n  1  n2    2n2  2n  1    2n2  2n  1  4n    m2  4n  .


2
       

Од n 2  n  12  m 2 следува дека n  2 и an1  m 2



 4n  m 2  4n  m  1  an  1.

Значи an  an1  1 за секој n. Исто така важи an1   n  1  n  2  



2 2

m 2
 4n  4 
и бидејќи m  1  m 2  4n  4  m  2 имаме an1  m  1. Значи
an1  an  m  1  m  1.

118. Виетнам 1985. Во множеството цели броеви реши ја равенката

x 3  y 3  2 xy  8.

Решение. Ако x  0 добиваме y  2. Ако y  0 имаме x  2.

Сега нека x, y  0 . Разгледуваме два случаи:

1) Нека xy  0. Ако x  0, y  0 имаме x 3  y 3  2 xy  8  8, од каде следува дека x  1.

За x  1 ја добиваме кубната равенка y3  2 y  7  0 која нема решение цел број.

129
200 ТЕОРИЈА НА БРОЕВИ - подготвителни задачи

Ако x  0, y  0 имаме y 3  x 3  2 xy  8  2 xy.

Од друга страна важи y 3  x 3  y 3   x   y 2   x   2 xy.


3 2

Добиваме противречност.

 
2) Нека xy  0. Од x 3  y 3  2 xy  8  0 и бидејќи x 3  y 3  x  y  x  y   3xy следува
2

дека x  y  0.

Ако x  y  1 , заменуваме x 1  y во x 3  y 3  2 xy  8 и ја добиваме квадратната


равенка y 2  y  7  0 која нема решение цел број.

 
Ако x  y  2 имаме: 2 xy  8  x 3  y 3  x  y  x  y   3xy  24  3xy   8  6 xy.
2

Од последното следува xy  0 , што е спротивно со xy  0.

Значи  0,  2  ,  2,0  се единствени решенија во множеството цели броеви.

119. Виетнам 2003. Најди го најголемиот природен број n така што равенката

x  12  y12  x  22  y22  ....  x  n2  yn2


има решение  x, y1 , y2 ,..., yn  во множеството цели броеви.

Решение. Ќе докажеме дека дадената равенка нема решение во множеството цели броеви
ако n  4 , па според тоа ќе нема решение и за секој природен број n  4. Ако m е цел
број тогаш важи

 1 mod 8, m  4k  1, m  4k  3,

m   0 mod 8, m  4k ,
2

 4 mod 8, m  4k  2.

Оттука следува дека за два цели броеви m и l важи

 2,1, 5 mod 8, m  4k  1, m  4k  3,



m  l  1, 0, 4 mod 8, m  4k ,
2 2

 5, 4, 0 mod 8, m  4k  2.

Нека претпоставиме дека постојат цели броеви x, y1 , y 2 , y3 и y 4 за кои важи

130
200 ТЕОРИЈА НА БРОЕВИ - подготвителни задачи

x  12  y12  x  22  y22  x  32  y32  x  42  y42 .

Постои s  N така што  x  1  y12   x  2   y22   x  3  y32   x  4   y42  s  mod8 .


2 2 2 2

Бидејќи броевите x  1, x  2, x  3 и x  4 имаат различни остатоци при делење со 4

следува дека нивните квадрати ќе имат остатоци 0,1, 4 при делење со 8.

Оттука s 2,1,5  1,0, 4  5, 4,0   , што не е можно.

Кога n  3 лесно се гледа дека  2, 0,1, 0 е решение на равенката. Значи nmax  3.

120. Предлог ИМО 2004. Нека k е фиксиран природен број поголем од 1 , и нека
m  4k 2  5. Докажи дека постојат природни броеви a и b така што сите членови на
 xn n1

низата дефинирана со x0  a, x1  b, xn 2  xn1  xn за n  0,1, 2, ... се заемно
прости со m.

Решение. Најпрво ќе докажеме дека постои природен број b така што b2  b  1 mod m  .
Бидејќи НЗД  4, 4k 2  m   1 следува дека b2  b  1  4b2  4b  4  mod m 

т.е. b2  b  1 mod m    2b  1  5  mod m  . Ако 2b  1  2k тогаш


2

4k 2  5  mod m   4k 2  2k  4  2k  1 mod m  
 
2 2k 2  k  2  2k  1  2b  mod m   b  2k 2  k  2  mod m  .

Сега, нека a  1 и b е природен број таков што b 2  b  1mod m.

Со помош на математичка индукција ќе докажеме дека xn  b n mod m.

x1  b  b1 mod m, x2  x0  x1  b  1  b 2 mod m.

Нека претпоставиме дека xk  b k mod m и x k 1  b k 1 mod m .

Тогаш xk  2  xk 1  xk  b k 1  b k  b k b  1  b k  b 2  b k  2 mod m.

Значи за секој природен број n важи xn  b n mod m. Ќе докажеме дека НЗД b, m  1.
Ако НЗД b, m  d  1 тогаш од m b 2  b  1 следува дека d b 2  b  1 и бидејќи d b

131
200 ТЕОРИЈА НА БРОЕВИ - подготвителни задачи

следува d 1. Сега нека НЗД xn , m  k  1. Од x n  b n mod m следува m x n  b n т.е.


k x n  b n . Бидејќи k x n следува дека k b. Но тогаш k НЗД m, b , што не е можно
бидејќи m и b се заемно прости броеви. Останува дека НЗД  xn , m   1.

n 3
121. Докажи дека n го дели бројот N   r  r ! ако и само ако n е прост број.
r 1

Решение. Од r  r ! r  1!r ! добиваме:

N  1  1!2  2!...  n  3  n  3! 2!1!  3!2!  ....  n  2!n  3!  n  2!1.

Ако овој идентитет го помножиме со n  1 добиваме n  1N  n  1 !n  1.

Од теорема на Вилсон следува дека n  1! 1mod n ако и само ако n е прост број.

Според тоа n  1N  0 mod n ако и само ако n е прост број.

Бидејќи НЗД n  1, n  1 следува дека n N ако и само ако n е прост број.

122. Докажи дека секој прост број од облик p  4k  1 може да се претстави како збир од
квадрати на два природни броја.

Решение. Од теорема на Вилсон имаме  p  1! 1mod p  , па следува дека

2
 p  1  p  1 p  1  p  1  
2
p 1

 1  1   p  1  2   p  2      p     1 2  1  2        ! .
 2   2   2   2  

 p  1 p 1
Притоа користевме дека p  i  i mod p , i  1, 2,....,  , како и  1 2   1  1.
2k

 2 

 p 1
Според тоа конгруенцијата x 2  1mod p  има решение x0    !.
 2 

Од лема на Туе следува дека постојат природни броеви a, b  p така што ax0  b или
ax0  b се дели со p. Значи ax0  b  mod p  .

По квадрирање на конгруенцијата добиваме a 2 x02  b 2 mod p  , и бидејќи x02  1mod p 


следува дека a 2  b 2 mod p  т.е. p a  b .
2 2

132
200 ТЕОРИЈА НА БРОЕВИ - подготвителни задачи

Бидејќи 0  a 2  b 2  2  p 2
 2 p и p a  b следува дека p  a 2  b 2 .
2 2

123. Предлог ИМО 2010. Најдете го најмалиот природен број n за кој постои множество

s1 , s 2 , ...., s n  кое се состои од n различни природни броеви така што важи

 1  1  1 51
1  1      1    .
 s1  s 2   s n  2010

Решение. Без губење на општоста земаме дека s1  s 2  ...  s n . Јасно е дека s1  2 бидејќи
1
ако s1  1 ќе имаме 1   0 , што не е можно. Значи 2  s1 , 3  s 2 , ...., n  1  s n .
s1

Според тоа имаме:

51  1  1  1   1  1   1  1 2 n 1
 1  1      1    1  1      1      
2010  s1  s 2   s n   2  3   n  1  2 3 n  1 n  1

2010
Значи n  1   39, т.е. n  39.
51

За n  39 го разгледуваме множеството s1 , s2 ,..., s39   2,3,...,33,35,..., 40, 67.

 1  1  1  1 2 32 34 39 66 17 51
Имаме 1  1      1              .
 s1  s 2   s n  2 3 33 35 40 67 670 2010

Значи n  39 е најмалиот природен број кој ги задоволува условите на задачата.

124. Претстави го бројот 459 како збир од квадрати на четири природни броеви.

Решение. Ќе ja користиме следното својство:

Својство. Ако природните броеви a и b можат да се претстават како збир од квадрати


на четири природни броја, тогаш и нивниот производ, бројот ab може да се претстави
како збир од квадрати на четири природни броеви.

Доказ: Нека a  a12  a22  a32  a42 и b  b12  b22  b32  b42 , каде a1 , a2 , a3 , a4 , b1 , b2 , b3 , b4 се
природни броеви. Од Ојлеровиот идентитет имаме:

133
200 ТЕОРИЈА НА БРОЕВИ - подготвителни задачи

  
ab  a12  a 22  a32  a 42 b12  b22  b32  b42  a1b1  a 2 b2  a3 b3  a 4 b4  
2

 a1b2  a 2 b1  a3 b4  a 4 b3   a1b3  a 2 b4  a3 b1  a 4 b2   a1b4  a 2 b3  a3 b2  a 4 b1  .


2 2 2

Според тоа добиваме


459  33  17  3 2  3  17  3 2 12  12  12  0 2 4 2  12  0 2  0 2   
3 2
4  1  0  0 2
 1  4  0  0  0  0  4  0  0  0  1  0 
2 2 2

2
 2 2 2 2

 3 5  3  4  1  3  5  3  3  3  4  3  1  15  9  12 2  3 2.
2 2 2 2 2 2

125. Докажи дека секој цел број може да биде претставен како збир од кубови на пет цели
броеви, не- неопходно различни.

Решение. За секој цел број m важи m  1  m  1  m 3  m 3  6m .


3 3

Бидејќи n 3  n  n  1nn  1 е производ на три последователни броеви следува дека


n3  n
6 n 3  n т.е. е цел број.
6

n3  n
Ако замениме m  во горниот идентитет добиваме
6
3 3 3 3
 n3  n   n3  1   n3  n   n3  n 
n 3  n    1    1      
 6   6   6   6 
3 3 3 3
 n3  n   n3  1   n3  n   n3  n 
т.е. n     1     1        n 3 .
 6   6   6   6 

126. БМО 2001. Нека a, b, n се природни броеви такви што 2 n  1  ab. Нека k  N е
таков што ab  a  b  1 се дели со 2k и ab  a  b  1 не се дели со 2k 1. Докажи дека k
е парен број.

2n  1
Решение. Од 2 n 1  ab имаме a  , па според тоа важи:
b

ab  a  b  1  2 n  1 
2n  1
 b 1

2 n  b  1 b  1 
 0 mod 2 k .  
b b

134
200 ТЕОРИЈА НА БРОЕВИ - подготвителни задачи

Нека s е највисокиот степен на 2 за кој важи 2 s b  1 , и t е највисокиот степен на 2

така што 2 t 2 n  b  1. Јасно е дека 2  2 , па според тоа t  n и 2 t 2 n .


t n

Значи t е највисокиот степен на 2 за кој 2t го дели b  1 , односно t  s.

Оттука 2 s е највисокиот степен на 2 така што 2 2 s 2 n  b  1 b  1.  


 
Бидејќи k е највисокиот степен на 2 за кој 2 k 2 n  b  1 b  1 , следува дека k  2s.

127. ММО 2011. Најди ги сите природни броеви така што секој природен број запишан со
n  1 единица и една седмица е прост број.

Решение. Нека бројот 11....11711...11 е еден од бараните броеви, и притоа нека после
цифрата седум има k  1 единици, каде k 1, 2, ...., n.

Имаме a  11....11711...11  10 n1  10 n2  ....  10 k  7  10 k 1  ...  10  1 

10 n  1 10 n  54  10 k 1  1
 10 n 1  10 n  2  ....  10  1  6  10 k 1   6  10 k 1  .
9 9

10n  1
Ако 3 n  3 тогаш 10n  103k  103   1k  1 mod 27  , па затоа 27 10n  1 т.е. 3
k
.
9

Во овој случај бројот a е делив со 3.

Нека n  6. Тогаш n  r  1, 2, 4,5  mod 6  . Ако n  3  mod 6  тогаш 3 n.

Бидејќи 106  1 mod 7  добиваме дека 10n  106k r  10r  t  3, 2, 4,5  mod 7  .

Оттука добиваме 10n  54 10k 1  1  t  2 10k 1 1 mod 7  . Секогаш можеме да најдеме
k 1, 2..., n така што 2 10k 1  t  1 mod 7  бидејќи 10m  1, 2,3, 4,5,6  mod 7  .

10n  54 10k 1  1
Тогаш 7  7 односно a не е прост број. Сега лесно се проверува за
9
n 1, 2,3, 4,5 дека условот на задачата го исполнува n  1, бројот 7 , и n  2, броевите
17 и 71.

135
200 ТЕОРИЈА НА БРОЕВИ - подготвителни задачи

128. Русија 2005. Определи го најмалиот природен број кој неможе да се претстави во
2 a  2b
облик , каде a, b, c, d се природни броеви.
2c  2 d

2k  2  2k 1
Решение. Бидејќи 2k  јасно е дека секој број од облик 2k може да се
22  2
претстави во дадениот облик. Ако природниот број n може да се претстави во облик
2 a  2b
тогаш истото важи и за бројот 2k n.
2c  2 d

 2k  2  2k 1   2a  2b  2k  a  2  2k  a 1  2k b  2  2k 1b 2k  a  2k b
Имаме 2k n    c d 
  c .
 2 2   2 2 
2

2 2c  2 d  2  2d

23  2 25  2 27  2 27  2
Важи 3  , 5 3 ,7  4 и 9 4 . Оттука и броевите 6  21  3,10  21  5
2 2
2
2 2 2 2 2 2
2a  2b
може да се запишат во обликот . Ќе докажеме дека 11 неможе да се претстави во
2c  2 d
2a  2b
дадениот облик.Нека претпоставиме спротивно, т.е. нека 11  за некои природни
2c  2 d
броеви a, b, c и d . Јасно е дека a  b и c  d . Имаме 2b  2a b  1  11 2d  2c d  1 . ..(1).

Јасно е дека b  d , па оттука следува дека 11 2a b  1. Бидејќи 210  1 mod11 следува дека

a  b  10k , за некој природен број k . Равенката (1) добива облик 2ab  10  11 2cd .

Ако c  d  2 тогаш 4 11 2c d , но  2a b  10  не се дели со 4. Останува c  d  1.

На крај добиваме дека 2a b  22  10  12, што не е можно. Значи бројот 11 е најмалиот


2a  2b
природен број кој неможе да се претстави во облик .
2c  2 d

  n 
 n n   
129. Докажи дека множеството A        n  1   содржи
, ,..., непарен број на
      2  
1 2
 
непарни броеви, ( n е непарен број).

Решение. Доволно е да докажеме дека збирот на елементите на множеството A е


непарен.

136
200 ТЕОРИЈА НА БРОЕВИ - подготвителни задачи

Тогаш јасно е дека множеството A содржи непарен број на непарни броеви.

 n  n  1
Нека S n е збирот на биномните коефициенти  , i  1, 2, ..., .
 
i  2 

n  n   n  1
Бидејќи важи      , за секој k  1, 2,..,  добиваме
k  n  k   2 

 n   n 
n n    n   n   
S n        ...   n  1         ...   n  1  .
   
1 2    n  1   n  2   
 2   2 

Оттука важи

 n   n 
n n  n   n 
2Sn        ...   n  1    n  1   ...    
 1  2       n  2   n  1
 2   2 

  n   n  
n n n  n   n   n  n  n
           ...   n  1    n  1   ....   
             2  2.
 n
  0  1   2  
 
   
            
 n 2 n 1 n 0 n
  2   2  

Значи S n  2 n1  1 е непарен број.

 n  n  n
Забелешка: Ја користивме формулата       ...     2n.
 0  1   n

130. ИМО 1967. Нека k, m и n се природни броеви така што m  k  1 е прост број
поголем од n  1. Нека c s  ss  1. Докажи дека бројот cm1  ck cm 2  ck     cm n c k 
се дели со c1c 2    c n .

Решение. Ќе користиме дека cr  c s  r r  1  ss  1  r 2  s 2  r  s  r  s r  s  1.

cm1  ck cm 2  ck     cm n c k  m  k  n! m  k  n  1!


Имаме   .
c1c 2    c k m  k !n ! m  k  1!n  1!
m  k  n!   m  k  n 
Јасно е природен број.
m  k !n !  n 

137
200 ТЕОРИЈА НА БРОЕВИ - подготвителни задачи

Исто така важи


m  k  n  1!  m  k  n  1! 
1  m  k  n  1
  .
m  k  1!n  1! m  k  1m  k !n  1! m  k  1  n  1 

 m  k  n  1
Бидејќи   е природен број следува дека m  k !n  1! m  k  n  1!.
 n  1 

Јасно е дека m  k  1 m  k  n  1!. Бидејќи m  k  1 е прост број поголем од n  1


следува дека броевите m  k  1 и m  k !n  1! се заемно прости.

Според тоа имаме m  k  1m  k !n  1! m  k  n  1! т.е.


m  k  n  1! е
m  k  1!n  1!
природен број.

131. Нека p е прост број. Докажи дека ако постои цел број x за кој p x 2  x  3 тогаш
постои цел број y за кој p y 2  y  25.

Решение. Случајот кога p  3 е тривијален. Нека p  5.


Ако p x 2  x  3, тогаш p 4 x 2  x  3 т.е. p  2x  1 2

 11 .

Според тоа постои цел број x за кој важи конгруенцијата 2 x  1  11mod p .


2

  11   99   9  11   11   99 


Значи    1. Оттука         1. Бидејќи    1 следува дека
 p   p   p  p   p   p 
конгруенцијата z 2  99  mod p  има решение. Од  2 x  1  11 mod p  добиваме дека
2

3 2 x 1  99  mod p  . Јасно е дека z е непарен број, па земаме z  2 y  1.


2

Добиваме p  2 y 1  99 , p 4y


2 2

 y  25 т.е. p y 2  y  25.

132. (ТСТ- Индија). Докажи дека за секој природен број n , n 7  7 не е полн квадрат.

Решение. Ќе докажеме дека равенката n 7  7  k 2 нема решение во множеството


природни броеви.

Дадената равенка е еквивалентна со равенката n 7  128  k 2  121

 
т.е. n 7  128  n 7  27  n  2 n 6  2n5  4n 4  ....  64  k 2  112 .

138
200 ТЕОРИЈА НА БРОЕВИ - подготвителни задачи

Важи a  n6  2n5  4n4  ...  64  n5  n  2  mod 4  . Според тоа важи a  0,3,0,3  mod 4  за
n  4s, 4s  1, 4s 2, 4s 3, соодветно. Ако сите прости делители на еден број се од облик
4t  1 тогаш и бројот е од таков облик. Значи n 7  128 има прост делител p од облик
4k  3. Важи k  11 mod p  , односно  121 е квадратен остаток на
2 2
p. Оттука
  11 
2
  11   11   1    1 
2 2 p 1 p 1
   1 . Имаме 1             1 2 . Добиваме дека  2m
 p   p   p  p   p  2
т.е. p  1mod 4, што е спротивно со p  3 mod 4.

p 1
133. Докажи дека ако p  2 n  1, n  2, е прост број, тогаш 3 2
 1 се дели со p.

Решение. Бидејќи 2 n  1   1n  1mod 3 јасно е дека n е парен број, во спротивно p ќе


p 1
3
се дели со 3 , што не е можно. Од Ојлеров критериум имаме    3 2
mod p  .
 p

Важи p  2 n  1  2 2k  1  4 k  1  1mod 4. Исто така p  4 k  1  2 mod 3 па според тоа


 p
конгруенцијата x 2  p mod 3 нема решение, односно важи    1. Од правило на
3
 3  p p 1
 p
квадратни реципрочности добиваме        1 2   1  1 и бидејки    1
2k

   
p 3 3
p 1 p 1
3 3
добиваме дека    1. Бидејќи    3 2 mod p  следува дека 3 2  1mod p 
 p  p
p 1

т.е. p 3 2
 1.

134. БМО 2009. Реши ја равенката 3 x  5 y  z 2 во множеството природни броеви.

Решение. Имаме 3 x  z 2  5 y . Бидејќи секој полн квадрат при делење со 3 има остаток 0
или 1 и бидејќи z 2  5 y  mod 3 следува дека 5 y  1mod 3 т.е y  2k .

  
Според тоа 3 x  z 2  5 y  z 2  52k  z  5 k z  5k . Постојат природни броеви a  b така
што z 5  3
k a
и z  5  3 . Со одземање на последните две равенки добиваме
k b

3  3  2  5 . Значи 3a  3ba  1  2  5k. Бидејќи 2  5k не се дели со 3 следува a  0 .


b a k

Оттука z  1  5k и 3b  2  5k  1. ..(1). Ако ја разгледаме р-ката (1) по модул 5 добиваме

139
200 ТЕОРИЈА НА БРОЕВИ - подготвителни задачи

дека 3b  1 mod 5 т.е. b  4s, додека по модул 4 следува 3b  3  mod 4  , односно b е


непарен број. Следува противречност, па оттука дадената равенка нема решение.

135. Чешка 2009. Природниот број n го нарекуваме добар ако и само ако постојат точно
четири природни броеви k1 , k 2 , k 3 , k 4 така што n  k i n  k i2 1  i  4 .

Докажи дека

а) 58 е добар број.

б) 2 p е добар ако и само ако p и 2 p  1 се прости броеви  p  2 .

Решение. Треба да важи n  x n  x 2 за точно четири природни броеви x.

   
Бидејќи n  x n 2  x 2 следува дека n  x n  x 2  n 2  x 2 т.е. n  x n 2  n .

Значи, бројот n е добар ако и само ако бројот n 2  n има точно четири делители
поголеми од n .

а) Имаме 582  58  58  59  2  29  59 . Јасно, 59, 2  59, 29  59 и 58  59 се точно четири


природни делители на 58 2  58 поголеми од 58 . Според тоа бројот 58 е добар.

б) Нека 2 p е добар број. Тогаш бројот  2 p   2 p има четири делители поголеми од 2 p.


2

2 p  2 p  2 p  2 p  1 следува дека 2 p  1, 2  2 p  1 , p  2 p  1 и 2 p  2 p  1 се
2
Бидејќи
четирите делители на  2 p   2 p поголеми од 2 p.
2

Нека p е сложен број. Постои природен број a така што a p.

Тогаш 2a  2 p  1 2 p  2 p  1 и 2a  2 p  1  2 p. Оттука 2 p
2
 2 p има пет природни
делители т.е. 2 p не е добар број. Значи p е прост број.

Сега нека 2 p  1 е сложен број. Постои природен број a така што a  2 p  1 .

Тогаш 2 pa 2 p  2 p  1 и 2 pa  2 p. Повторно добиваме контрадикција .

Ако p и 2 p  1 се прости броеви тогаш јасно е дека 2 p  1, 2  2 p  1 , p  2 p  1 и


2 p  2 p  1 се единствени делители на бројот  2 p   2 p поголеми од 2 p.
2

140
200 ТЕОРИЈА НА БРОЕВИ - подготвителни задачи

Оттука 2 p е добар број.

136. Докажи дека равенката a 2  b3  c4 има бесконечно многу решенија во множеството


природни броеви.

 n  n  1 
2

Решение. Ако го искористиме идентитетот 1  2  ...  n   3 3


 добиваме
3

 2 

  n  1 n   n  n  1 
2 2

 n   , за n  1. Доволно е да докажеме дека постојат бесконечно


3

 2   2 
n  n  1
многу природни броеви така што  k 2 , за некој природен број k .
2
  n  1 n 
Во тој случај ќе добиеме  a, b, c    , n, k  .
 2 

n  n  1
 k 2 следува n  n  1  2k 2  4n2  4n  1  8k 2  1  2n  1  2  2k   1.
2 2
Од
2

Равенката на Пел x 2  2 y 2  1 има бесконечно многу решенија во множеството природни


броеви. Јасно е дека за секое решение  x, y  , x  2 y 2  1 е непарен број, x  2m  1.
Според тоа важи y 2  2m2  2m  2m  m  1 , т.е. y е парен број. Значи парот  x, y  е од
n  n  1
облик  2n  1, 2k  . Според тоа постојат бесконечно многу n така што  k 2.
2

137. ИМО 2002. Нека n  2 е природен број , со делители 1  d1  d2  ...  dk  n.

Докажи дека d1d2  d2 d3  ...  dk 1dk  n2 , и определи кога d1d2  d2 d3  ...  dk 1dk n2 .

Решение. Важи d1dk  d2 dk 1  ...  n. Неравенството кое треба да го докажеме го добива


следниот облик:

n2 n2 n2 1 1 1
  ...   n2    ...   1.
d k d k 1 d k 1d k 2 d 2 d1 d k d k 1 d k 1d k 2 d 2 d1

1 1 1
Важи   , i  1,.., k  1 бидејќи di 1  di .
di di 1 di di 1

141
200 ТЕОРИЈА НА БРОЕВИ - подготвителни задачи

1 1 1  1 1   1 1  1 1  1 1 1
  ...        ...        1   1.
d k d k 1 d k 1d k 2 d 2 d1  d k 1 d k   d k 2 d k 1   1
d d 2  d1 d k n

n2 n2 n2 n
Нека S  d1d2  d2 d3  ...  dk 1dk и S n 2 . Тогаш важи 1      d2 .
S d k 1d k d k 1  n d k 1

n2 n2 n2
Јасно, е делител на n 2 и бидејќи 1   d 2 следува дека  d2 .
S S S

Според тоа S  dk 1dk т.е. k  1  1, k  2.

138. ММО 2009. Во множеството цели броеви реши ја равенката

x2010  2006  4 y 2009  4 y 2008  2007 y.

Решение. Дадената равенка ја запишуваме во еквивалентна форма

x2010  1  4 y 2009  4 y 2008  2007 y  2007   y  1  4 y 2008  2004  3 . ...(1).

Бидејќи 4 y 2008  2004  3  4  y 2008  501  3  4k  3 следува дека постои барем еден прост
делител на 4k  3 од облик 4t  3.

Од друга страна x 2010  1   x1005   1  a 2  1. Ако ја искористиме Својство 5.9 добиваме


2

дека сите прости делители  p  3 на бројот x 2010  1 се од облик 4k  1.

Според тоа дадената равенка нема решение во множеството цели броеви.

139. Јапонија 2011. Најди ги сите петорки од природни броеви a, n, p, q, r  така што
  
важи a n  1  a p  1 a q  1 a r  1 . 
Решение. Ако a  1 јасно е дека било кои природни броеви n, p, q и r се решение на
равенката. Сега нека a  1 , и без губење на општоста земаме дека p  q  r.

   
Следува дека a n  1  a p  1 a p  1 a p  1  a p  1  a 3 p  3a p a p  1  1  a 3 p  1.
3

Значи n  3p. Од a p  1 a n  1 и од Својство 5.5. добиваме

НЗД  a p  1, a n  1  a p  1  a НЗД  n, p   1 , од каде следува p  НЗД  p, n  т.е. p n.

142
200 ТЕОРИЈА НА БРОЕВИ - подготвителни задачи

Значи n  p или n  2 p.

  
Ако n  p имаме дека a q  1 a r  1  1 , од каде се добива a  2, q  r  1. Според тоа ги
добиваме решенијата  2, n, n,1,1 ,  2, n,1, n,1 ,  2, n,1,1, n .

Ако n  2 p се добива  a p  1 a p  1   a p  1 a q  1 a r  1 т.е. a p  1  aqr  a q  a r  1.

Последната р-ка ја делиме со a r и имаме a pr  aq  aq r  1 т.е. a pr  aq r  aq  1 ..(1).

Ако q  r  0 тогаш и p  r  0 , следува левата страна на (1) е делива со a , додека десната


не е. Останува q  r. Треба да ја решиме равенката a p q  a q  2.

Ако a  2 тогаш p  q  1, q  2. Оттука p  3, n  6, r  q  2  2,6,3, 2, 2  е решение како и


сите други пермутации на p, q, r , т.е.  2,6, 2,3, 2  ,  2,6, 2, 2,3 се решение на равенката.

Ако a  2 и p  q  0 тогаш a q  a p q  2 се дели со a , што не е можно. Значи p  q.

Добиваме a q  3, a  3, q  1. Оттука p  q  r  1, n  2 p  2 т.е.  3, 2,1,1,1 е решение на


равенката.

140. Јапонија 2001.

Најди ги сите природни броеви N така што N  a1  1a2  1......an  1 каде a i се
цифрите на бројот N во неговата декадна репрезентација.

Решение. Имаме N  an an1 ....a2 a1  a1  10a2  100a3  ...  10 n1 an 1.

Ако некоја од цифрите на бројот N е еднаква на нула тогаш ai  1  1, a j  1  10, j  i.

Оттука N  a1  1a2  1......an  1  10



10  ....
  10  10 n1 , што не е можно заради 1.

n 1

Ако некоја од цифрите е еднаква на девет тогаш 10 N , што исто така не е можно бидејќи
бројот N не завршува на нула. ( ai  0, i 1, 2,..., n ).

Ако n  2 имаме:

10n1 an  10n2 an1  10n1 an1  10n2 an2  ...  10a2  a1  N 

143
200 ТЕОРИЈА НА БРОЕВИ - подготвителни задачи

  аn  1 an1  1   a1  1   аn  1 an1  1 10 10 10   аn  1 an1  1 10n2.
n  2десетки

Значи 10n1 an  10n2 an1   an  1 an1  1 10n2. Ако поделиме со 10n2 добиваме

10an  an1  an an1  an  an1  1 т.е. an 10  an1   an  1. Значи an 10  an1   an т.е.

an1  9, што не е можно. Ако n  2 добиваме единствено решение N  18.

103
141. Сингапур 2008. Најди ги сите прости броеви p, за кои p го дели бројот n
n 1
p 1
.

Решение. Ако p  103 тогаш НЗД  i, p   1 , каде i 1, 2,...,103. Од малата теорема на
103
Ферма важи n
n 1
p 1
 1  1  ...  1 103  mod p  . Следува p 103 , што не е можно бидејќи

p  103 . Значи p  103. Важи n p 1  1 mod p  кога НЗД  n, p   1 и n p 1  0  mod p 


кога НЗД  n, p   p т.е. p n.

103  103   103


Оттука следува n p 1
 1 103      mod p  . Бидејќи p  n
p 1
добиваме дека p е
n 1   p  n 1

103 
непарен прост број за кој важи 103     mod p  . Нека 103  pq  r каде 0  r  p  1.
 p 

103 
Бидејќи    q следува дека r  q  mod p  . Ќе разгледаме два случаи:
 p 

1) Ако q  p тогаш од r  q  mod p  следува дека r  q. Значи 103  pr  r  r  p  1 ,

од каде следува r  1 и p  1  103. Добиваме p  102 што не е можно.

2) Ако q  p добиваме 103  pq  r  p 2 т.е. p  103  11. Останува p  3,5,7 но


со проверка се добива дека p  3 е единствено решение.

1
142. Хонг Конг 1999. Најди ги сите позитивни рационални броеви r  1 такви што r r 1 е
рационален број.

144
200 ТЕОРИЈА НА БРОЕВИ - подготвителни задачи

1 q
 p  p 1  p  p  q
1
p
Решение. Нека r  , p, q  Z , q  0 и НЗД  p, q   1. Следува r r 1    q    .
q q q

q
p q
 p  pq a
q
 p a
Нека a, b  Z , b  0, НЗД a, b  1 така што    т.е.      .
 
q b  
q b

a c
Бидејќи НЗД  q, p  q   НЗД  p, q   1 дропката е q  ти степен на дропка ,
b d
p q
p c
т.е можеме да замениме a  c q , b  d q , НЗД  c, d   1 и добиваме   .
q d 

Сега, ако p  q  0 земаме k  p  q е природен број и притоа важи p  c k и q  d k .

Равенката ck  d k  k каде k е природен број има решение само кога k  1.

q 1
Значи p  q  1 т.е. бараниот рационален број е .
q

q
На ист начин, ако q  p  0 добиваме решение рационален број од облик .
q 1

 n 1 n 
Според тоа r   , .
nN  n n  1

143. Украина 2002. Во множеството цели броеви реши ја равенката

n 2002  mm  nm  2n  ..  m  2001n.

Решение. Лесно се зебележува дека 0, 0 е едно целобројно решение на дадената равенка.

Ќе докажеме дека равенката нема други решенија. Нека претпоставиме дека m, n  0.

Важи НЗД  m, n   НЗД  m  kn, n  , k  N бидејќи ако d n и d m  kn тогаш и d m.

Јасно, важи и обратното. Ако НЗД  m, n   1 тогаш НЗД  m  kn, n   1, за сите


k  1, 2,.., 2001. Оттука m  m  n  m  2n    m  2001n  не се дели со n , што не е можно
бидејќи m  m  n  m  2n    m  2001n   n2002 . Значи НЗД  m, n   d  1.

Постојат цели броеви m1 , n1 така што m  m1d , n  n1d и НЗД  m1 , n1   1.

145
200 ТЕОРИЈА НА БРОЕВИ - подготвителни задачи

Заменуваме во дадената р-ка и добиваме n12002  m1  m1  n1  m1  2n1    m1  2001n1  ..(1).

Сега НЗД  m1  kn1 , n1   1 , за сите k  1, 2,.., 2001. Равенката (1) нема решение, претходно
покажавме.

144. Албанија 2010. Дадена е низата на Фибоначи дефинирана со a1  a 2  1 и


an 2  an1  an ,  n  1.

а) Докажи дека a 2010 се дели со 10.

б) Дали a1005 се дели со 4 ?

Решение. а) Од Теорема 9.8. имаме:

Бидејќи 15 2010 следува дека a15 a2010 . Лесно се проверува дека a15  610 -се дели со
10,

од каде следува дека 10 a2010 .

б) Бидејќи НЗД  аm , аn   aНЗД  m, n добиваме НЗД  a6 , a1005   aНЗД  6,1005  a3  2

т.е. НЗД 8, a1005   2. Значи a1005 не се дели со 4.

145. БМО 2005. Најди ги сите прости броеви p за кои p 2  p  1 е точен куб на
природен број.

Решение. Нека y е природен број таков што p 2  p  1  y 3 .

 
Следува p p  1   y  1 y 2  y  1 ...(1). Бидејќи p 2  p  1  p 3 имаме дека y  p.

Јасно p y  1 или p y 2  y  1. Бидејќи y  p следува дека p y 2  y  1, од каде постои


природен број s така што y 2  y  1  ps. Ако замениме во (1) добиваме p  sy  s  1.

Од y 2  y  1  ps со заменување p  sy  s  1 се добива y 2  y 1  s 2   s 2  s  1  0. ..(2).


Последната квадратна равенка по y има решенија природни броеви ако и само ако
нејзината дискриминанта е точен квадрат, т.е. акко 1  s 2   4  s 2  s  1  s 4  6s 2  4s  3
2

146
200 ТЕОРИЈА НА БРОЕВИ - подготвителни задачи

е полн квадрат. Лесно се проверува дека s  3 бидејќи за s  1 се добива y  p, додека за


s  2 се добива квадратната равенка y 2  3 y  3  0 која нема решенија во множеството
природни броеви.

Важи  s 2  3  s 4  6s 2  4s  3   s 2  1 па оттука s 4  6s 2  4s  3   s 2  3 или


2 2 2

 
2
s 4  6s 2  4s  3  s 2  2 . Првата равенка има решение s  3, додека втората равенка
нема решение во множеството природни броеви. Заменуваме s  3 во  2  и добиваме
дека y  7 е решение на квадратната равенка y 2  8 y  7  0. Второто решение y  1 не е
можно. На крај добиваме p  sy  s  1  19, е единствено решение.

146. Јапонија 1996. Нека m и n се природни броеви такви што НЗД m, n  1.

Пресметај НЗД  5n  7n ,5m  7m  .

Решение. Нека s n  5 n  7 n. Ако n  2m забележуваме дека важи

5n  7n  5n  5m7nm  7m5nm  7n  7m5n m  5m7n m  5m  7m 5n m  7n m   5m 7m 5n 2m  7n 2m  ,


т.е. s n  s m s nm  5 m 7 m s n 2 m .

Оттука следува НЗД sn , sm   НЗД sm snm  5m 7 m sn2m , sm   НЗД sm , sn2m .

На ист начин, ако m  n  2m , добиваме sn  sm snm  5nm7nm s2mn , од каде следува

НЗД  sn , sm   НЗД  s2mn , sm  .

Од Евклидовиот алгоритам добиваме:

Ако m  n е парен број тогаш НЗД  sn , sm   НЗД  s1 , s1   12.

Ако m  n е непарен број тогаш НЗД  sn , sm   НЗД  s0 , s1   НЗД  2,12   2.

x4 1 y 4 1
147. Виетнам 2007. Нека x  1, y  1 се цели броеви така што  е цел број.
y 1 x 1
Докажи дека бројот x 4 y 44  1 се дели со x  1.

147
200 ТЕОРИЈА НА БРОЕВИ - подготвителни задачи

Решение. Забележуваме x 4 y 44  1  x 4  y 44  1  x 4  1. Јасно е дека x 4  1 се дели со x  1 и


y 44  1 се дели со y 4  1. Оттука, доволно е да докажеме дека y 4  1 се дели со x  1.

x4  1 a y 4  1 c
Нека ставиме  ,  , каде a, b, c и d се цели броеви за кои важи
y 1 b x 1 d
НЗД  a, b   НЗД  c, d   1 и b, d  0.

a c ad  bc
Добиваме    t т.е. ad  bc  bdt , за некој цел број t. Важи b ad  bc па
b d bd
оттука b ad т.е. b d , бидејќи НЗД  a, b   1. На ист начин се добива d b , од каде
следува дека b  d .

a c x4 1 y 4 1
Од друга страна  
b d

x 1 y 1
   
  x  1 x 2  1  y  1 y 2  1 е цел број, и бидејќи
a
b
и

c y4 1
се нескратливи дропки следува дека b  d  1. Значи  c е цел број
d x 1

т.е. x  1 y 4  1.

148. ИМО 2000. Определи дали постои природен број n така што n има точно 2000
различни прости делители, и 2 n  1 се дели со n.

Решение. Ќе докажеме дека за секој цел број a  2 постои прост број p кој го дели
бројот a 3  1 , но не го дели a  1.

 
Бидејќи a 3  1  a  1 a 2  a  1 доволно е да се покаже дека постои прост број кој го
дели бројот a  a  1 но не го дели бројот a  1.
2

 
Важи a 2  a  1  a  1a  2  3 , од каде НЗД а 2  а  1, а  1  НЗД  a  1,3 1,3.
 
Ако НЗД a  a  1, a  1  1 тогаш својството е докажано.
2

 
Ако НЗД a 2  a  1, a  1  3 тогаш јасно е дека 3 a  1 и 3 a  1  3 т.е. 3 a  2.

Бидејќи a  2 следува дека a 2  a  1  3 , од каде добиваме дека постои прост број p  3


кој го дели a 2  a  1, но не го дели a  1. Со тоа својството е докажано.

Според тоа постојат 2000 непарни различни прости броеви p1 , p 2 ,..., p 2000 , така што

148
200 ТЕОРИЈА НА БРОЕВИ - подготвителни задачи

p1  3, p 2  3, p 2 2 3  1  2
 и 
pi 2 3  1
i
 но p i не е делител на
i 1
23  1 за секој
3  i  2000.

Ќе докажеме дека бројот n  p12000  p 2   p 2000  32000  p 2    p 2000 ги исполнува условите на

задачата. За секој 2  i  2000 важи 3i 3 2000 како и pi 23  1 23  1.


i 2000

Со помош на математичка индукција ќе докажеме дека 3k 1 23  1, за секој природен број


k

k . За k  1 важи 9 9. Нека претпоставиме дека 3k 1 23  1. Забележуваме дека


k

23   1  1 mod 3 . Според тоа 23  


2
 23  1   1   1  1  0  mod 3 т.е.
k 3k k k 2

 
2
 23  1.
k k
3 23

  1  2    2
 1  23  1  a  3k 1  b  3  3k  2 c , односно
k 1 3 2
Сега имаме: 23  1  23
k
3k k
3k

 
k 1
3k  2 23  1. Со тоа индукцијата е докажана. Оттука следува дека n 23  1 2n  1, бидејќи
2000

n е непарен број кој се дели со 32000.

149. ИМО 1981. Одреди ја најголемата вредност на изразот m 2  n 2 ако m и n се


природни броеви помали од 1981 за кои важи

n 2  mn  m 2  1.

Решение. Најпрво ќе ја решиме равенката n 2  mn  m 2  1 во множеството на природни


броеви. Ако m  n добиваме m  n  1.

Ако парот m, n  каде m  n , е решение на горната равенка тогаш важи

n 2  mn  m 2  1 или m 2  mn  n 2  1 .

Од n 2  1  m 2  mn јасно е дека n  m. Истото се добива и од втората равенка бидејќи

mn  1
n 2  m 2  mn  1 т.е. n  m   0.
nm

Од n  m следува дека постои природен број k таков што n  k  m.

149
200 ТЕОРИЈА НА БРОЕВИ - подготвителни задачи

Со заменување на n  k  m во n 2  mn  m 2  1 добиваме дека k 2  km  m 2  1

т.е. k, m е решение на равенката. Значи ако парот m, k  m е решение на равенката
тогаш и парот k, m е решение. Важи и обратното. Ако парот k, m е решение на горната
равенка тогаш и парот m, k  m е решение. Бидејќи k  n  m добиваме дека парот
n  m, m индуцира нов пар решение m, n. Бидејќи 1,1 е решение добиваме n  2, m  1
т.е. 1, 2 е ново решение. Сега n  m  1, m  2 од каде добиваме ново решение 2, 3.

Паровите 1,1, 1, 2, 2, 3, 3, 5, 5, 8, 8,13,....987,1597, 1597, 2584.. се решенија на дадената
равенка (членови на низа на Фибоначи). Според тоа 987 2  1597 2 е најголемата вредност
на изразот m 2  n 2 .

a n2
150. Предлог ИМО 1994. Нека a0  1994 и a n 1  за секој природен број n.
an  1

Докажи дека 1994  n е најголемиот цел број помал или еднаков на an , 0  n  998.

a n 1 an
Решение. Јасно an  0, за секој природен број n. Бидејќи   1 добиваме дека
an an  1
a n n0 е строго монотоно опаѓачка низа.

a k2  ak 1
Ако искористиме дека a k 1  a k   ak    1 добиваме:
ak  1 ak  1 ak  1

 1   1 
a n  a 0  a1  a 0   a 2  a1   ..  a n  a n 1   1994    1    1  ....
 0
a  1   1 a  1 
 1   1 1 
...    1  1994  n   
1
 ...    1994  n. 1
 n 1
a  1   0
a  1 a 1  1 a n 1  1 

Од 1 следува дека a997  1994  997  997. Бидејќи a n n 0 е опаѓачка низа важи

1

1
,
1

1
,
1

1
, .....,
1

1
2
a0  1 a997  1 a1  1 a997  1 a 2  1 a997  1 a997  1 a997  1

998 998
Ако 2 го замениме во 1 добиваме a n  1994  n   1994  n   1995  n.
a997  1 997  1

Значи 1994  n  an  1995  n, па следува дека a n   1994  n.

150
200 ТЕОРИЈА НА БРОЕВИ - подготвителни задачи

151. Предлог ИМО 1990.

Нека f 0  f 1  0 и f n  2  4 n 2 f n  1  16 n1 f n  n  2 n ,


2

n  0,1, 2,.. . Докажи дека броевите f 1989, f 1990, f 1991 се делат со 13.

Решение. Нека f n  g n  2 n . Од f n  2  4 n 2 f n  1  16 n1 f n  n  2 n ја добиваме


2 2

рекурентната формула g n  2  2 n 2   4 n 2 g n  1  2 n1  16 n1 g n  2 n  n  2 n ,


2 2 2 2

т.е.

g n  2  2 n  2 4n  2 4  4 n  4 2 g n  1  2 n  2 2n  2  16 n  16 g n  2 n  n  2 n ,
2 2 2 2

16  2 n  16 n g n  2  2  g n  1  g n  n  2 n .
2 2

2
По делење на последното равенство со 2 n добиваме дека важи

g  n  2   2  g  n  1  g  n   n 16 n1 ..(1).

Сега ако во (1) замениме n  0,1,.., n  1 и ги собериме n  те равенства ќе добиеме:

n 1
 g  2  g 3  ..  g  n  1  2  g 1  g  2   ..  g  n    g  0   g 1  ..  g  n 1   n 16  n 1

n 0

 1 1 1   1 1   1 1  1
т.е. g  n  1  g  n    2  3  ...  n    3  ...  n   ..   n 1  n  n 
 16 16 16   16 16   16 16  16

1  15n  1 
 1  .
152  16n 

Оттука добиваме g  n    g  n   g  n  1    g  n  1  g  n  2    ...   g 1  g  0   

1  15  n  1  1 15  n  2   1 15 1  1 
 2 
1 n 1
1 n2
 ...  1  
15  16 16 161 
1  1 1 1   15  1 2 n 1 
   n  1  1  15n    2  ...  n 1    2  n 1  n  2  ..  
152   16 16 16  15  16 16 16 
1   16n 1  1   1  n  1 1  16n 1  1  
2   n  1  1  15n   
n 1      
n 1  
15   15 16   15  15 15  15 16  
1  15n  2 
 15n  32  .
153  16n 1 

151
200 ТЕОРИЈА НА БРОЕВИ - подготвителни задачи

Според тоа имаме

1  15n  2  n2 1  15n  2   n  22


f n  3 
15n  32  n 1 
 2  3 15n  32  2 16n 1 
15  16  15  16n 1 

3 
15n  2  15n  32  16n 1   2 n  2 .
1 2

15

Сега, нека f  n   a  mod13 .

 
Добиваме 15n  2  15n  32 16n1 2 n2  153 f  n   153 a  23 a  mod13 .
2

Оттука f  n   15n  2  15n  32 16n1  2n  2   2n  6   3n1  mod13 . Сега заменуваме


n  1990. Бидејќи 33  1 mod13 следува дека 31990  31989  3  3  mod13 . На крај се добива
дека f 1990   0  mod13 . На ист начин се добива дека f 1989   0  mod13 и
f 1991  0  mod13 .

152. Низата a0 , a1 , a2 ,... е дефинирана со: a0  0, a1  1, an2  2an1  an . Докажи дека 2k ak


ако и само ако 2k n.

Решение. Бидејќи an2  2an1  an  an mod 2 следува дека a n и an2 имаат иста
парност. Бидејќи a0  0 и a1  1 добиваме дека a 2 k се парни, додека a 2 k 1 се непарни
броеви. Карактеристичната равенка за дадената рекурентна формула е r 2  2r  1  0 чии
решенија се r1  1  2 и r2  1  2 .

1 1
Важи an  c1r1n  c2 r2n . Ако замениме n  0 и n  1 наоѓаме дека c1  , c2  
8 8

односно општиот член на низата е a n 


1 
8
 1  2  1  2  .

n n


   
Дефинираме нова низа bn n 0 со иста формула како и низата an n 0 т.е. bn 2  2bn1  bn
 

но b0  b1  2 . Од исти причини како кај an  следува дека bn  е низа чии членови се
парни броеви. Бидејќи bn2  2bn1  bn  bn mod 4 и бидејќи b0  b1  2 следува дека
bn  2mod 4 за секој природен број n . Аналогно како кај an  добиваме дека


bn  1  2   1  2  . Со множење се добива
n n

152
200 ТЕОРИЈА НА БРОЕВИ - подготвителни задачи

a n  bn 
1 

 1 2
8
  1  2 
2n 2n
  a .
 2n

Со помош на математичка индукција по k ќе докажеме дека 2 k an  2 k n .

За k  1 е јасно. Претпоставуваме дека важи 2 k an  2 k n .

Сега 2 k 1 a n  2 k 1 a n  b n и бидејќи b n е парен број кој не се дели со 4 следува дека


2 2 2

n
2 k 1 a n  b n  2 k a n  2 k  2 k 1 n .
2 2 2
2

Со тоа доказот е завршен.

x y zu
153. Предлог ИМО 2001. Го разгледуваме системот  .
 2 xy  zu

x
Најди ја најголемата вредност за реалната константа m така што m  за секое решение
y
природни броеви x, y, z, u  , така што x  y.

Решение. Имаме x  y   x 2  2 xy  y 2  x  y   4 xy  z 2  u 2 .
2 2

Оттука добиваме дека z, u, x  y  е Питагорина тројка. Од x  y јасно е дека x  y  0.

Со примена на својството за генерирање на Питагорини тројки добиваме дека постојат


природни броеви a, b, c  a  b  , така што

 x  y  ca  b 
   
z  c a 2  b 2 , u  2abc и x  y  c a 2  b 2 . Сега, од системот 
2 2

2
  
 x  y  c a  b  2abc
2

  
добиваме дека x  c a  ab , y  c ab  b . Нека
2 2
 a
b
 k.

Имаме

153
200 ТЕОРИЈА НА БРОЕВИ - подготвителни задачи

a2 a
 

x a  ab b 2 b k 2  k
2
 

k 2  1  k  1  2 
 k 11
2 
 3   k  1 
2 

y ab  b 2
a k 1 k 1 k 1  k 1
1
b

 3  2 k  1 
2
 3  2 2.
k  1
2
Равенство се достигнува кога k  1  , односно за k  1 2. Значи m  3  2 2.
k 1

154. Германија 2010. Определи ги сите парови од природни броеви m, n  за кои важи

3m  7 n  2.

Решение. Ако n  1 следува дека m  2.

Ако n  2 тогаш 3m  51 па затоа m  4. Нека a  m  2  2 и b  n  1 1.


Имаме 3m  7 n  2  3m  9  7 n  7  9 3a  1  7 7 b  1 .   
 
Од последната равенка следува дека 7 3 a  1 , и бидејќи ( од малата теорема на Ферма)
 
важи 7 36 k  1 добиваме дека 6 a. Од 3 6
 
 1 3 a  1 и бидејќи 13 36  1 имаме


13 3a  1 . 
   
Значи 13 7 b  1 . Бидејќи 13 712k  1 следува дека 12 b. Значи 712  1 7 b  1.

   
Сега 43 712  1 па затоа 43 3 a  1 . На ист начин следува дека 42 a , па оттука
3 42
   
 1 3 a  1 . Од 49 3 42  1 следува дека 49 7 7 b  1 , што не е можно бидејќи

7 b  1 не се дели со 7. Значи парот  2,1 е единствено решение на равенката.

Забелешка: Од Ојлеровата теорема важи 3


  49
 342  1 mod 49  .

155. Хонг Конг 2008. Нека n  4 е сложен број и притоа важи n  n n  1 , каде   n 

е Ојлерова функција, а  n  е функцијата која го пресметува збирот на сите природни


делители на бројот n . Докажи дека n има најмалку три различни прости делители.

154
200 ТЕОРИЈА НА БРОЕВИ - подготвителни задачи

Решение. Ојлеровата функција   n  секогаш добива вредност парен број.

Оттука  n n  1 е непарен број, па следува и n е непарен број. Ќе докажеме дека во
каноничната факторизација на n степенот на секој прост делител p е еднаков на еден.

Нека претпоставиме спротивно, нека степенот на p е најмалку два, т.е n  a  p  ,   2 .

Од p n следува дека p  n n  1 . Имаме  n  p  1  p  1q . Јасно p  n  .

Од p  n  и p  n n  1 следува дека p  1 , што не е можно. Значи   1.

Сега нека претпоставиме дека n има најмногу два прости делители, т.е. нека n  p  q .
Важи  n   p  1q  1 и  n  1  p  q  pq   p  1q  1 .

p2  q2  2
Значи pq  p  1q  1 p  1q  1  1 , pq p 2 q 2  p 2  q 2  2 т.е.  k е цел број.
pq

Нека  a, b  е пар од природни броеви за кои важи p 2  q 2  2  kpq и притоа збирот a  b


е најмал. Без губење на општоста земаме дека a  b. Ја разгледуваме квадратната равенка
b2  2
a2  kba  b2  2  0 ...(1) по непознатата a. Од Виетовите правила следува дека е
a
 b2  2 
решение на (1). Оттука  , b  е нов пар решение на (1). Од минималноста на a  b
 a 
b2  2
следува дека a  b   b од каде следува дека a  b, што е спротивно на a  b.
a

Според тоа n има најмалку три различни прости делители.

156. Предлог ИМО 2008. Најди ги сите цели броеви a и b кои се решение на равенката

a7 1 5
 b  1.
a 1

Решение. Најпрво ќе ја докажеме следната лема:

a7 1
Лема: Ако p е прост делител на бројот тогаш p  1mod 7 или p  7.
a 1

Доказ. Нека p  1 не се дели со 7. Тогаш НЗД  p  1, 7  1.

155
200 ТЕОРИЈА НА БРОЕВИ - подготвителни задачи

Бидејќи p a7  1 следува дека a7  1 mod p  т.е. НЗД  а, p   1. Од теорема на Безу


постојат цели броеви m, n така што  p  1m  7n  1, додека од теорема на Ферма
a p 1
 1mod p . Според тоа добиваме a  a  p 1m a 7 n  1mod p .

a7 1
Значи  a 6  a 5  a 4  a 3  a 2  a  1  7 mod p , т.е. p 7 што е можно за p  7.
a 1

Со тоа лемата е докажана. Нека b  1  p1 p 2  ...  pk . Бидејќи pi  1mod 7 или pi  7 ,
1 2 k

за секој i  1, 2,..., k добиваме дека b  1  1mod 7 или b  1  0 mod 7.

a7  1
Нека d е произволен делител на . Тогаш d  p11  ptt , и притоа pi  1 mod 7  или
a 1
pi  7. Оттука следува d  1 mod 7  или d  0  mod 7  .

Ако b  2 mod 7 тогаш b 4  b 3  b 2  b  1  3 mod 7 , што не е можно бидејќи секој


a7 1
делител на при делење со седум има остаток нула или еден.
a 1

Ако b  1mod 7 тогаш b 4  b 3  b 2  b  1  5 mod 7 , што исто така не е можно од исти


причини. Значи дадената равенка нема решение во множеството цели броеви.

157. Предлог ИМО 2000. Определи ги сите природни броеви n  2 за кои важи следниот
услов: За сите цели броеви a, b заемно прости со n,

a  b mod n ако и само ако ab  1mod n.

Решение. Ако од a  b следува ab  1mod n тогаш a 2  ab  1mod n.

Ако a 2  1mod n тогаш од ab  1mod n следува a 2  ab mod n т.е. a  b mod n.

Значи доволно е да ги најдеме сите цели броеви a заемно прости со n за кои важи

a 2  1mod n. Нека n  p1 p 2    p k е канонична факторизација на бројот n.


1 2 k

Бидејќи pi i n следува дека a 2  1 mod pi  , за секој i  N . Доволно е да ги најдеме сите
i

цели броеви заемно прости со p i за кои важи a 2  1 mod pi  . i


Во тој случај, од
кинеската теорема за остатоци , системот конгруентни равенки

156
200 ТЕОРИЈА НА БРОЕВИ - подготвителни задачи

a 2  
 1 mod p11
 2
a  1 mod p  2


2

 .........
a 2
 
 1 mod p k k 
има единствено решение модул p1 p 2    p k  n .
1 2 k

Ако pi  3 тогаш НЗД 2, pi   1 па треба да важи 2 2  1 mod pi  . Од последната i

конгруенција имаме pi  3 и  i  1.

Ако p j  2 , од НЗД2, 3  1 следува 32  1 mod 2  ј


. Добиваме дека  j  1, 2, 3.

Останува n 2, 3, 4, 6, 8,12, 24.

158. Виетнам 2002. Најди ги сите природни броеви n за кои равенката

x  y  u  v  n xyuv

има решение  x, y, z, u  во множеството цели броеви.

Решение. Дадената равенка е еквивалентна со равенката x  y  u  v 2  n 2 xyzv .

Имаме x  y  u  v 2  x   y  u  v 2  x 2  2 x y  u  v    y  u  v 2  n 2 xyzv .

Нека n е природен број за кој равенката има решение x0 , y0 , u0 , v0  и притоа збирот
x0  y0  u0  v0 е најмал.

Без губење на општоста претпоставуваме дека x0  y0  u0  v0 .

Бидејќи  y  u  v 2  n 2 xyzv  x 2  2 x y  u  v  јасно е дека x0  y0  u 0  v0  .


2

Исто така x 0 е природен број кој е решение на квадратната равенка

 
f  x   x 2  x 2  y0  u0  v0   n2 y0u0v0   y0  u0  v0   0 ...(1).
2

Од Виетовите правила следува дека квадратната равенка има второ решение x1 за кое
 y0  u 0  v0 2
важи x0 x1   y0  u0  v0  т.е. x1 
2
е природен број.
x0

157
200 ТЕОРИЈА НА БРОЕВИ - подготвителни задачи

Според тоа и четворката x1 , y0 , u0 , v0  е решение на равенката (1).

Бидејќи претпоставивме дека збирот x0  y0  u0  v0 е најмал јасно е дека x1  x0 .

Важи y0  x0 , x1  и бидејќи за x   , x0   x1 ,  f x   0 следува дека f  y0   0.

Имаме:

0  f  y0   y02  2 y0  y0  u0  v0    y0  u0  v0   n 2 y02 u0 v0 
2

 y02  2 y0  y0  y0  y0    y0  y0  y0   n 2 y02 u0 v0  16 y02  n 2 y02 u0 v0


2

т.е. n 2 u 0 v0  16. Јасно n 2  n 2 u0 v0  16 , од каде n  1, 2, 3, 4.

Лесно се забележува дека за секоја вредност n  1, 2, 3, 4 дадената равенка има единствено


решение 4, 4, 4, 4, 2, 2, 2, 2, 1,1, 2, 2 и 1,1,1,1 , соодветно.

159. Предлог ИМО 2005. Нека a и b се природни броеви така што a n  n го дели
b n  n за секој природен број n. Докажи дека a  b.

Решение. Нека a  b. Бидејќи a n  n b n  n следува дека a  b. Нека p  b е прост број и


n  a  1 p  1  1. Јасно е дека n  1mod p  1 . Од n  a  1 p  1  1  ap  p  a
следува дека n  a mod p . Оттука следува дека за секој цел број r заемно прост со p

важи r  r r
n
 
p 1 a 1
 r mod p  . Притоа користивме теорема на Ферма r p 1  1mod p .

Јасно НЗД  а, р   1 бидејќи p  b  a па можеме да земеме r  a. Важи a n  a mod p  .


Значи a n  n  a  a  0 mod p . Според тоа p b  n , и бидејќи b n  b mod p  добиваме
n

дека p b  a. Последното не е можно бидејќи p  b и a  b. Останува a  b.

160. ИМО 2009. Нека n е природен број и a1 , a 2 , ...., a k ( k  2 ) се различни цели броеви
од множеството 1, 2, ...., n така што n го дели ai ai 1  1 , за секој i  1,...., k  1.

Докажи дека n не го дели бројот a k a1  1.

Решение. Со помош на математичка индукција по k ќе докажеме дека n не го дели


a k a1  1. Нека k  2. Од условот на задачата важи a1 a2  a1 mod n .

158
200 ТЕОРИЈА НА БРОЕВИ - подготвителни задачи

Ако n го дели a 2 a1  1 тогаш a1 a2  a2 mod n. Бидејќи a1a2  a1  mod n  следува дека
a1  a 2 mod n , што не е можно бидејќи a1  a2 и a1 , a2 1, 2,..., n. Значи n не го дели
бројот a 2 a1  1.

Сега нека претпоставиме дека за секој 1  i  k , n не го дели бројот ai a1  1.

Ако конгруенцијата ai ai 1  ai mod n ( која важи од условот на задачата) ја помножиме


со a1 a 2    ai 1 добиваме a1 a2    ai ai 1  a1 a2    ai mod n.

Оттука имаме

a1 a 2  a k  a1 a 2    a k 1  ....  a1 a 2  a1 mod n ....(1).

Слично, од условот на задачата имаме а2  а2 а3  а2 а3а4  ...  а2 а3  аk  mod n  ...(2).

Ако претпоставиме дека n го дели a k a1  1 односно a k a1  a k mod n , и замениме во (1)


добиваме а1аk а2 а3  аk 1  а2 а3  аk 1ak  a1  mod n  ...(3). Сега од (2) и (3) следува дека

а2  а2 а3 ...аk  а1  mod n  , што не е можно, докажавме погоре. Значи останува дека бројот
аk  a1  1 не се дели со n.

161. Предлог ИМО 2006. Низата f 1, f 2, f 3,... е дефинирана со формулата

1 n n n


f n      ....    ,
n   1   2  n

каде x  го означува целиот дел од x.

а) Докажи дека f n  1  f n за бесконечно многу природни броеви n,

б) Докажи дека f n  1  f n за бесконечно многу природни броеви n.

n n n


Решение. Нека g n         ....     nf n  и дефинираме g 0  0.
1 2 n

 n   n  1  n   n  1
Лесно се забележува дека       1 ако n се дели со k , и  k    k   0
k   k     

ако n не се дели со k . Според тоа имаме:

159
200 ТЕОРИЈА НА БРОЕВИ - подготвителни задачи

  n   n  1    n   n  1    n   n  1 
g n   g n  1            2    2    ...    n    n    d n  ,
  1   1            

каде d n  го означува бројот на природни делители на бројот n.

Добиваме g n  g n  1  d n  g n  2  d n  1  d n  ....  d 1  d 2  ....  d n.

d 1  d 2  ....  d n 


Оттука f n   .
n

d 1  d  2   ...  d  n   d  n  1
а) Условот f n  1  f n е еквивалентен со  f  n ,
n 1

и бидејќи d 1  d  2   ...  d  n   nf  n  следува дека доволно е да докажеме дека


постојат бесконечно многу природни броеви n за кои важи d n  1  f n . За
бесконечно многу природни броеви n важи d n  1  d 1, d n  1  d 2,....., d n  1  d n
, па според тоа имаме

d 1  d 2  ....  n 


d n  1   f n .
n

б) Условот f n  1  f n е еквивалентен со d  n  1  f  n  и е исполнет за бесконечно


многу природни броеви n за кои n  1 е прост број. Во тој случај d n  1  2  f n.

162. Хрватска 1999. Во множеството реални броеви реши ја равенката

 x  2x  4x  8x  16x  32x  12345.


Решение. Нека x е реално решение на равенката. Јасно е дека x  0.

b c d e f
Нека x  a       k , каде a е ненегативен цел број, b, c, d , e, f 0,1
2 4 8 16 32

1
и 0k  . Ако замениме во горната равенка добиваме
32

160
200 ТЕОРИЈА НА БРОЕВИ - подготвителни задачи

 b c d e f   c d e f   d e f 
a   2  4  8  16  32  k   2a  b   2  4  8  16  2k   4a  2b  c   2  4  8  4k  
          
 e f   f 
 8a  4b  2c  d     8k   16a  8b  4c  2d  e    16k  
 2 4   2 
 32a  16b  8c  4d  2e  f  32k   12345.
(1).

1
Бидејќи b, c, d , e, f  1, k 
лесно се докажува дека вредноста на изразите во заграда во
32
(1) е помала од еден. Ако искористиме дека s  t   s, кога s е ненегативен цел број и
t  1 е реален број, равенката (1) го добива следниот облик:

a  2a  b  4a  2b  c  8a  4b  2c  d  16a  8b  4c  2d  e  32a  16b  8c  4d  2e  f  12345,


т.е. 63a  31b  15c  7d  3e  f  12345.

Од последната равенка добиваме: 63a  12345  63a  31  15  7  3  1  63a  57,

12288 12345
т.е. a . Следува 195  a  196 , што не е можно бидејќи a е цел број.
63 63

Значи дадената равенка нема реално решение.

Забелешка: Секој позитивен реален број x може да се запише во облик

a1 a2 a 1
x a  2  ....  nn  k , каде a е природен број, ai  0,1 и 0  k  n .
2 2 2 2

1 1 1  1
Важи a  x  a    2  ...  n   n  a  1. Оттука  x   a.
 2 2 2  2

163. Предлог ИМО 1988. Нека a е најголемиот позитивен корен на равенката


   
x 3  3x 2  1  0. Докажи дека a 1788 и a 1988 се деливи со 17.

Решение. Бидејќи 17 34 x и 17 119 имаме x 3  3x 2  1  x 3  3x 2  34 x  120 mod 17.

Добиваме:

x 3  3x 2  34 x  120  x 3  3x 2  54 x  20 x  120  x 3  6 x 2  9 x 2  54 x  20 x  120 


 
 x 2 x  6  9 xx  6  20x  6  x  6 x 2  9 x  20  x  4x  5x  6.

161
200 ТЕОРИЈА НА БРОЕВИ - подготвителни задачи

Според тоа x3  3x2  1   x  4 x  5 x  6  mod17  .

Нека a, b, c ( a  b  c ) се корени на равенката x3  3x2  1  0.

Важи 0.6  c  0.5, 0.6  b  0.7 и 8  a  3. Од Виетовите правила за равенката


3 0 1
x3  3x2  1  0 имаме a  b  c    3, ab  bc  ca   0, abc    1.
1 1 1

Дефинираме низа kn  a n  bn  cn , n 0,1, 2,...

  
kn 3  a n 3  b n 3  c n 3   a  b  c  a n  2  b n  2  c n  2   ab  bc  ca  a n 1  b n 1  c n 1 

 abc a  b  c
n n n
  3k n2  kn .

Бидејќи k0  3, k1  a  b  c  3, k2  a 2  b2  c 2   a  b  c   2  ab  bc  ca   9,..
2

kn3  3kn 2  kn следува дека низата kn  е низа од природни броеви.


Јасно е дека 0  bn  cn  1,  n  2 , па оттука a n   kn  bn  c n   kn  1. 
Од x3  3x2  1   x  4  x  5 x  6  mod17  следува a n  bn  c n  4n  5n   6   mod17  .
n

a n   k  1  a n  bn  cn  1  4n  5n   6   1 mod17  .
n

Значи   n

Останува да докажеме дека 17 41788  51788   6   1 и 17 41988  51988   6 


1788 1988
 1.

Од теорема на Ферма важи 416 k  516 k   6   1 mod 17  .


16 k

Добиваме

41788  51788   6   1  416111  412  516111  512  (6)16111  (6)12  1  412  512   6   1 
12
1788

 1  4  13  1  0  mod17  .

Аналогно се докажува дека 41988  51988   6   1  0  mod17  .


1988

164. Предлог ИМО 1998. Определи ги сите парови a, b  од реални броеви така што важи

abn  ban за сите природни броеви n.

162
200 ТЕОРИЈА НА БРОЕВИ - подготвителни задачи

Решение. Јасно е дека

1) a  b, 2) a  0 или b  0, 3) a и b се цели броеви,

се решенија на равенката. Ќе докажеме дека равенката нема други решенија освен 1), 2) и
3). Нека претпоставиме дека a и b се ненулти и различни реални броеви.

Со помош на математичка индукција ќе докажеме дека  2n a   2n  a  и  2n b   2n b.

Нека n  1. Важи 2  a   2a  2  a   1  2  a   2 , оттука следува  2a   2  a  или

2a  2 a  1. Ако 2a  2 a  1 тогаш a 2b  b 2a  b  2 a  1  2b a  b  2a b  b,
b
па ако поделиме со a добиваме  2b  2 b 
. Значи a  b, што не е можно бидејќи
a
претпоставивме a  b. Значи  2a   2  a  . Аналогно се докажува дека  2b  2 b.

Сега нека претпоставиме дека за 2  k  n важи  2k a   2k  a  и  2k b   2k b.

Ако во дадената равенка замениме n  2k 1 имаме a 2k 1 b   b  2k 1 a  .

Како и претходно, 2k 1 a   2  2k a    2c   2 c  или  2c   2 c   1.

Значи  2k 1 a   2 2k a  или 2k 1 a   2 2k a   1. Ако 2k 1 a   2 2k a   1 тогаш


добиваме

 
a 2k 1 b  b 2k 1 a   b 2 2k a   1  2b 2k a   b  2k 1 b  a   b  2k 1 a b  b.

b
Ако идентитетот a 2k 1 b   2k 1 a b  b го поделиме со a добиваме  2k 1 b   2k 1 b  .
a

Оттука следува a  b , што не е можно. Значи, 2k 1 a   2k 1  a  , 2k 1 b  2k 1 b.

Со тоа индукцијата е завршена.

Бидејќи  2n x   2n  x  и  2n y   2n  y  , за секој природен број n, јасно е дека x и y мора


да бидат цели броеви. Ако x и y не се цели броеви тогаш постои природен број n така
што  2n x   2n  x  и  2n y   2n  y . Јасно, ако  x  a тогаш доволно е да избериме

163
200 ТЕОРИЈА НА БРОЕВИ - подготвителни задачи

природен број n така што  2n x   2n a т.е.  2n x   2n a  1. Според тоа добиваме дека


1
2n x  2n a  1  2n  x  a   1  2 n  .
xa

165. Предлог ИМО 1996. Најди ги сите природни броеви a и b за кои важи

 a 2  b 2   a 2  b 2 
     ab.
 b   a   ab 

Решение. Ако a  b добиваме дека 2a  2  a 2 , од каде следува дека a не е природен


број. Значи a  b и без губење на општоста претпоставуваме дека a  b.

Ако b  1 тогаш бидејќи a  b имаме a 2  2a т.е. 2,1 е едно решение на дадената


равенка.

a2  b 2  a2 b2
Сега нека b  1. Заменуваме    m и    n и добиваме m и  n,
b a b a

a2  b2 
од каде следува ab  mn. Бидејќи a 2  b 2  2ab важи    2.
 ab 

Ако замениме во дадената равенка ја добиваме неравенката m  n  2  mn која ја


трансформираме во еквивалентната неравенка  m  1 n  1  1...(1).

 a2 
Бидејќи a  b следува дека m     1. Според тоа од неравенката (1) следува дека
b
 b2 
n  0. Сега од    n  0 добиваме дека b2  a, т.е. постои природен број c  0 така
a
што a  b2  c. Сега дадената равенка ја трансформираме на следниот начин:

 a 2   b2 
   
 

 b 4  2b 2c  c 2   b b  2bc c 2 
3

  3
 
 c2 
    2   b 2bc   . ..(1).
 b   ab   b   b b
 b

 a 2  b2   b 4  2b 2c  c 2  b 2   
 b2 b2  c  b2  c  1  c 2 
   ab   
 
 b c b  
2
  b3  bc 
 ab    
b2  c b   
b2  c b 

164
200 ТЕОРИЈА НА БРОЕВИ - подготвителни задачи

 b 2  c  1  c 2 
b   b3  bc. ...(2).

 b 2  c b  
 c 2   b 2  c  1  c 2 
Со израмнување на (1) и (2) се добива равенката b  c  1       . ..(3).
 b    b  c  b 
2

Ако c 1 тогаш b 2
 c  b   b2  1 b  b3  b  2b2  b  2b2  1  b2  c  1  c 2 . Значи

 b 2  c  1  c 2 
   0. Равенката (3) секогаш важи бидејќи двете нејзини страни се еднакви
 
 b 2  c b 

на нула. Оттука a  b2  1 па според тоа  a, b    n2  1, n  или  a, b    n, n2  1 е решение


на дадената равенка.

b 2  c  1  c 2  b 2  c  1  c 2   c2 
Ако c  1 тогаш од (3) добиваме    b  c  1     b  c  1
b 2
c b  
 b  c b 
2
 b

 
од каде се добива неравенката c 2 b2  1  b2 c b2  2  b2  1  0. ..(4).   
Имаме
        
c 2 b2  1  b2 c b2  2  b2  1  4 b2  1  b2 2b2  4  b2  1  4 b2  1  b2 b2  5      
 b4  b2  4. Бидејќи b2  4 добиваме

     
c 2 b2  1  b2 c b2  2  b2  1  b4  b2  4  4b2  b2  4  3b2  4  8  0

добиваме дека (4) важи само ако b  2.

Во тој случај (4) добива облик 3c2  8c  20  0 која не важи за c  2.

Според тоа останува дека за секој природен број n, паровите  n2  1, n  и  n, n2  1 се


решенија на дадената равенка.

166. Предлог ИМО 2009. Нека f е неконстантна функција од множеството природни


броеви во множеството природни броеви, така што a  b го дели f a   f b за сите
различни природни броеви a и b. Докажи дека бројот на сите прости делители на
броевите од множеството  f 1 , f  2  , f  3 ,... е бесконечен.

165
200 ТЕОРИЈА НА БРОЕВИ - подготвителни задачи

Решение. Претпоставуваме спротивно, за секој природен број n бројот f n  има прости


делители од конечното множество  p1 , p2 ,..., pk .

Нека f 1  p11 p 2 2    pk k , каде  1 ,  2 ,....,  k се ненегативни цели броеви.

Избираме природни броеви 1 ,  2 , ....,  k такви што 1  1 ,  2   2 ,....,  k   k и земаме

b  p11 p2 2    pk k . Нека f b  1  p1 1 p2 2    pk k . Од условот на задачата важи
b f b  1  f 1 , односно p1  p2  pk
1 2 k
p 1
1 2 k 1 2
 p2  pk  p1  p2  pk k
  P.
Највисокиот степен на pi за кој pi P е еднаков на min i ,  i  т.е. pi
mini ,  k 
P.

Од друга страна pii P и i  i  min i ,  i  , што е противречно.

Според тоа i   i , i  1, 2,..., k , од каде следува дека f b  1  f 1.

Нека n е природен број таков што f n  f 1. Имаме b  1  n f b  1  f n , т.е.
b  1  n f 1  f n.

Постојат бесконечно низи 1 ,  2 ,...,  k  за кои  i   i , па според тоа постојат бесонечно
броеви b  1  n. Оттука следува дека бројот f 1  f n има бесконечено делители, што
не е можно.

167. Предлог ИМО 2010.

Најди го најмалиот природен број n така што постојат полиноми f 1 , f 2 , ..., f n со


рационални коефициенти и притоа важи x  7  f1 x   f 2 x   ....  f n x  .
2 2 2 2

Решение. Бидејќи x 2  7  x 2  2 2  12  12  12 следува дека n  5. Ќе докажеме дека

x 2  7 не може да се претстави како збир од четири (или помалку) квадрати на полиноми


со рационални коефициенти.

Нека x 2  7  f1 x   f 2 x   f 3 x   f 4 x  . Некои од полиномите може да бидат и


2 2 2 2

нулти. Јасно е дека степените на f 1 , f 2 , f 3 , f 4 се најмногу еден.

Затоа земаме f i x   ai x  bi , i  1,2,3,4 , каде ai , bi се рационални броеви.

166
200 ТЕОРИЈА НА БРОЕВИ - подготвителни задачи

  
Добиваме x 2  7  x 2 a12  a22  a32  a42  2 xa1b1  a2 b2  a3b3  a4 b4   b12  b22  b32  b42 
Оттука важи: a12  a 22  a32  a 42  1, a1b1  a2 b2  a3b3  a4 b4  0 и b12  b22  b32  b42  7.

Од Ојлеровиот идентитет имаме

a 2
1  
 a22  a32  a42 b12  b22  b32  b42   a1b1  a2b2  a3b3  a4b4    a1b2  a2b1  a3b4  a4b3  
2 2

  a1b3  a3b1  a4b2  a2b4    a1b4  a4b1  a2b3  a3b2  .


2 2

Броевите a1b2  a2b1  a3b4  a4b3 , a1b3  a3b1  a4b2  a2b4 , a1b4  a4b1  a2b3  a3b2

се рационални, па земаме

a b c
a1b2  a2b1  a3b4  a4b3  , a1b3  a3b1  a4b2  a2b4  , a1b4  a4b1  a2b3  a3b2  ,
d d d

каде a, b, c се цели броеви и d е природен број.

Добиваме a2  b2  c2  7d 2 , односно a2  b2  c2  d 2  8d 2 . ...(1).

a b c d 
Нека d  d0 е најмалиот природен број за кој важи (1). Тогаш четворката  , , , 0 
2 2 2 2 
a 2 b2 c 2 d02 d2
исто така е решение на (1) бидејќи важи     8 0
4 4 4 4 4

т.е. a 2  b2  c2  d02  8d02 . Последното е спротивно со тоа дека d 0 е најмалиот природен


број за кој важи (1). Значи nmin  5.

168. Предлог ИМО 2004. Нека f : N  N е функција за која важи: За секој m, n  N


бројот m 2  n  се дели со f m  f n. Докажи дека f n  n, за секој природен број n.
2 2

Решение. Ако m  n  1 тогаш f 2 1  f 1 4 , од каде следува дека f 1  1.

Ќе докажеме дека f  p  1  p  1 за секој прост број p. Ако m  1, n  p  1

добиваме  f 1  f  p  1 1  p  1


2 2
т.е. 1  f  p  1  p 2 . Според тоа 1  f  p  1  p 2
или 1  f  p  1  p . Ако f  p  1  p 2  1 и ако m  p  1 и n  1 добиваме дека важи

 f  p  1  f 1  p   
од каде следува дека  p 2  1  1   p  12  1 
2 2 2
1 1
2 2

167
200 ТЕОРИЈА НА БРОЕВИ - подготвителни задачи

 
т.е. p 4  2 p 2  2   p  12  1 . Последното неравенство е еквивалентно со
2

10 p 2  2  4 p3  8 p , кое ќе докажеме дека не важи.

Имаме
A. M G .M

4 p3  8 p  3 p3  p3  4 p  4 p   3 p3  4 p 2  4 p  3 p  p 2  4 p 2  4 p 

 3 p  p 2  4 p 2  4 p  3  2  p 2  4 p 2  4 p  10 p 2  4 p  10 p 2  2.

Останува дека f  p  1  p  1. Нека избериме произволен природен број n N и нека



m  p  1. Тогаш f n    p  1
2
  p  1 2
n .
2

Имаме:

 p 1  n   p 1  f  n   2  p  1
2 2
  p  1  2  p  1 n  n 2  f n  f 2 n 
2 4 2 2 2

 p 1  f  n   2  p 1  f  n   n    f  n   n   f  n   n  
2
2  p  1 n  n 2 
2 2 2

   p  1  f  n     f  n   n   2  p  1  f  n   n  .
2 2 2

 f  n   n   2  p  1 2
 f n  n  е цел број.
Значи, за секој прост број p и n N бројот
f  n    p  1
2

Имаме

 f  n   n   2  p  1 2
 f n  n    f  n   n   2  p 1  f  n    f  n   n  
2

f  n    p  1 f  n    p  1
2 2

 f n  n
2

 2  f n  n  .
f  n    p  1
2

За доволно големо p можеме да добиеме дека f  n    p  1 е поголем број од f  n   n.


2

 f n  n
2

па оттука бројот не е цел. Следува дека f  n   n  0 т.е. f  n   n.


f  n    p  1
2

168
200 ТЕОРИЈА НА БРОЕВИ - подготвителни задачи

169. Предлог ИМО 2008. Нека n е природен број и p е прост број. Докажи дека ако

a, b, c се цели броеви за кои важи a n  pb  b n  pc  c n  pa , тогаш a  b  c.

Решение. Лесно се докажува дека ако два од броевите a, b, c се еднакви тогаш и третиот е
еднаков на нив. Затоа нека a  b  c  a.

an  bn bn  cn cn  an
Од a n  pb  b n  pc  c n  pa следува дека     p.
bc ca ab

an  bn bn  cn cn  an
Според тоа     p3.
ab bc ca

an  bn
Ако n е непарен број тогаш a n  b n и a  b имаат исти знак, па затоа  0.
ab

an  bn bn  cn cn  an
Оттука важи    0 , што не е можно бидејќи  p 3  0.
ab bc ca

Значи n е парен број. Нека НЗД  a  b, b  c, c  a   d .

Постојат цели броеви e, f , g така што a  b  ed , b  c  fd , c  a  gd и притоа


НЗД  e, f , g   1. Важи 0  a  b  b  c   c  a   d e  f  g  , од каде e  f  g  0.

Од a n  b n   pb  c  добиваме a  b  pb  c  т.е. e pf .

Аналогно се добива дека f pg и g pe. Нека претпоставиме дека p е прост број таков
што не ги дели e, f и g . Следува дека e f , f g и g e , од каде e  f  g  1.

Последното не е можно бидејќи e  f  g  0. Според тоа p дели најмалку еден од


броевите e, f и g . Без губење на општоста претпоставуваме дека p e. Постои природен
број e1 така што e  pe1. Од e pf следува дека pe1 pf т.е. e1 f . Исто така важи f g

и g e1. Бидејќи e1 e следува дека НЗД  е1 , f , g   1. Оттука e1  f  g  1.

Добиваме pe1  f  g  e  f  g  0, и бидејќи f  g 2,  1,0,1, 2 и pe1   p следува


дека p  2.

169
200 ТЕОРИЈА НА БРОЕВИ - подготвителни задачи

Значи f  g  2e1  2 од каде следува дека f  g  1. Оттука e    f  g   2 т.е.


a  b  2d . Јасно b  c  fd  d , c  a  gd  d , (знакот на f и g е ист, и е спротивен
на знакот на e ) па следува дека a  b  2  b  c  .

Бидејќи n е парен број земаме n  2k и добиваме

a n  bn   p  b  c    a k  bk  a k  bk   2  b  c   a  b. ..(1).

Бидејќи  a  b   a k  bk  од (1) следува дека a k  bk  1. Значи еден од броевите a и b


мора да биде парен, што е спротивно со a  b  2  b  c  е парен број. Од последната
контрадикција следува дека не постојат цели броеви a, b, c такви што a  b  c  a.

Останува дека a  b  c.

170. Предлог ИМО 2011. Го разгледуваме полиномот Px   x  d1 x  d 2     x  d 9 ,


каде d1 , d 2 , ...., d 9 се девет различни цели броеви. Докажи дека постои цел број N таков
што за сите цели броеви x  N бројот Px  се дели со прост број поголем од 20.

Решение. Без губење на општоста претпоставуваме дека d1 , d 2 , ...., d 9 се природни


броеви.

Ќе докажеме дека за N  d 8 е исполнет условот на задачата, каде d  maxd1 , d 2 ,..., d 8 .

Нека претпоставиме спротивно, за N  d 8 нека постои цел број x  N таков што Px 

не се дели со прост број поголем од 20. Според тоа, за секој 1  i  9 важи

x  d i  p11 p 2 2    p88 , каде  i  0 , и p i е i - тиот прост број.

Бидејќи x  d i  x  N  d 8 јасно е дека постои k  1, 2,...,8 така што p k k  d .


Од принцип на Дирихле, бидејќи имаме девет броеви x  d i и k  1, 2,...,8 следува дека
постои j  i, j 1, 2, ...,9 за кој p k k е делител на x  d j и p k k  d .
 

Нека p k k  p k k . Бидејќи p k k x  d i и p k k x  d j следува дека p k k x  d i   x  d j  

170
200 ТЕОРИЈА НА БРОЕВИ - подготвителни задачи

т.е. p k k d i  d j . Од друга страна имаме 0  d i  d j  max d i , d j  d  p k k , што е


противречност со pkk di  d j . Оттука следува дека постои N  d 8 така што за секој x  N
, полиномот P  x  има прост делител pi  p9  23  20.

2001
171. Канада 2003. Најди ги последните три цифри на бројот 20032002 .

 1  1 
Решение. Бидејќи 1000  2 3  53 добиваме  1000  1000  1  1    400 .
 2  5 

2003  3  mod1000  и добиваме 20032002  32002  mod1000 . Од


2001 2001
Ојлеровата формула
имаме 3400  3    1 mod1000  . Ќе докажеме дека 20022001  22001  352  mod 400 .
 1000

Важи 22002  24  21997  24  t  mod 24  25 , од каде следува 21997  t  mod 25 .

Бидејќи важи   25  20 следува дека 220  1 mod 25 . Според тоа

   217  22  mod 25  т.е. 22002  16  22  352  mod 400 .


99
21997  217  21980  217  220

   3352  3352  9176  mod1000  .


k
20022001  400k  352, па оттука 20032002  3400 k 352  3400
2001

Останува да се определи остатокот при делење на 9176 со 1000.

Од Биномната формула добиваме

176  175 176  174 176  3 176  176 


9176  10  1
176
 10176   10   10  ....   10   100  
2
10  1 
 1   2  173  174  175 
176  2 176  176 
 10   10  1    100  1760  1  0  760  1  241 mod1000  .
 174   175   2 

2001
Значи последните три цифри на кој завршува бројот 20032002 се 2, 4,1.

172. Предлог ИМО 1991. Најди го најголемиот природен број k така што 1991k го дели
 19921991 .
1992 1990
бројот 19901991

Решение. Ако a и n се природни броеви и  е ненегативен цел број, пишуваме a  n

171
200 ТЕОРИЈА НА БРОЕВИ - подготвителни задачи

кога n се дели со a  но не се дели со a  1 .

Лема. За секој непарен број a  3 и цел број n  0 важи

a n 1 a  1  1 и a n 1 a  1  1.
an an

Доказ. Ќе ја докажеме првата релација со помош на математичка индукција, на ист начин


се докажува и втората.

За n  0 јасно е дека важи a a  1  1 т.е. a a.

Сега нека претпоставиме дека за n  0 важи a n 1 a  1  1 .


an

Оттука постои природен број m така што a  1  m  a  1.


n 1 an

Бидејќи a n  2 не го дели a  1  1 следува a n  2 не го дели m  a n1 , т.е. a не го дели m.


an

Добиваме

a a a a


a  1a
n 1
 a  1
a n a
 a

 m  a n 1  1  1   ma n 1   m 2 a 2 n  2   m 3 a 3n 3  ...   m a a an a 
1  2 3  a
a  a a a 
 1  a  ma n 1   m 2 a 2 n  2  a 3n 3    m 3   m 4 a n 1  ....   m a a a 3n  a 3  
2 3  4 a 
 a 
 1  a  ma n 1   m 2 a 2 n  2  s  a 3n 3 .
2

Јасно е дека a  1  1  Ma n 2 т.е. a n  2 a  1a


a n 1 n 1
 1.

Бидејќи m не се дели со a следува дека a  ma n 1 не се дели со a n 3 .

a
Очигледно е дека  m 2 a 2 n  2  s  a 3n 3 се дели со a n 3 , па од последните две релации
2
следува дека a  1
a n 1
 1 не се дели со a n 3 , што требаше да се докаже.

Ако во a n 1 a  1  1 ставиме a  1991, n  1990 имаме 19911991 19921991


an
 1.
1990

Ако во a n 1 a  1  1 ставиме a  1991, n  1992 имаме 19911993 19901991


an
 1.
1992

172
200 ТЕОРИЈА НА БРОЕВИ - подготвителни задачи

Јасно k  1991 е најголемиот природен број за кој важи 19911991 19901991  19921991 .
1992 1990

173. Предлог ИМО 2011. Нека p е непарен прост број. За секој цел број a, го
a a 2
a p 1
дефинираме бројот S a    ...  . Нека m и n се цели броеви така што
1 2 p 1
m
S 3  S 4  3S 2  . Докажи дека p го дели m.
n

2 4 3 9 15 4 16
Решение. Нека p  3. Тогаш S 2    4, S 3    , S 4    12.
1 2 1 2 2 1 2

15 15
Оттука S 3  S 4  3S 2   12  12  . Значи m  15 и 3 15.
2 2

a k p 1  a    1  a    1
p 1 k k p 1 k k 1
Сега нека p  5 . Имаме: S a      .
k 1 k k 1 k k 1 k

Ќе докажеме дека
 1k 1 
1  p  p 1  p
  mod p . Бидејќи p   следува дека   е цел број.
k p k  k p k

1  p   p  1 p  2     p  k  1  1   2     k  1  1 k  1!  1
k 1 k 1
     mod p .
p  k  k! k! k! k

1 p 1  p 1 p  p p a  1 p  a p  1 .
Добиваме S a     a          a      1   a   
k k

p k 1  
k p  k 0  k  p

Оттука следува дека S a 


a  1 p  a p  1 mod p .
p

S 3  S 4  3S 2 
2 p
    
 3 p  1  3 p  4 p  1  3 1p  2 p  1 4  2 p  4 p  4
 
2p  2   mod p .
2

p p p
Од теорема на Ферма јасно е дека 2 p  2 mod p  т.е. p 2 p  2 . Значи p 2 2 p  2    
2
т.е

p
2 p
2 
2

. На крај имаме S 3  S 4  3S 2  0 mod p  .


p

173
200 ТЕОРИЈА НА БРОЕВИ - подготвителни задачи

174. Предлог ИМО 2012. Определи ги сите цели броеви m  2 така што секој природен
m m  n 
број n таков што  n  го дели биномниот коефициент  .
3 2  m  2n 

 n 
Решение. Јасно е дека 2n  m  3n. Ако m  2n или m  3n тогаш    1, од каде
 m  2n 
следува дека n  1. Во овој случај добиваме дека m  2 и m  3. Ќе ги разгледаме случаите
кога m не се дели со 2 и 3. Јасно m е непарен број, па најпрво разгледуваме кога е
сложен број, т.е. нека m   2k  1 s, s е прост број. Од условот на задачата имаме
2 1  1 2 1  1
 k   s  n   k   s. Јасно е дека  k   s  ks   k   s, па може да избереме
 3 3   2   3 3   2
 ks   ks 
n  ks. Тогаш треба да биде исполнето ks      . Значи, треба да важи
  2k  1 s  2ks   s 
 ks  k   ks  1   ks  2    ks  s  1  ks  1   ks  2    ks  s  1 .
ks    т.е. s
 
s  s  1 !  s  1!

Земаме
 ks  1   ks  2    ks  s  1  t.
 s  1!

Важи t  s  1!   1 2   (s  1   1  s  1! mod s  .


s 1
Од теорема на Вилсон имаме
 s 1!  1 mod s  . Добиваме t  1 mod s  . Значи t не се дели со s.

 n  n  n  1! n  n 1 
Сега нека m е прост број. Важи     .
 m  2n   3n  m  3n  m  1! m  2n ! 3n  m  m  2n 

 n 1 
Бидејќи m е прост број следува дека НЗД  n,3n  m   1 па оттука  3n  m   .
 m  2n 

n  n 1   n 
Останува дека n   т.е. n   . Значи m е прост број.
3n  m  m  2n   m  2n 

175. Предлог ИМО 1994. Нека M е подмножество од множеството 1, 2, ....,15 така што
производот од три различни елементи од M не е полн квадрат. Определи го
максималниот број на елементи во M .

Решение. За секое од подмножествата A1  1, 4, 9, A2  2, 6,12, A3  3, 5,15 и


A4  7, 8,14 производот на нивните елементи е полн квадрат. Бидејќи овие

174
200 ТЕОРИЈА НА БРОЕВИ - подготвителни задачи

подмножества се попарно дисјунктни, ако M има најмалку 12 елементи тогаш постои


барем едно од подмножествата A1 , A2 , A3 , A4 кое целосно се содржи во M .

Тоа е спротивно со условот на задачата. Значи M  11.

Нека претпоставиме дека M  11. Во овој случај во M не припаѓат по точно еден


елемент од A1 , A2 , A3 , A4 . Јасно 10  M бидејќи 10  Ai , i  1, 2,3, 4. Ниедно од
подмножествата 1, 4,9,2,5, 6,15 и 7,8,14 не е подмножество на M бидејќи
1 4  9  62 , 2  5 10  102 ,6 15 10  302 ,7  8 14  282. Оттука 3,12  M . Според тоа
ниедно од подмножествата 1 , 4 , 9 , 2,6, 5,15 и 7,8,14 не е подмножество на M .
Значи M  15  6  9. Последното е спротивно со M  11.

Следува дека M  10 т.е. M  10 бидејќи M  1, 4,5,6,7,10,11,12,13,14 е 10 


елементно подмножество кое ги исполнува условите на задачата.

176. Предлог ИМО 1996. Конечната низа од цели броеви a0 , a1 ,...., a n ја нарекуваме
квадратна ако за секој i 1, 2, ...., n важи ai  ai 1  i .
2

а) Докажи дека за секои два цели броеви b и c , постои природен број n и квадратна
низа таква што a0  b и a n  c.

б) Најди го најмалиот природен број n за кој постои квадратна низа таква што a0  0 и
a n  1996.

Решение.

а) Треба да важи a0  b, a1  12  b, a2  12  2 2  b,..., an  12  2 2  ...  n 2  b  c .

Од произволноста на b и c доволно е да докажеме дека постои квадратна низа за која

an  a  c  b  12  2 2  ...  n 2 . Оттука ќе следува ai  12  2 2  ...  i 2 , a0  0.

Значи, за произволен цел број a треба да докажеме дека постои природен број n и
квадратна низа за која a0  0, a n  a.

Бидејќи n  12  n  22  n  32  n  42  4 забележуваме дека ако условот на


задачата е исполнет за даден цел број a тогаш е исполнет и за бројот a  4.

175
200 ТЕОРИЈА НА БРОЕВИ - подготвителни задачи

Во тој случај важи a0  0, an 4  a  4.

Според тоа доволно е да докажеме дека за a  0,1,2, 3 е исполнет условот на задачата.

Ако a  3 тогаш 3  12  2 2 , т.е. n  2. Ако a  2 тогаш 2  12  2 2  32  4 2 , т.е.


n  4.

Ако a  1 тогаш 1  1 , т.е. n  1.


2

nn  12n  1 nn  12n  1


б) Бидејќи a n  12  2 2  ...  n 2  добиваме 1996  т.е.
6 6

n  18. Бројот a18  1  2  ...  18 има иста парност со бројот 2009  12  2 2  ...  18 2
2 2 2

па оттука a18  1996. Значи n  19.

Од 12  2 2  ...  19 2  2470  1996  2  237  1996  2  4 2  5 2  14 2  добиваме

 
1996  12  2 2  ...  19 2  2 4 2  5 2  14 2  12  2 2  32  4 2  5 2  6 2  ....  132  14 2  ..  19 2
т.е. n  19.

177. СМО 2007. Одреди ги сите парови од природни броеви x, n  кои се решение на
равенката x 3  2 x  1  2 n.

Решение. Ако n  1 тогаш ја добиваме равенката x3  2 x  1 која нема решение во


множеството природни броеви. За n  2 добиваме x  1 т.е. 1, 2 е еден пар решение на
равенката. Ќе докажеме дека за n  3 равенката нема решение.

 
Бидејќи x x 2  2  2 n  1 е непарен број следува дека x е непарен број.

Оттука x 2  1mod 8 , односно x 2  2  3 mod 8. Јасно е дека за n  3

важи 2 n  1   1mod 8 , па според тоа 3x  1mod 8 . Следува дека x  5 mod 8.

Ако x  0  mod 3 тогаш 2n  1 mod 3 . Ако x  1, 2  mod 3 тогаш x 2  1 mod 3 , од каде


следува дека 3 x 2  2. Оттука 2n  1 mod 3 , односно n е парен број.


Ако на двете страни на дадената равенка додадеме 2 добиваме x  1 x 2  x  3  2 n  2 . 
Бидејќи n е парен број следува дека 2 n е полн квадрат.

176
200 ТЕОРИЈА НА БРОЕВИ - подготвителни задачи


Ако p е непарен прост делител на бројот x  1 x 2  x  3 следува дека конгруенцијата 
x 2  2 mod p  има решение x  2 k , каде k  . Според тоа  2 е квадратен остаток на
n
2

  2
секој непарен прост делител p на x  1 x 2  x  3 . Значи  
  1.
 p 

  2    1  2  p 1 p 1  4 p 5
2 2

       1 2  1 8   1
p
Добиваме 1   8 , од каде следува дека
 p   p  p 

p 2  4 p  5  16s. Лесно се проверува дека p  1mod 8 или p  3 mod 8.

   
Значи x  1 x 2  x  3  1mod 8 или x  1 x 2  x  3  3 mod 8. .....(1).

Од друга страна, бидејќи x  5 mod 8 се добива дека x  1 x 2  x  3  7 mod 8 ,  


што е спротивно од (1). Останува дека парот 1, 2 е единствено решение.

178. Ако x е природен број докажи дека предпоследната цифра на секој повеќецифрен
прост делител на бројот 5x 2  1 е парна.

Решение. Нека p е прост број таков што p 5 x 2  1. Следува 5x   5 mod p , односно
2

 5   5    1  5  p 1  5 
   1. Од Теорема 7.2. б) добиваме         1 2  .
 p   p   p  p   p

 5  p  p 1 51
Од правилото на квадратни реципрочности имаме:      1 2 2  1.

  
p 5

 5   p  5  
p 1 p
Според тоа     . Значи,     1 2    1.
 p  5   p  5

Можни се два случаи:


p 1
 p p 1
 p
1)  1 2  1 ,    1 2)  1 2  1 ,    1.
5 5
p 1 p 1
1) Од  1 2  1 следува дека  2k  1, p  4k  3, p  3 mod 4.
2

177
200 ТЕОРИЈА НА БРОЕВИ - подготвителни задачи

 p
Од    1 следува дека конгруентната равенка x 2  p mod 5 нема решение.
5

Значи p  2 mod 5 или p  3 mod 5. Ако p  2 mod 5 тогаш го решаваме системот

 p  3  mod 4 
 . Добиваме p  4s  3, p  5t  2, s, t  N0 .
 p  2  mod 5 

Важи 4s  3  5t  2 т.е. 4  s  1  5  t  1 . Јасно е дека s  1  5m, од каде следува


 p  3  mod 4 
p  20m  7 , т.е. p  7  mod 20  . Аналогно го решаваме и системот 
 p  3  mod 5 

од каде добиваме p  3  mod 20  .

p 1
2) Сега  2s , и p  0,1, 4  mod 5 бидејќи во тој случај x 2  p  mod 5 има решение.
2
 p  1 mod 4   p  1 mod 4   p  1 mod 4 
По решавање на системите  ,  и 
 p  0  mod 5   p  1 mod 5   p  4  mod 5 

добиваме p  5  mod 20  , p  1 mod 20  и p  9  mod 20  .

Значи, p  1,3,5,7,9  mod 20  , од каде е јасно дека предпоследната цифра на p е парен


број.

179. Докажи дека за n  5, f n  f n1  1 има најмалку n 1 прости делители.


( Бројот f n  2  1 е број на Ферма).
2n

Решение. Имаме f n1  f n  1  2 2


n 1
 22  1  22
n
   2  2 1  2   2
n 2
2n 2n 1
2
2n
  
2
 1  22
n 1 2


 22  22
n n 1
 12  2  1.
2n 2n 1


Значи f n1  f n  1  2 2  2 2n1
n
 1 f n  f n1  1  an  f n  f n1  1 , каде an  2 2  2 2
n n 1
1 и
јасно е дека an  1, за секој природен број n. Со помош на математичка индукција ќе
докажеме дека f n  f n1  1 има најмалку n  1 прости делители.

178
200 ТЕОРИЈА НА БРОЕВИ - подготвителни задачи

За n  5 добиваме f 5  f 4  1  3  7  13  97  241  673 , односно f 5  f 4  1 има шест прости


делители. Сега претпоставуваме дека за секој k  5 бројот f k  f k 1  1 има најмалку
k  1 прости делители.

Важи f k 1  f k  1  ak  f k  f k 1  1 . Ќе докажеме дека a k и f k  f k 1  1 се заемно


прости.

НЗД  f k  f k 1  1, ak   НЗД  f k  f k 1  1, f k  f k 1  1  НЗД  f k  f k 1  1, f k  f k 1  1  f k  f k 1  1 


 НЗД  f k  f k 1  1, 2 f k 1  2   НЗД f k  f k 1  1, 2  2 2
k 1
  1.
Според тоа секој прост делител на a k е различен од простите делители на бројот
f k  f k 1  1. Значи f k 1  f k  1 има најмалку k  2 прости делители.

180. Предлог ИМО 1992. Дали постои множество од 1992 природни броеви такво што
збирот на елементите на секое негово подмножество е квадрат, куб или повисок степен на
некој природен број.

Решение. Најпрво со помош на математичка индукција ќе го докажеме следното својство:

За секој природен број n постои природен број d така што секој елемент од
множеството d ,2d ,3d ,...nd  е квадрат, куб или повисок степен на некој природен број.

За n  1 е јасно.

Претпоставуваме точност за природен број n т.е. нека важи d  m1k1 ,2d  m2 2 ,.....nd  mnkn
k

За n  1 избираме D  d n  1k d k каде k  НЗС k1 , k 2 ,...k n .

Множеството D,2D,3D...nD, n  1D ги исполнува условите од горното својство.

Ако s  1,2,...n важи sD  sd n  1 d k  msks n  1


k
 k'
  d   m  n  1 d 
ks
k'
ks
s
k' k'
ks

т.е. sD е k s  ти степен на природен број. Исто така n  1D  n  1d k 1 .

Со тоа својството е докажано.

179
200 ТЕОРИЈА НА БРОЕВИ - подготвителни задачи

1992 1993
Според тоа, за n  постои природен број d така што секој елемент на
2
K  d , 2d ,..., nd  е квадрат, куб или повисок степен на некој природен број. На крај
избираме M  d , 2d ,...,1992d  е бараното множество. Збирот на елементите на секое
негово подмножество е број од множеството d , 2d ,....1992d ,1993d ,..., d  2d  ...  1992d 
кое е еднакво на множеството K .

181. Предлог ИМО 2008. Нека а0 , а1 , а2 ,... е низа од природни броеви така што
најголемиот заеднички делител на два последователни членови е поголем од претходниот
член, односно НЗД  аi , ai 1   ai 1. Докажи дека аn  2n за сите n  0.

Решение. Со помош на математичка индукција по n ќе докажеме дека аn  2n за сите


n  0. Јасно е дека an  an1 , за секој природен број n.

Важи a0  1. Нека НЗД  a1 , a2   d  a0  1 од каде следува дека а1  2 и а2  kd  2  2  4.

Нека претпоставиме дека за i 3, 4,..., n  1 важи аi  2i. Ќе докажеме дека an  2n. Нека
претпоставиме спротивно, т.е. нека an  2n. Тогаш важи 2n1  an1  an  2n. Сега, нека
НЗД  аn , an1   d  an2  2n2 и нека аn  kd  2n , k  2. Ако k  4 тогаш бидејќи d  2n2
добиваме дека аn  kd  22  2n2  2n , што е спротивно со претпоставката дека аn  2n.

Значи, останува дека k  2 или k  3.

1) Ако k  2 тогаш аn  2d , што повлекува дека аn1  d  2n1. Оттука имаме дека
аn  2d  2n , што е спротивно со претпоставката дека аn  2n.

2) Ако k  3 тогаш аn  3d . Оттука следува аn1  d или аn1  2d . Ако аn1  d имаме
контрадикција како во случај 1). Значи аn1  2d .

Нека НЗД  an1 , an2   НЗД  2d , an2   s  an3  2n3. Бидејќи 2d  an2 следува
дека 2d  st , t  2. Бидејќи st  2d  аn1  an  2n и t  2 следува дека s  2n1.

Од аn1  2d  st  2n1 , ако t  4 тогаш s  2n3. Последното не е можно бидејќи


добивме дека s  2n3. Останува дека t  2 или t  3. Ако t  2 тогаш s  d , оттука

180
200 ТЕОРИЈА НА БРОЕВИ - подготвителни задачи

НЗД  2d , an2   d , т.е. аn2  d . Имаме НЗД  an , an1   НЗД  3d , 2d   d  an2  d ,


што не е можно. Ако t  3 тогаш 2d  3s.

Добиваме НЗД  an1 , an2   НЗД  3s, an2   s. Ако an2  s  2n2 тогаш
3 n2 9
2d  3  2 n  2  d 
 2 . Оттука следува дека an  3d   2n 2  2n , што е
2 2
спротивно на претпоставката дека аn  2n. Ако an2  2s  2n2 тогаш
3
d s  2s  an 2 . Бидејќи НЗД  an , an1   d  an2  d , добиваме контрадикција.
2

Значи останува дека важи an  2n за секој n  0,1, 2..

182. Предлог ИМО 2011. Нека f е функција од множеството цели броеви во


множеството природни броеви таква што за секои два цели броеви m и n важи

f m  n f m  f n. Ако за сите m, n важи f m  f n тогаш докажи дека f m f n.

Решение. Во условот f m  n f m  f n заменуваме n  0 и добиваме дека


f  m  f  m   f  0  . Оттука f m f 0. Ако замениме m  0 следува f  n f n .

Од исти причини важи и f n  f  n  , од каде следува f n  f  n, односно дадената

функција е парна. Имаме f  m  n   f  m   n   f  m   f  n   f  m   f  n  , и бидејќи

f  m   f  n  добиваме f  m  n   f  n   f  m  . ..(1).

Исто така важи f  n   f  m  n  m  f  m  n   f  m  , па затоа ќе разгледаме два можни


случаи:

1) f  n   f  m  n   f  m  ,

2) f  n   f  m   f  m  n  .

1) Од (1) добиваме f  m  n   f  n   f  m    f  m  n   f  m    f  m  т.е. 2 f  m   0 ,


што не е можно бидејќи функцијата добива позитивни вредности.

2) Од f  m   f  n   f  m  n  имаме f  n   f  m   f  m  n   f  n   f  m  n   f  m  n 

181
200 ТЕОРИЈА НА БРОЕВИ - подготвителни задачи

т.е. 2 f  m  n   0, што исто така не е можно.

Останува дека f  m  n   f  m   0  f  m  n   f  m  .

На крај добиваме f  m  f  m  n   f  n   f  m  f  m   f  n   f  m  f  n  .

183. Предлог ИМО 2007. Нека b, n  1 се цели броеви. Претпоставуваме дека за секој
k  1 постои цел број a k така што b  a kn е делив со k . Докажи дека b  A n за некој цел
број A.

Решение. Нека b  p11 p 2 2   p k k , p1 , p 2 ,..., p k се различни прости броеви, е канонична


факторизација на природниот број b. Доволно е да докажеме дека степените  i се делат
со n. Нека k  b 2 . Од условот на задачата добиваме pi2 i a kn  b , па според тоа pi i a kn .

Бидејќи за секој 1  i  k важи 1   i  2 i имаме p  i 1 p 2 i , од каде следува дека

pi i 1 a kn  b. Јасно е дека a kn не се дели со pi i 1 бидејќи во тој случај b ќе се дели со


pi i 1 , што не е можно. Значи, највисокиот степен на p i кој го дели a kn е pi i .

Бидејќи akn е точен n  ти степен следува дека  i се дели со n.

184. ИМО 2003. Нека p е прост број. Докажи дека постои прост број q така што за секој
цел број n бројот n p  p не се дели со q.

Решение. Ако p  2 земаме q  5. Во тој случај n 2  2 не се дели со 5 , бидејќи


n 2  0,1, 4 mod 5. Нека p  3. Го разгледуваме изразот

 
p p  1   p  1 p p1  p p2  ...  p 2  p  1 .

Јасно е дека s  p p 1  p p 2  ...  p 2  p  1 е непарен број (збир од непарен број на


 
непарни броеви) и притоа s  p  1 mod p 2 . Оттука постои непарен прост делител q на s
, кој не е во облик kp  1. Ако за секој прост делител q i на s важи qi  1mod p 2  тогаш
2

  
јасно е дека s  q11  q2 2    qr r  1 mod p 2 , што е спротивно со s  p  1 mod p 2 . 

182
200 ТЕОРИЈА НА БРОЕВИ - подготвителни задачи

Ќе докажиме дека q го задоволува условот на задачата. Нека претпоставиме спротивно,


тогаш постои цел број n така што n p  p mod q . Тогаш n p   p
 p p 1mod q  т.е.
n p  1mod q . ..(1). Јасно, n не се дели со q , па од малата теорема на Ферма следува
2

дека n q 1  1mod q . ..(2). Од (1) и (2) добиваме n НЗД  p , q 1   1mod q . Забележуваме дека
2

q  1 не се дели со 
p 2 , бидејќи во тој случај q  1 mod p 2 , што не е можно од 
дефинираноста на q. Останува  2

НЗД p , q 1  p или  
НЗД p 2 , q  1  1. Ако
 2

НЗД p , q 1  p тогаш n  1mod q . Од n  p  mod q  и n  1 mod q  следува дека
p p p

p  1 mod q  .

Од друга страна s  1  1  ..  1  p  1 mod q  , што не е можно бидејќи q s.

Значи n  1mod q  од каде повторно се добива дека p  1 mod q  , што не е можно.

185. Предлог ИМО 1994. Нека k е природен број. Докажи дека постојат бесконечно
многу полни квадрати од облик n  2 k  7 , каде n е природен број.

Решение. Најпрво со помош на математичка индукција ќе докажеме дека за секој


природен број k постои природен број a k таков што a k2  7 mod 2 k . Од последната
конгруенција ќе следува дека постои природен број аk таков што ak2  n  2k  7.

Потоа лесно се утврдува дека броевите a k  r  2 k , r  1,2,... се бесконечно многу природни


броеви за кои a k  r  2 k   7 mod 2 k . За k  1 доволно е да избериме a1  1.
2

Сега претпоставуваме дека 


a k2  7 mod 2 k .  Ако важи 
a k2  7 mod 2 k 1  земаме
a k 1  a k .

Ако a k2  7 не се дели со 2 k 1 , од a k2  7 mod 2 k  следува дека постои непарен број s


таков што a k2  7  2 k  s. Значи ak2  2 k 2s1  1  7  2 k  7 mod 2 k 1  .

Земаме a k 1  a k  2 k 1 и добиваме дека ak21  ak2  ak  2 k  2 2k 2  7 mod 2 k 1 .

Оттука јасно е дека постои природен број n така што ak21  n  2k 1  7.

186. Виетнам 1997. Докажи дека за секој природен број n постои природен број k така
што 19 k  97 се дели со 2 n .

183
200 ТЕОРИЈА НА БРОЕВИ - подготвителни задачи

Решение. За n  1, 2 избираме k  2, бидејќи 192  97  361  97  264 се дели со 2 и 4.

Со помош на математичка индукција најпрво ќе докажеме дека за n  3 важи


2n  2
19  1  2 mn , m n е непарен.
n

За n  3 имаме 192 1  361  8  45  23  45. Нека претпоставиме дека за секој природен


k 2
број 2  k  n важи 192  1  2k mk , каде mk е непарен број.

     
k 1 k 2 2 2
Сега, 192  1  192  1  2k mk  1  1  2k 1 mk 2k 1 mk  1  2k 1 mk 1

каде mk 1  2k 1 mk 2  mk е непарен број.

Повторно со математичка индукција ќе докажеме дека за секој природен број n постои


природен број k така што 19 k  97 се дели со 2 n .

Ако n  3 избираме k  2. Нека претпоставиме дека за секој 4  m  n постои природен


број km така што 19km  97 се дели со 2m. Оттука постои природен број k n така што
19kn  97 се дели со 2n. Според тоа постои природен број t така што 19kn  97  2n t. Ако
t е парен број тогаш 19kn  97 се дели со 2n1 , од каде следува дека kn1  k n .

Ако t е непарен број тогаш избираме kn1  kn  2n2 и добиваме:

19kn1  97  192
n 2

19 kn
  n 2

 n 2

 n 2
 97  97 192  1  192  2n t  97  2n mn  2n 192 t  97mn . 
n2
Јасно е дека 192  t  97  mn е парен број бидејќи t и mn се непарни броеви.

187. Предлог ИМО 2007. Нека k е природен број. Докажи дека бројот 4k 2  1  
2
има
природен делител од облик 8kn  1 ако и само ако k е парен број.

Решение. Ќе ја користиме следната лема:

Лема. Ако x и y се произволни природни броеви тогаш бројот 4 xy  1 го дели бројот


4 x 2
 2
 1 ако и само ако x  y.

Доказ. Ако x  y тогаш 4 xy  1  4 x 2  1 го дели бројот 4 x 2  1 .  


2

За парот од природни броеви x, y  x  y  велиме дека е добар ако 4 xy  1 го дели


бројот 4 x  1 2

2

184
200 ТЕОРИЈА НА БРОЕВИ - подготвителни задачи

Својство 1. Ако x, y  x  y  е добар пар тогаш и парот  y, x  е добар.

 
Бидејќи 4 xy  1mod 4 xy  1 и 4 x 2  1  0 mod 4 xy  1 имаме:
2

4 y 2
 
 1  4 y 2  4 xy 
2
  4 y 1  4x 
2 2 2 2 2
 
 16 y 4 4 x 2  1  0 mod 4 xy  1.
2

Својство 2. Ако x, y  x  y  е добар пар тогаш постои природен број z  x така
што x, z  е добар пар.

Нека k 
4 x 12

2

 
. Јасно k 4 xy  1  4 x 2  1 и  1  4 xy  1mod 4 x  .
2

4 xy  1

 
Добиваме k   k    1  k 4 xy  1   4 x 2  1  16 x 4  8x 2  1   1mod 4 x .
2

Оттука следува дека постои природен број z така што k  1  4 xz , т.е. k  4 xz  1.

Од x  y имаме 4 xz  1  k 
4 x 1
2
 
2


4x 2  1 
2

 4 x 2  1 , од каде следува дека z  x.


4 xy  1 4x 2  1

Исто така јасно е дека 4 xz  1 го дели бројот 4 x 2  1 , односно x, z  е добар пар. 
2

Сега нека претпоставиме дека постои најмалку еден добар пар , и избираме x, y  за добар
пар за кој 2 x  y добива најмала вредност. Ако x  y тогаш од својство 2 следува дека
постои природен број z за кој x, z  е добар пар и z  y. Тогаш 2 x  z  2 x  y , што е
противречно со минималноста на 2 x  y .

Ако y  x , од својство 1 следува дека  y, x  е добар пар па важи 2 y  x  2 x  y , што


исто така не е можно. Со тоа лемата е докажана.

Сега, користејќи ја лемата, ако ставиме x  k , y  2n добиваме дека бројот 8kn  1 го дели

 
2
бројот 4k 2  1 ако и само ако k  2n.

188. Предлог ИМО 2007. Најди ги сите парови k, n  од природни броеви за кои 7 k  3n
го дели k 4  n 2 .

Решение. Нека парот k, n  ги задоволува условите на задачата. Бидејќи 7 k  3n е парен


број следува дека и k 4  n 2 е парен број, од каде следува дека k и n имаат иста парност.
Ако k и n се непарни броеви тогаш k 4  1mod 4 и n 2  1mod 4 , т.е. важи

185
200 ТЕОРИЈА НА БРОЕВИ - подготвителни задачи

k 4  n 2  2 mod 4. Од друга страна важи 7 k  3n   1   1  1  1  0 mod 4 , што


k n

не е можно, бидејќи 7 k  3n се дели со 4 додека k 4  n 2 не се дели. Значи k и n се


7 a  3b
парни броеви, и земаме k  2a, n  2b. Тогаш 7 k  3n  7 2 a  32b 
2
 
 2 7 a  3b , од

каде следува дека бројот 27 a  3b  е делител на 7 k  3n. Значи 27 a  3b  7 k  3n k 4  n 2


односно 7 a  3b 8a 4  2b 2 . Според тоа имаме 7 a  3b  8a 4  2b 2 .

Со помош на математичка индукција ќе докажеме дека 8a 4  7 a за a  4 , 2b 2  3b за


b  1 и 2b 2  9  3b за b  3.

1 5
За a  4 важи 8  a4  8  44  2048  2401  74. Од a  4 следува дека 1   .
a 4

Нека претпоставиме дека 8a 4  7a за a  4.

 a  1
4 4 4
a  1 a 5
Добиваме 8a  1  8a 4   a 625
  7  1    7     7   7 a  7  7 a 1.
4

 a   a 4 256

Ако b  1 важи 2b2  2  31  3b , за b  2 важи 2b2  8  9  32  3b. Ќе докажеме дека

2b2  9  3b за b  3 , од каде ќе следува дека и 2b2  2b2  9  3b.

1 4
За b  3 важи 1   , и од индуктивната претпоставка 2b2  9  3b имаме:
b 3

 b 1  b  1
2 2

2b  1  9  2b  1  9
2 2 2 9 

  b  1  2  2   2b  9 
2
 
 b   b   b 
2
4 16
 3b     3b   3b  3  3b 1.
 
3 9

За a  4 добиваме 7a  3b  8a4  2b2 што е спротивно на условот 7a  3b  8a 4  2b2 .

Значи a  3, и ги разгледуваме следните случаи:

Случај 1: Нека a  1.

Тогаш k  2a  2 и треба да важи 8  2b2  7  3b т.е. 2b2  1  3b. Последната неравенка


важи за b  2. Ако b  1 тогаш n  2b  2 и ако замениме во условот на задачата добиваме
дека 72  32 24  22 т.е. 40 20 , што не е можно.Ако b  2 тогаш n  4, и ако замениме во

186
200 ТЕОРИЈА НА БРОЕВИ - подготвителни задачи

условот на задачата имаме 72  34 24  42 т.е. 32 32 , кое е исполнето. Следува парот

 2, 4  е решение.
Случај 2: Нека a  2.

Тогаш k  4 и k 4  n2  256  4b2  74  3n  492  32b  49  3b   49  3b  .

Најмалата вредност на множителот 49  3b изнесува 22 и се добива за b  3.

Следува дека 128  2b2  11  49  3b   539  11 3b , што не е можно бидејќи 3b  2b2 .

Случај 3: Нека a  3.

Тогаш k  6 и k 4  n2  1296  4b2  76  3n  3432  32b  343  3b   343  3b  .

На ист начин како погоре важи 343  3b  100 па оттука следува

1296  4b2  100   343  3b  т.е. 324  b2  25   343  3b  .

Очигледно е дека последна неравенка нема решение. На крај, останува дека  k , n    2, 4  е


единствено решение.

189. Предлог ИМО 2006. За x  0,1 нека y  0,1 е број чија n -та цифра после
n
децималната запирка е 2 -тата цифра после децималната запирка на бројот x . Докажи
дека ако x е рационален број тогаш и y е рационален број.

Решение. Нека f n  е n - тата цифра после децималната запирка на бројот x и

g  n  е n - тата цифра после децималната запирка на бројот y . Тогаш за децималниот


запис на y важи g n  f 2 n . 
Од рационалноста на x следува дека неговите цифри периодично се повторуваат
почнувајќи од некоја цифра. Значи, постои природен број m - должината на периодот на
x и природен број x 0 така што за секој x  x0 важи f x   f x  m.

Со помош на математичка индукција лесно се докажува дека за сите x, y  x0 за кои

187
200 ТЕОРИЈА НА БРОЕВИ - подготвителни задачи

x  y mod m следува дека f x   f  y . Ако x  y  mod m  тогаш постои природен број t


така што x  tm  y. Ако t  1 имаме f  x   f  y  m   f  y  . Нека претпоставиме дека
f  x   f  y  за фиксирана вредност t. Сега за x   t  1 m  y добиваме дека важи:

x  y  mod m  и f  x   f  t  1 m  y   f  tm  y  m  f  tm  y  f ( y).
Нека m  2 k d , каде d е непарен број. Бидејќи d 2 d   1 и 2 k 2 x за k  x, следува дека

m  2 k d 2 x 2 d   1  2 x  d   2 x т.е. 2 x  2 x d  mod m.

   
Според тоа за x  maxx0 , k  важи f 2 x d   f 2 x т.е. g x   d   g x .

Значи g x  е периодична функција со период   d  , од каде следува дека y е рационален


број.

190. ИМО 2006. Определи ги сите парови цели броеви x, y  така што

1  2 x  22 x1  y 2 .

Решение. Ако x  1 тогаш y 2 не е цел број, додека за x  1 имаме y 2  2 , равенката


нема целобројно решение. Оттука x  0. Ако x  0 добиваме y 2  4 т.е. 0,  2 е
2 x 1
решение на равенката. За x  1, x  2 лесно се проверува дека 1  2  2 x
y 2
нема
решение.

Нека x, y  е решение кога x  3. Без губење на општоста земаме дека y  0.

Имаме 2x 1  2  2 x    y  1 y  1 . Јасно е дека y е непарен број па следува дека


НЗД  y  1, y  1  НЗД  y  1, y  1  y  1  НЗД  2, y  1  2. Оттука едниот од броевите
y  1 и y  1 е делив со 2 но не е делив со 4 , додека другиот број е делив со 2 x1 но не е
делив со 2 x.

Според тоа y  2x 1 m  k , каде k  1 и m е непарен број. Ако замениме во дадената р-ка


имаме 2x 1  2  2x   y 2  1  22 x 2 m2  2x mk  k 2  1. Користиме k 2  1  0 и по делење на
последната равенка со 2 x ја добиваме равенката 2 x 2  m2  8  1  mk. Јасно е дека не е
можно m  1, па следува дека m  3. Тогаш 2 x 2  m2  8  0 т.е. 1  mk  0 , од каде k  1.

188
200 ТЕОРИЈА НА БРОЕВИ - подготвителни задачи

Користејќи x  3 добиваме m  1  2 x2  m2  8  2  m2  8 . Последната неравенка е можна


само ако m  3, бидејќи за m  4 важи m  1  2  m2  8 .

Значи, 2x2  m2  8  2 x2  4  x  4, y  23. Решенија на равенката се:  0,  2  и  4,  23 .

191. Канада 2009. Најди ги сите парови од цели броеви a, b  така што 3a  7 b е полн
квадрат.

Решение. Ако еден од броевите a и b е негативен тогаш 3a  7 b е збир од природен


број и број помал од еден. Ако a  0 и b  0 тогаш a  a1 , b  b1 , каде a, b  N .

1 1 1 1 10
Важи 3a  7 b      , што не е можно.
3a1 7 b1 3 7 21

Останува дека a, b  0 . Нека 3a  7 b  n 2 , каде n  N . Разгледувајќи ја равенката модул


4 имаме n 2   1a   1b mod 4 . Бидејќи n 2  0,1mod 4 следува дека a и b имаат
различна парност.

1) Нека a е парен број, b е непарен број.

  
Имаме a  2k , k  N . Оттука 7 b  n  3k  n  3k . Јасно, постојат k1 , k 2  N така што

n  3k  7 k1 и n  3k  7 k2 , за кои важи k1  k2 и k1  k 2  b . Со одземање се добива


равенката 2  3k  7k1  7k2 k1  1 . Бидејќи 2  3k не се дели со 7 следува дека k1  0 , k 2  b .

Оттука n  3k  70  n  3k  1. Добиваме 32 k  7b   3k  1  32k  2  3k  1 7b  1  2  3k.


2

Важи  7  1  7b1  7b2  ...  7  1  2  3k  7b1  7b2  ...  7  1  3k 1.

Ако k  1 тогаш a  2k  2, b  1, т.е. парот  2,1 е едно решение.

Ако k  2 тогаш b  2 и важи 3 7b1  7b2  ...  7  1. Од друга страна е исполнето

7b1  7b2  ...  7  1  1  1  ...  1  b  mod 3 . Значи b  3c, за некој природен број c.

189
200 ТЕОРИЈА НА БРОЕВИ - подготвителни задачи

Ја добиваме равенката 73c  1  2  3k   7c  1 72c  7c  1  2  3k. ..(1). Јасно е дека


7c  1
7c  1  1  1  0  mod 6  ,  N, како и 72c  7c  1  1  1  1  0  mod 3 односно
6
7 2 c  7c  1  7c  1   7 2c  7c  1  k  2
 N . Од (1) следува     3 . .(2). Ако 7  1  6d тогаш важи
c

3  6   3 
7 2c  7c  1  7c  1 7 2c  7c  1 
 12d 2  6d  1. Оттука НЗД  , 2

  НЗД d ,12d  6d  1  1. 
3  6 3 

7c  1 7 3c  1 7 3  1
Од (2) следува дека  1 т.е. c  1, b  3c  3. На крај имаме 3k    171 ,
6 2 2
од каде следува дека k не е природен број.

2) Нека a е непарен број, b е парен број.

Сега b  2k , k  N . Следува 3a  n2  72k   n  7k  n  7k  ..(1). Постојат природни броеви


k1 , k2 , k1  k2 за кои важи n  7k  3k1 и n  7k  3k2 и притоа k1  k2  a.

Имаме 2  7k  3k1  3k2 k1  1 , и бидејќи 2  7k не се дели со 3 следува дека k1  0, од каде


следува k2  a. Важи n  7k  3k1  7k  1. Ако замениме во (1) ја добиваме равенката

3a  2  7k  1. Јасно е дека 3a  1 mod 7  од каде следува дека a  6s, s  N . Последното не


е можно бидејќи a е непарен број. Останува k  0 од каде добиваме a  1, n  2, b  0.

Значи паровите  2,1 и 1, 0  се целобројни решенија на дадената равенка.

192. БМО 2013. Определи ги сите природни броеви x, y и z така што x5  4 y  2013z.

Решение. Бидејќи 3 2013 имаме x5  4 y  0  mod 3 . Значи x5  1 mod 3 , од каде


следува дека x  1 mod 3 . Од теорема на Ферма важи x10  1 mod11 кога x не се дели
со 11 или x  0  mod11 кога x се дели со 11. Значи x5  0, 1 mod11 . Сега, 11 2013
па имаме 4 y  0, 1 mod11 .

Бидејќи не е можно 4 y  0,  1 mod11 , следува дека 4 y  1 mod11 , од каде добиваме


y  5k , k  N .

190
200 ТЕОРИЈА НА БРОЕВИ - подготвителни задачи

a  b .
5
a 5  b5 a  b
Од неревенството 5  добиваме дека a5  b5 
2 2 16

 a  b
5

  a  b  бидејќи  a  b   16 , во случај кога a  x  1, b  4k  4.


2 3
Јасно е
16

 x5  45k 
Значи важи x5  45k   x  4k  . Имаме x5  45k   x  4k  
2
k 
 3z 11z  61z.
 x4 

x5  45k
Бидејќи x  4k  и 3 x  4k добиваме дека x  4k  3z или x  4k  33z.
x  4k

1) Ако x  4k  3z тогаш x  3z  4k , и добиваме  3z  4k   45k  3z 11z  61z т.е.


5

35 z  5  34 z  4k  10  33z  42k 10  32 z  43k  5  3z  44k  45k  45k  3z 11z  61z. ..(1).

Ако равенката (1) ја поделиме со 3z и добиената равенка ја разгледуваме модул три


имаме 5  4k  11z  61z т.е. 11z  61z  2  mod 3 . Останува 11z  2  mod 3 , што е можно само
ако z е непарен број.

Ако k  1 тогаш x  3z  4k  3  4  7  mod8 , односно x5  45k  7 5 0  7  mod8 .

Ако k  2 тогаш важи x  3z  4k  3  0  3  mod8 , од каде x5  45k  3 5 0  3  mod8 .

Од друга страна важи 2013z  5z  5  mod8 . Добиваме противречност.

2) Ако x  4k  33z имаме  33z  4k   45k  2013z.


5

335 z  5  334 z  4k  10  333 z  42k 10  332 z  43k  5  33z  44k  45k  45k  33z  61z. ..(2).

По делење на равенката (2) со 33z добиваме 5  44k  61z  mod 3 . Последната конгруенција
не е можна бидејќи 5  44 k  2  mod 3 , додека 61z  1 mod 3 .

Според тоа равенката нема решение во множеството природни броеви.

191
200 ТЕОРИЈА НА БРОЕВИ - подготвителни задачи

193. ИМО 1997. Најди ги сите парови од природни броеви x, y за кои важи x y  y x .
2

Решение. Најпрво забележуваме дека x и y имаат исти прости делители.

Нека x  p11 p 2 2    p k k и y  p11 p 2 2    p k k се каноничните факторизации на двата броја.

p  
  
2 k 
p12 1 p22 2  pk p11 p22  pk k
Важи 1
1 p22  pkk  p11 p22  pkk . Следува дека

i p p  pk x m
i 1, 2,..., k  ,
1 2 k
2 1
p22 2  pk2 k p2  pk k
1  2
pii p1  pii p1 , т.е.  2 1 22  2  , за секој
i p1 p2  pk2 
1 2
y n k

m
m, n  N и НЗД  m, n   1. Бидејќи е нескратлива дропка важи m  i .
n
1 2 k
i i
Нека t  p1m p2m  pkm . Јасно x  t m , y  t n бидејќи  .
m n

Ако t  1 тогаш парот 1,1 е решение на равенката. Ако t  1 тогаш дадената равенка

добива облик t mt  t nt т.е. m  t 2n  n  t m . Очигледно m  n , па разгледуваме два случаи:


2n m

n
1) Нека m  n. Тогаш t 2nm  , n  m  t 2nm . Земаме 2n  m  s и добиваме
m
  од каде s  m 2t s  1 . ...(1). Бидејќи t  1 важи 2t s  1  s , па
m 2t s 1
 
2 n m
t s  t 2 mt m
t ,
равенката (1) нема решение.

2) Нека m  n. Од m  t 2n  n  t m добиваме m  2n. Постои природен број d  0 така


што m  2n  d . Важи m  n  t m2n  n  t d , па затоа d  m  2n  n  t d  2  . ..(2).

Со помош на математичка индукција лесно се докажува дека 2d  2  d за секој


природен број d  3. Во ваков случај t d  2  2d  2  d , што не е можно според
(2).

Ако d  1 тогаш 1  n  t  2  од каде следува дека n  1, t  3, m  3.

Значи x  33  27, y  31  3. Ако d  2 имаме 2  n  t 2  2  , т.е. n  1, t  2, m  4.

Оттука x  t m  24  16, y  t n  21  2. Значи 1,1 ,  27,3 и 16, 2  се решенија на


равенката.

192
200 ТЕОРИЈА НА БРОЕВИ - подготвителни задачи

194. Предлог ИМО 2010. Најди ги сите парови m, n  од ненегативни цели броеви за кои


m 2  2  3n  m 2 n1  1 .
Решение. За фиксирани вредности на n дадената равенка добива облик на квадратна
равенка по m. За 0  n  5 имаме:

1) n  0. Се добива квадратна равенка m 2  m  2  0 која нема целобројни решенија.


2) n  1. Се добива квадратна равенка m 2  3m  6  0 која нема целобројни
решенија.
3) n  2. Се добива квадратна m 2  7m  18  0 која нема целобројни решенија.
4) n  3. Се добива квадратна равенка m 2  15m  54  0 која има решенија m  6 и
m  9. Оттука 6, 3 и 9, 3 се решенија на дадената равенка.
5) n  4. Се добива квадратна равенка m 2  31m  162  0 која нема целобројни
решенија.
6) n  5. Се добива квадратна равенка m 2  63m  486  0 која има решенија m  9 и
m  54. Оттука 9, 5 и 54, 5 се решенија на дадената равенка.

Ќе докажеме дека за n  6 равенката нма решение. Нека m, n  е пар кој ја задоволува
дадената равенка и притоа n  6 . Од дадената равенка добиваме 2  3n  m2 n1  1  m ,
т.е. m 2  3n. Значи m  3a , 0  a  n или m  2  3b , 0  b  n.

Во првиот случај нека n  a  c. Ја добиваме равенката 3a  2  3c  2 n1  1 и c  n  a.

Во вториот случај имаме 4  32b  2  3bd  2  3b  2n1  1  2  3b  3d  2n1  1, каде d  n  b.

Од двете равенки јасно е дека останува да се реши равенката 2n1  1  3 p  2  3q ...(1),

каде p и q се ненегативни цели броеви за кои важи p  q  n.

Од (1) добиваме:

n 1 n 1 2 n 1 n n 2n 2 n 1

3 p  2n1  8 3
9 3
3 3
и 2  3q  2n1  2  2n  2  8 3  2  9 3  2  3 3  2  3 3
.

2  n  1 2  n  1 2  n  1 n2
Според тоа имаме p, q  . Од p  следува q  n  p  n   и
3 3 3 3
2  n  1 n2
p  nq  n  .
3 3

193
200 ТЕОРИЈА НА БРОЕВИ - подготвителни задачи

n2 52
Нека r  min  p, q. Јасно r    1. Десната страна на (1) се дели со 3h , и
3 3
бидејќи r  1 следува дека 9 3r 2n1  1. Важи 26  1 mod 9  па оттука n  1  6k.

Добиваме

2n1  1  26 k  1  43k  1   4k  1 42 k  4k  1   2k  1 2k  1 42k  4k  1 .

Јасно е дека 42k  4k  1  1  1  1  0  mod 3 но не се дели со 9 бидејќи

 
42 k  4k  1  4k  1  3  4k  3  4k  mod 9 
2
и 4k не се дели со 3. Следува дека

3r 1  2k  1 2k  1 . Броевите 2k  1 и 2k  1 се непарни и заемно прости, па затоа важи

3r 1 2k  1 или 3r 1 2k  1. И во двата случаи важи 3r 1  2k  1.

n 1
n 1
Тогаш 3r 1  2k  1  3k  3 6
, од каде следува r  1  .
6

n2 n 1 n  2 n 1
Сега важи, 1  r 1   1   n  11.
3 6 3 6

Бидејќи 6 n  1 и 6  n  11 следува дека не постои таков n.

195. Предлог ИМО 2004. Нека со  n  го означуваме бројот на природни делители на


природниот број n. Докажи дека постојат бесконечно многу природни броеви a така што
равенката  an   n нема решение n.

Решение. Нека m  an. Дадената равенка се сведува на  m  


m
.
a

Ќе докажеме дека ако a  p p 1 , каде p  3 е прост број, равенката  m  


m
нема
a
решение.

Значи m  p p 1 m. Јасно p p 1 m , па затоа m  p s  k , каде k , s  N , s  p  1.

Нека k  p11  p 2 2    pr r е канонична факторизација на бројот k .

194
200 ТЕОРИЈА НА БРОЕВИ - подготвителни задачи

Важи  k   1  1   1   2     1   r . Според тоа  m  1  s  k , па дадената равенка е


ps  k
еквивалентна на равенката 1  s  k    p s  p 1  k. (1)
p p 1

Да забележиме дека s  p  1. Ако s  p  1 тогаш 1  s  k   p   k  се дели со p,

додека p s  p 1  k  k не се дели со p. Значи s  p  1 т.е. s  p, од каде следува дека

p
p s  p 1   s  1 . Бидејќи p s p1  p   s  p  1  s  1 следува дека p s  p1 не го дели
p 1
s  1 , па од 1  s    k   p s  p 1  k следува дека p   k  . Оттука постои i 1, 2,..., r така
што p si  1. Значи si  1  p т.е. si  p  1 за некое i.

pix
Бидејќи функцијата f  x   е монотоно растечка функција за x  0 следува дека
x 1
pisi p p 1 2 p 1 p si 2 p 1
 i  . Според тоа k  i   k     k .
si  1 p p si  1 p

p 2 p 1 2 p 1
Добиваме p s  p 1  k   s  1    k    s  1    k    s  1   k  ,
p 1 p p 1

што е противречност на (1).

196. Предлог ИМО 2000. За природниот број n , со d  n  го означуваме бројот на сите


природни делители на бројот n. Најди ги сите природни броеви n така што d  n   4n.
3

Решение. Нека n  p11 p22  pkk е каноничната факторизација на бројот n и


p1  p2  ...  pk се прости броеви. Бидејќи 4n е точен куб следува дека n се дели со 2 ,
т.е. p1  2. Значи d  n   21  2 p22  pkk па оттука важи 1  31  1,  2  32 ,...,  k  3k ,
3

каде i  0. Бидејќи d  n   1  1 2  1   k  1   31  2  32  1  3k  1


добиваме

31  2 32  1   3k  1  23 1 1 p232  pk3k


3 3 3

31  232  1  3k  1  2  1 p2 1 2


 pkk ....(1).

195
200 ТЕОРИЈА НА БРОЕВИ - подготвителни задачи

Јасно е дека ниеден од броевите на десната страна на (1) не се дели со 3 , па следува дека
pi  5, за секој i 2,3,..., k. Равенката (1) ја запишуваме во еквивалентна форма

31  2 p22 pkk


  ...(2).
21 1 3 2  1 3 k  1

За i 2,3,..., k  важи pii  5i  1  4  i  1  4i  1  3i .


31  2 p22 p k
Значи 1 1
  k  111  1. Лесно се докажува дека за 1  2 важи
2 3 2  1 3 k  1

31  2
21 1  31  2. Останува 1  2. Ако 1  0 или 1  2 тогаш  1,
21 1

pii
т.е.  1, i  2. Последното е можно само ако i  0, бидејќи во спротивно
3i  1
pii  5i  1  4  i  1  4i  1  3i .

Ако 1  0 тогаш 1  1 , и бидејќи i  0 кога i  2 следува i  0, i  2 . Оттука


n  21  2.

Ако 1  2 тогаш 1  7 па следува дека n  27  128.

5
Ако 1  1 тогаш левата страна на (2) е еднаква на . Од друга страна, ако pi  5 или
4
pii 5 5 5 2
i  1 тогаш  . Значи p2  5 од каде следува дека  т.е  2  1.
3i  1 4 4 3 2  1

Според тоа n  21  52  231 1  532  24  53  2000. Значи n  2,128, 2000 се бараните
броеви.

197. Предлог ИМО 2005. Најди ги сите природни броеви n  1 за кои постои единствен
цел број a така што 0  a  n ! и a n  1 се дели со n !.

Решение. Ќе докажеме дека n е прост број. Ако n  2 тогаш a  1.

Нека n  2 е парен број. Важи a n  0,1 mod 4  и 4 n ! . Бидејќи a n  1  1, 2  mod 4  и

196
200 ТЕОРИЈА НА БРОЕВИ - подготвителни задачи

4 n! a n  1 добиваме противречност.

Нека n е непарен сложен број. За бројот a  n! 1 важи 0  a  n! и n ! a n  1 .

Последното важи бидејќи a n  1   n! 1  1   1  1  0  mod n! . Бидејќи n е сложен


n n

n!
број постои прост делител p, p n. Јасно е дека 0   1  n ! . Ќе докажеме дека
p
n n n n 1 n2
 n!   n!   n !   n  n !   n  n !   n  n ! 
n !   1  1. Имаме   1  1             ...     .
p   p   p  1  p   2  p   n  1 p 
...(1).
n
 n! 
Секој од собироците во (1) е делив со n ! , оттука следува дека n !   1  1.
p 

Значи, за секој непарен сложен број n, постојат барем два природни броеви
n!
a1  n ! 1, a2   1 за кои важи 0  a1 2  n! и n! a1, 2 n  1 .
p

Ако n е прост број ќе докажеме дека a  n! 1 е единствен број за кои се исполнети
условите на задачата.

Сега ќе докажеме дека ако n е непарен прост број за кој n ! a n  1 тогаш n ! a  1, од каде
следува a  n! 1 е единствен цел број за кој важи 0  a  n ! и n ! a n  1.

an  1
Нека p  n е прост број за кој важи p . Јасно a n  1 mod p  па оттука
a 1
 a    1  a n   1   1  1 mod p  p  a   1.
n n n n
односно Јасно е дека
НЗД  n , p 1
НЗД  a, p   1 па важи p  a   1. Според тоа p  a 
p 1
 1 т.е. p  a  1.

an  1
Значи a  1 mod p  . ..(1). Важи  a n 1  a n 2  ...  a  1  n  0  mod p  , па следува
a 1
an  1
дека n  p. Бидејќи n е единствен прост број за кој n следува дека
a 1
 аn  1  an  1
НЗД  ,  n  1!  1. Од n ! a n  1 следува дека  n  1! n   a  1 , па затоа
 a 1  a 1

197
200 ТЕОРИЈА НА БРОЕВИ - подготвителни задачи

 n  1! a  1. Од (1) добиваме дека n a  1. Бидејќи  n  1! и n се заемно прости броеви


следува дека n!  (n  1)! n a  1, што требаше да се докаже.

198. Предлог ИМО 1989. Нека m е непарен природен број, m  3. Најди го најмалиот
го дели m n  1.
1989
природен број n таков што 2

Решение. Нека n е бараниот експонент и нека претпоставиме n  2 a  b , каде b е


непарен број и a  0.

Имаме m n  1  m 2
a
b
 1  m2    1  m
a b
b 2a

 1 m2
a
b 1
 m2
a
b  2 
  
 ....  m 2  1  m 2  1  c.
a a

b 1 b 2 
Јасно c  m 2  m2  ....  m 2  1 е збир од b ( непарен број) непарни собироци,
a a a

па следува c е непарен број.

   
Од 21989 m n  1 следува 21989 m 2 1  c т.е. 21989 m 2  1 . Оттука n  2 a .
a
 a

Важи
m n  1  m 2  1   m 2
a


a 1
  2

 12   m 2

a 1
1 m2  a 1
  
 1  ....  m 2  1 m 2  1 m 4  1    m 2   a 1

1.

Нека s е најголемиот природен број за кој m  1 mod 2s  . ..(1).

Јасно m2  1   m  1 m  1 се дели со 2s1 бидејќи m  1 и m  1 се парни броеви и

 
m  1 mod 2s . Од (1) важи m4  1  2  mod 2s  , m8  1  2  mod 2s  ,..., m2  1  2  mod 2s .
a1

a1
Очигледно е дека броевите m4  1, m8  1,..., m2  1 се делат со 2 , но не се делат со 4.

Според тоа mn  1 се дели со 2s 1  2  2  ...  2  2a  s но не се дели со 2a  s 1.


a 1

Бидејќи 21989 m2  1 следува дека a  s  1989 т.е. a  1989  s. Значи n  21989s , ако
a

s  1989 и n  1 ако s  1989.

198
200 ТЕОРИЈА НА БРОЕВИ - подготвителни задачи

199. Предлог ИМО 1998. Определи ги сите природни бреови n за кои постои природен
број m така што 2n  1 е делител на m2  9.

Решение Нека претпоставиме дека n има непарен делител d  1. Тогаш 2d  1 2n  1 , па


од условот на задачата следува 2d  1 m2  9. Бидејќи 2d  1  0  1  1 mod 4  постои
прост делител p на 2d  1 кој е во облик 4k  1. Во спротивно, ако сите прости делители
на 2d  1 се од облик 4k  1 тогаш и 2d  1 е од таков облик.

Лема: Ако p  4k  1 е прост број и p a 2  b2 тогаш p a и p b.

Доказ. Нека p  4k  1 е прост број, p a 2  b2 и нека претпоставуваме дека a и b не се


делат со p. Следува НЗД  p, a   1  НЗД  p, b  . Од теорема на Ферма имаме

a p 1  a 4k 2  1 mod p  и b p 1  b4k 2  1 mod p  .

Оттука a p 1  b p 1   a 2   
2 k 1 2 k 1
 b2  2  mod p  ..(1).

Бидејќи p a 2  b2  a 2   
2 k 1 2 k 1
 b2 следува дека a p 1  b p 1  0  mod p  ..(2).

Од (1) и (2) добиваме дека 2  0  mod p  т.е. p  2  4k  1. Следува дека p a или p b.

Од p a 2  b2 добиваме дека p a и p b.

Сега, од p m2  32 следува дека p m и p 3. Останува p  3 и 3 2d  1. Последното не


е можно бидејќи кога d е непарен број важи 2d  2  mod 3 . Затоа n  2t , за некој
природен број t. Значи n нема непарен делител.

Со помош на математичка индукција по степенот t ќе ја докажеме егзистенцијата на m.

За t  1 имаме 2n  1  3 и избираме m  3.

Сега, нека за секој 2  t  s постои природн број m така што 22  1 е делител на m2  9.


t

    
s 1 2
Имаме 22  1  22  1  22  1 22  1 .
s s s

Од индуктивната претпоставка постои природен број ms така што 22  1 ms 2  9.


s

199
200 ТЕОРИЈА НА БРОЕВИ - подготвителни задачи

     
2
Важи 22  1 9 22  1 и 9 22  1  3  22
s s
 32.
s s1

s 1
Нека ms'  3  22 и го разгледуваме системот од конгруентни равенки:


m  m mod 22s  1
 s 
 (1)

m  ms' mod 22  1 .

s


 
Бидејќи НЗД 22  1, 22  1  1 , од кинеската теорема за остатоци следува дека системот
s s

(1) има единствено решение m по модул 22  1 22  1 .  s

 s


 
Од (1) имаме m2  9  ms2  9 mod 22  1 , оттука 22  1 m2  9. На ист начин 22  1 m2  9.
s s s


Бидејќи НЗД 22  1, 22
s s
 1  1 следува  2 2s
 
 1 22  1 m2  9 т.е. 22  1 m2  9.
s s 1

Со тоа индукцијата е завршена. Значи n  2t , каде t е позитивен цел број.

200. Предлог ИМО 2012. За целиот број a велиме дека е пријателски ако равенката
m 2
 
 n n 2  m  am  n има решение во множеството природни броеви.
3

а) Докажи дека постојат најмалку 500 пријателски броеви во множеството


1, 2, ...., 2012.
б) Провери дали бројот a  2 е пријателски.

Решение. а) Лесно се проверува дека секој број a  4k  3 , каде k  2, е пријателски број.


Во ваков случај броевите m  2k  1  0 и n  k  1  0 се решение на дадената равенка.

Важи

m 2
   
 n n 2  m  4k 2  4k  1  k  1 k 2  2k  1  2k  1  4k 2  3k k 2  4k  3k 3  am  n .   3

Броевите 5  4  2  3, 9,13,17, 21,.., 2009  4  503  3 се од облик 4k  3 и нивниот број


изнесува 502.

б) Јасно е дека m  n. Од  m2  n    n2  m    m  n  m  n  1 и ако земиме y  n2  m

добиваме m2  n  y   m  n  m  n  1 . Оттука важи y  y   m  n  m  n  1   2  m  n 


3

200
200 ТЕОРИЈА НА БРОЕВИ - подготвителни задачи

т.е. ја добиваме квадратната равенка y 2  y  m  n  m  n  1  2  m  n   0.


3

Бидејќи y  N следува

D   m  n   m  n  1  8  m  n    m  n 
2 2 3 2
 m  n 1  8 m  n 
2

е полн квадрат. Останува дека  m  n  1  8  m  n  мара да биде полн квадрат.


2

Јасно е дека  m  n  1   m  n  1  8  m  n   m2  n2  6m  10n  2mn  1   m  n  3 .


2 2 2

Значи,  m  n  1  8  m  n    m  n  1 или  m  n 1  8  m  n    m  n  2 .


2 2 2 2

Во првиот случај се добива m  3n и ако замениме во m 2


 
 n n2  m  2  m  n  се
3

добива равенката 3n2  4n  1  0 која нема решение во N .

Во вториот случај се добива равенката 2m  14n  3 која нема решение бидејќи 2m е


парен број додека 14n  3 е непарен број.

Според тоа равенката m 2


 
 n n2  m  2  m  n 
3
нема решение во множеството
природни броеви, т.е. a  2 не е пријателски број.

201
Литература
1. Andreescu, T., Feng, Z., Andrica, D., 104 Number Theory Problems

2. Kuczma, M., International Mathematical Olympiads 1986-1999, Mathematical Association of


America, 2003.

3. Doob, M., The Canadian Mathematical Olympiad 1969-1993, University of Toronto Press, 1993.

4. Rassias, M., Problem-Solving and Selected Topics in Number Theory, Springer 2010.

5. Andreescu, T., Andrica, D., Number Theory, Birkhauser, Boston 2009.

6. Djukic, D., Jankovic, V., Matic, I., Petrovic, N., The IMO Compendium, Springer-Verlag, New
York, 2006.

7. Koblitz, M., A Course in Number Theory and Cryptography, Springer-Verlag, New York, 1994.

8. Erdos, P., Suranyi, J., Topics in the Theory of Numbers, Springer-Verlag, New York, 2003.

9. Mollin, R., An Introduction to Cryptography, Champman and Hall, 2007.

10. Crandall, R., Pomerance, C., Prime Numbers, Springer, 2005.

11. Shortlisted Problems IMO 2005-2006, 2006.

12. Shortlisted Problems IMO 2007, 2007.

13. Shortlisted Problems IMO 2008, 2008.

14. Shortlisted Problems IMO 2009, 2009.

15. Shortlisted Problems IMO 2010, 2010.

16. Филиповски, С., Збирка подготвителни задачи за натпревари, 2013.

17. Chau, L.H., Khoi, L.H., Selected Problems of the Vietnamese Mathematical Olympiad (1962-
2009), 2009.

18. www.artofproblemsolving.com/Forum/resources.php

19. Kumar, S. A., Algorithmic Number Theory, 2002.

20. Burton, D. M., Elementary Number Theory, Sixth Edition, McGraw Hill, 2010

21. Clark, W. E., Elementary Number Theory, University of South Florida, 2002.

22. Guy, R. K., Unsolved Problems in Number Theory, 2nd edition, Springer – Verlag, New York,
1994.

202
200 ТЕОРИЈА НА БРОЕВИ - подготвителни задачи

Додаток

1) Нека a1 , a2 ,..., an се позитивни реални броеви. Тогаш важат следните неравенства:

a12  a22  ...  an2 a1  a2  ...  an n n


  a1a2  an 
n n 1 1 1
  ...  .
a1 a2 an

2) Нека a1 , a2 ,..., ak , m, n се позитивни реални броеви и m  n.

Неравенство помеѓу средини од ред m и ред n за броевите a1 , a2 ,..., ak :

a1m  a2m  ...  akm n a1n  a2n  ...  akn


m  .
k k

3) Нека x  1 е реален број и r е позитивен рационален број.

Неравенство на Бернули: 1  x   1  rx.


r

4) За тројката природни броеви  a, b, c  велиме дека е Питагорина тројка ако


a b  c .
2 2 2

5) Нека x е реален број и нека k е цел број таков што k  x  k  1.

Функција цел дел од x означуваме со  x  и важи  x   k .

Функција дробен (децимален) дел од x означуваме со  x и важи  x  x   x .

6) Нека 1  d1  d2  ....  dk  n се природни делители на n. Ако d1  d2  ...  dk  2n


тогаш за бројот n велиме дека е совршен или перфектен.

7) Fn  22  1 - број на Ферма.
n

8) Идентитет на Хермит:

 n  1
 x  1 
1  2
  1    ....  1    nx .
 n  n  n 

203
ИМО – Интернационална Математичка Олимпијада

БМО – Балканска Математичка Олимпијада

ЈБМО – Јуниорска Балканска Математичка Олимпијада

ММО – Македонска Математичка Олимпијада

ЈММО – Јуниорска Македонска Математичка Олимпијада

УМО – Унгарска Математичка Олимпијада

СМО – Српска Математичка Олимпијада

204

You might also like